You are on page 1of 146

Calculus - I

M.Thamban Nair
Department of Mathematics
Indian Institute of Technology Madras

June 2006/Aug-Dec. 2010/Aug-Dec.2011

Contents

Preface

vi

1 Sequence and Series of Real Numbers


1.1

1.2

Sequence of Real Numbers . . . . . . . . . . . . . . . . . . . . . . . .

1.1.1

Convergence and divergence . . . . . . . . . . . . . . . . . . .

1.1.2

Monotonic sequences . . . . . . . . . . . . . . . . . . . . . . .

1.1.3

Subsequences . . . . . . . . . . . . . . . . . . . . . . . . . . .

1.1.4

Further examples . . . . . . . . . . . . . . . . . . . . . . . . .

11

1.1.5

Cauchy sequence . . . . . . . . . . . . . . . . . . . . . . . . .

16

1.1.6

Additional exercises . . . . . . . . . . . . . . . . . . . . . . .

19

Series of Real Numbers . . . . . . . . . . . . . . . . . . . . . . . . . .

21

1.2.1

Convergence and divergence of series . . . . . . . . . . . . . .

21

1.2.2

Some tests for convergence . . . . . . . . . . . . . . . . . . .

23

1.2.3

Alternating series . . . . . . . . . . . . . . . . . . . . . . . . .

26

1.2.4

Absolute convergence . . . . . . . . . . . . . . . . . . . . . .

27

1.2.5

Additional exercises . . . . . . . . . . . . . . . . . . . . . . .

30

2 Limit, Continuity and Differentiability of Functions


2.1

M.T. Nair

M.T. Nair 32

Limit of a Function . . . . . . . . . . . . . . . . . . . . . . . . . . . .

32

2.1.1

Limit point of a set D R . . . . . . . . . . . . . . . . . . .

32

2.1.2

Limit of a function f (x) as x approaches a . . . . . . . . . .

33

Limit of a function in terms of a sequences . . . . . . . . . .

35

2.1.3

Some properties . . . . . . . . . . . . . . . . . . . . . . . . .

36

2.1.4

Some Examples . . . . . . . . . . . . . . . . . . . . . . . . . .

37

ii

Contents

2.2

2.1.5

Left limit and right limit

. . . . . . . . . . . . . . . . . . . .

39

2.1.6

Limit at and at . . . . . . . . . . . . . . . . . . . . .

40

2.1.7

Additional Exercises . . . . . . . . . . . . . . . . . . . . . . .

42

Continuity of a Function . . . . . . . . . . . . . . . . . . . . . . . . .

43

2.2.1

Definition and some basic results . . . . . . . . . . . . . . . .

43

2.2.2

Some examples . . . . . . . . . . . . . . . . . . . . . . . . . .

44

2.2.3

Exponential and logarithm functions . . . . . . . . . . . . . .

46

2.2.4

Some properties of continuous functions . . . . . . . . . . . .

51

Attaining max f and min f . . . . . . . . . . . . . . . . . . .

52

Intermediate value theorem . . . . . . . . . . . . . . . . . . .

52

Additional Exercises . . . . . . . . . . . . . . . . . . . . . . .

53

Differentiability of functions . . . . . . . . . . . . . . . . . . . . . . .

54

2.3.1

Some properties of differentiable functions . . . . . . . . . . .

55

2.3.2

Some Examples . . . . . . . . . . . . . . . . . . . . . . . . . .

57

2.3.3

Maxima and minima . . . . . . . . . . . . . . . . . . . . . . .

59

2.3.4

Some important theorems . . . . . . . . . . . . . . . . . . . .

61

Rolles theorem . . . . . . . . . . . . . . . . . . . . . . . . . .

61

Lagranges mean value theorem . . . . . . . . . . . . . . . . .

62

Cauchys generalized mean value theorem . . . . . . . . . . .

63

LHospitals rules . . . . . . . . . . . . . . . . . . . . . . . . .

64

Taylors formula . . . . . . . . . . . . . . . . . . . . . . . . .

66

2.3.5

Increasing and decreasing functions . . . . . . . . . . . . . . .

70

2.3.6

More about local maxima and local minima . . . . . . . . . .

71

2.3.7

Additional exercises . . . . . . . . . . . . . . . . . . . . . . .

73

2.2.5
2.3

iii

3 Definite Integral

M.T. Nair 74

3.1

Introduction . . . . . . . . . . . . . . . . . . . . . . . . . . . . . . . .

74

3.2

Lower and Upper Sums . . . . . . . . . . . . . . . . . . . . . . . . .

75

3.3

Integrability and Integral . . . . . . . . . . . . . . . . . . . . . . . .

76

3.3.1

78

Some necessary and sufficient conditions for integrability . . .

iv

Contents
3.4

Integral of Continuous Functions . . . . . . . . . . . . . . . . . . . .

83

3.5

Some Properties . . . . . . . . . . . . . . . . . . . . . . . . . . . . .

86

3.6

Some Results . . . . . . . . . . . . . . . . . . . . . . . . . . . . . . .

88

3.6.1

First fundamental theorem . . . . . . . . . . . . . . . . . . .

88

3.6.2

Second fundamental theorem . . . . . . . . . . . . . . . . . .

88

3.6.3

Applications of fundamental theorem . . . . . . . . . . . . . .

90

3.7

Appendix

M.T. Nair . . . . . . . . . . . . .

4 Improper Integrals
4.1

92

M.T. Nair 93

Definitions . . . . . . . . . . . . . . . . . . . . . . . . . . . . . . . . .

93

4.1.1

Examples . . . . . . . . . . . . . . . . . . . . . . . . . . . . .

95

Integrability by Comparison . . . . . . . . . . . . . . . . . . . . . . .

96

4.2.1

Examples . . . . . . . . . . . . . . . . . . . . . . . . . . . . .

97

4.2.2

Gamma and Beta Functions . . . . . . . . . . . . . . . . . . .

98

4.3

Integrability Using Limits . . . . . . . . . . . . . . . . . . . . . . . .

99

4.4

Additional Exercises . . . . . . . . . . . . . . . . . . . . . . . . . . . 100

4.2

5 Geometric and Mechanical Applications of Integrals


5.1

5.2

M.T. Nair101

Computing Area . . . . . . . . . . . . . . . . . . . . . . . . . . . . . 101


5.1.1

Using Cartesian Coordinates . . . . . . . . . . . . . . . . . . 101

5.1.2

Using Polar Coordinates . . . . . . . . . . . . . . . . . . . . . 102

5.1.3

Some Examples . . . . . . . . . . . . . . . . . . . . . . . . . . 102

Computing Arc Length

. . . . . . . . . . . . . . . . . . . . . . . . . 103

5.2.1

Using Cartesian Coordinates . . . . . . . . . . . . . . . . . . 103

5.2.2

Using Polar Coordinates . . . . . . . . . . . . . . . . . . . . . 104

5.2.3

Examples . . . . . . . . . . . . . . . . . . . . . . . . . . . . . 104

5.3

Computing Volume of a Solid . . . . . . . . . . . . . . . . . . . . . . 106

5.4

Computing Volume of a Solid of Revolution . . . . . . . . . . . . . . 107

5.5

Computing Area of Surface of Revolution . . . . . . . . . . . . . . . 108

5.6

Centre of Gravity . . . . . . . . . . . . . . . . . . . . . . . . . . . . . 108


5.6.1

Centre of gravity of a material line in the plane . . . . . . . . 109

Contents
5.6.2
5.7

5.8

Centre of gravity of a material planar region . . . . . . . . . 110

Moment of Inertia . . . . . . . . . . . . . . . . . . . . . . . . . . . . 111


5.7.1

Moment of inertia of a material line in the plane . . . . . . . 111

5.7.2

Moment of inertia of a circular arc with respect to the centre

5.7.3

Moment of inertia of a material sector in the plane . . . . . . 113

112

Additional Exercises . . . . . . . . . . . . . . . . . . . . . . . . . . . 113

6 Sequence and Series of Functions


6.1

M.T. Nair116

Sequence of Functions . . . . . . . . . . . . . . . . . . . . . . . . . . 116


6.1.1

Pointwise Convergence and Uniform Convergence . . . . . . . 116

6.1.2

Continuity and uniform convergence . . . . . . . . . . . . . . 121

6.1.3

Integration and uniform convergence . . . . . . . . . . . . . . 122

6.1.4

Differentiation and Uniform Convergence . . . . . . . . . . . 122

6.2

Series of Functions . . . . . . . . . . . . . . . . . . . . . . . . . . . . 123

6.3

Additional Exercises . . . . . . . . . . . . . . . . . . . . . . . . . . . 126

7 Power Series

M.T. Nair128

7.1

Convergence and Absolute convergence . . . . . . . . . . . . . . . . . 128

7.2

Uniform Convergence

7.3

Differentiation and Integration . . . . . . . . . . . . . . . . . . . . . 130


7.3.1

7.4

Series that can be converted into a power series . . . . . . . . 132

Additional Exercises . . . . . . . . . . . . . . . . . . . . . . . . . . . 132

8 Fourier Series
8.1

8.2

M.T. Nair133

Fourier Series of 2-Periodic functions . . . . . . . . . . . . . . . . . 133


8.1.1

Fourier series and Fourier coefficients . . . . . . . . . . . . . . 133

8.1.2

Even and odd expansions . . . . . . . . . . . . . . . . . . . . 135

Fourier Series of 2`-Periodic Functions . . . . . . . . . . . . . . . . . 138


8.2.1

8.3

. . . . . . . . . . . . . . . . . . . . . . . . . . 130

Fourier series of functions on arbitrary intervals . . . . . . . . 138

Additional Exercises . . . . . . . . . . . . . . . . . . . . . . . . . . . 139

Preface
This is based on a course that the author gave to B.Tech. students of IIT Madras
many times since 1996 for the use of students and teachers of IIT Madras.
Comments and suggestions from the readers are welcome.
June 2006

M. T. Nair
mtnair@iitm.ac.in

Revised: August-November 2010


Revised: August-November 2011

vi

1
Sequence and Series of Real Numbers

1.1

Sequence of Real Numbers

Suppose for each positive integer n, we are given a real number an . Then, the list
of numbers,
a1 , a2 , . . . , an , . . . ,
is called a sequence, and this ordered list is usually written as
(a1 , a2 , . . . , . . .)

or

(an ).

More precisely, we define a sequence as follows:


Definition 1.1 A sequence of real numbers is a function from the set N of natural
numbers to the set R of real numbers. If f : N R is a sequence, and if an = f (n)
for n N, then we write the sequence f as (an ) or (a1 , a2 , . . .).
A sequence of real numbers is also called a real sequence.

Remark 1.1 (a) It is to be born in mind that a sequence (a1 , a2 , . . . , . . .) is different


from the set {an : n N}. For instance, a number may be repeated in a sequence
(an ), but it need not be written repeatedly in the set {an : n N}. As an example,
(1, 1/2, 1, 1/3, . . . , 1, 1/n, . . .) is a sequence (an ) with a2n1 = 1 and a2n = 1/(n + 1)
for each n N, where as the set {an : n N} is same as the set {1/n : n N}.
(b) Instead of sequence of real numbers, we can also talk about a sequence of
elements from any nonempty set S, such as sequence of sets, sequence of functions
and so on. Thus, given a nonempty set S, a sequence in S is a function f : N S.
For example, for each n N, consider the set An = {j N : j n}. Then we
obtain a sequence of subsets of N, namely, (A1 , A2 , . . .).
In this chapter, we shall consider only sequence of real numbers. In some of the
later chapters we shall consider sequences of functions as well.

EXAMPLE 1.1 (i) (an ) with an = 1 for all n N a constant sequence with
value 1 throughout.
(ii) (an ) with an = n for all n N.
(iii) (an ) with an = 1/n for all n N.
1

Sequence and Series of Real Numbers

M.T. Nair

(iv) (an ) with an = n/(n + 1) for all n N.


(v) (an ) with an = (1)n for all n N the sequence takes values 1 and 1
alternately.

Question: Consider a sequence (a1 , a2 , . . .). Is (a2 , a3 , . . .) also a sequence? Why?

1.1.1

Convergence and divergence

A fundamental concept in mathematics is that of convergence. We consider convergence of sequences.


Consider the sequences listed in Example 1.1 and observe the way an vary as n
becomes larger and larger:
(i) an = 1: every term of the sequence is same.
(ii) an = n: the terms becomes larger and larger.
(iii) an = 1/n: the terms come closer to 0 as n becomes larger and larger.
(iv) an = n/(n + 1): the terms come closer to 1 as n becomes larger and larger.
(v) an = (1)n : the terms of the sequence oscillates between 1 and 1, and does
not come closer to any number as n becomes larger and larger.
Now, we make precise the statement an comes closer to a number a as n
becomes larger and larger.
Definition 1.2 A sequence (an ) in R is said to converge to a real number a if for
every > 0, there exists a natural number N (in general depending on ) such that
|an a| <

n N,

number a is called the limit of the sequence (an ).

Remark 1.2 (a) Note that, different can result in different N , i.e., the number
N may vary as varies. We shall illustrate this in Example 1.2.
(b) In Definition 1.2, the relation |an a| < can be replaced by |an a| < c0
for any c0 > 0 which is independent of n. In other words, a sequence (an ) in R
converges to a R if and only if for any c0 > 0 and for every > 0, there exists
N N such that |an a| < c0 for all n N .

If (an ) converges to a, then we write
lim an = a

or an a

or simply as
an a.

as

Sequence of Real Numbers

Note that
|an a| <

n N

if and only if
a < an < a +

n N.

Thus, limn an = a if and only if for every > 0, there exists N N such that
an (a , a + )

n N.

Thus, an a if and only if for every > 0, an belongs to the open interval
(a, a+) for all n after some finite stage, and this finite stage may vary according
as varies.
Remark 1.3 Suppose (an ) is a sequence and a R. The to show that (an ) does
not converge to a, we should be able to find an > 0 such that infinitely may an s
are outside the interval (a , a + ).

Exercise 1.1 Show that a sequence (an ) converges to a if and only if for every open
interval I containing x, there exits N N such that an I for all n N .
J
EXAMPLE 1.2 The sequences (1/n), ((1)n /n), (1 n1 ) are convergent with
limit 0, 0, 1 respectively:
For the sake of illustrating how to use the definition to justify the above statement, let us provide the details of the proofs:
(i) Let an = 1/n for all n N, and let > 0 be given. We have to identify an
N N such that 1/n < for all n N . Note that
1
1
< n > .
n

 
1
Thus, if we take N =
+ 1, then we have

|an 0| =

1
<
n

n N.

Hence, (1/n) converges to 0.


Here dxe denotes the integer part of x.
(ii) Next, let an = (1)n /n for all n N. Since |an | = 1/n for all n N, in this
case also, we see that
 
1
|an 0| <
n N :=
+ 1.

Hence, ((1)n /n) converges to 0.

Sequence and Series of Real Numbers

(iii) Now, let an = 1


have

M.T. Nair

1
n

for all n N. Since, |an 1| = 1/n for all n N, we


 
1
|an 1| <
n N :=
+ 1.

Hence, (1 1/n) converges to 0.

EXAMPLE 1.3 Every constant sequence is convergent to the constant term in


the sequence.
To see this, let an = a for all n N. Then, for every > 0, we have
|an a| = 0 <

n N := 1.

Thus, (an ) converges to a.

EXAMPLE 1.4 For a given k N, let Let an = 1/n1/k for all n N. Then an 0
as n .
To see this, first let > 0 be given. Note that
1
1
< n > k .

n1/k
Hence,
1
n1/k

<


1
n N := k + 1.

Thus, 1/n1/k 0.

Exercise 1.2 Corresponding to a sequence (an ) and k N, let (bn ) be defined by


bn = ak+n for all n N. Show that, for a R, an a if and only if bn a.
J
Exercise 1.3 A sequence (an ) is said to be eventually constant if there exists
k N such that ak+n = ak for all n 1. Show that every eventually constant
sequence converges.
J
Theorem 1.1 Limit of a convergent sequence is unique. That, is if an a and
an a0 as n , then a = a0 .
Proof. Suppose an a and an a0 as n , and suppose that a0 6= a. Now,
for > 0, suppose N1 , N2 N be such that
an (a , a + ) n N1 ,

an (a0 , a0 + )

n N2 .

In particular,
an (a , a + ),

an (a0 , a0 + ) n N := max{N1 , N2 }.

If we take < |a a0 |/2, then we see that (a , a + ) and (a0 , a0 + ) are


disjoint intervals. Thus the above observation leads to a contradiction.

Sequence of Real Numbers

An alternate proof. Note that


|a a0 | = |(a an ) + (an a0 )| |a an | + |an a0 |.
Now, for > 0, let N1 , N2 N be such that
|a an | < /2

for all

n N1 ,

|a0 an | < /2 n N2 .

Then it follows that


|a a0 | |a an | + |an a0 | <

n N := max{N1 , N2 }.

Since this is true for all > 0, it follows that a0 = a.


Exercise 1.4 Prove the following.
1. Let b 0 such that b < for all > 0. Then b = 0.
2. Let an 0 for all n N such that an a. Then a 0.
J
The following theorem can be easily proved.
Theorem 1.2 Suppose an a, bn b as n . Then we have the following :
(a)

an + bn a + b as n ,

(b)

For every real number c, can c a as n

(c) If an bn for all n N, then a b.


(d) (Sandwitch theorem) If an cn bn for all n N, and if a = b, then
cn a as n .
Exercise 1.5 Prove Theorem 1.2.

Exercise 1.6 If an a and there exists b R such that an b for all n N, then
show that a b.
J
Exercise 1.7 If an a and a 6= 0, then show that there exists k N such that
an 6= 0 for all n k.
J

1/n
1
, n N. Then
Exercise 1.8 Consider the sequence (an ) with an = 1 +
n
show that lim an = 1.
n

[Hint: Observe that 1 an (1 + 1/n) for all n N.]

1
Exercise 1.9 Consider the sequence (an ) with an = k , n N. Then show that
n
for any given k N, lim an = 0.
n

[Hint: Observe that 1 an 1/n for all n N.]

Sequence and Series of Real Numbers

M.T. Nair

Definition 1.3 A sequence which does not converge is called a divergent


sequence.

Definition 1.4 (i) If (an ) is such that for every M > 0, there exists N N such
that
an > M
n N,
then we say that (an ) diverges to +.
(ii) If (an ) is such that for every M > 0, there exists N N such that an < M
for all n N , then we say that (an ) diverges to .

Definition 1.5 If (an ) is such that an an+1 < 0 for every n N, that is an changes
sign alternately, then we say that (an ) is an alternating sequence .

An alternating sequence converge or diverge. For example, (Verify that) the
sequence ((1)n ) diverges, whereas ((1)n /n) converges to 0.
Definition 1.6 A sequence (an ) is said to be bounded above if there exists a
real number M such that an M for all n N; and the sequence (an ) is said to be
bounded below if there exists a real number M 0 such that an M 0 for all n N.
A sequence which is bound above and bounded below is said to be a bounded
sequence.

Exercise 1.10 Show that a sequence (an ) is bounded if and only if there exists
M > 0 such that |an | M for all n N.
J
Exercise 1.11 Prove the following.
1. If (an ) is not bounded above, then there exists a strictly increasing sequence
(kn ) of natural numbers such that akn + as n .
2. If (an ) is not bounded below, then there exists a strictly increasing sequence
(kn ) of natural numbers such that akn as n .
J
Theorem 1.3 Every convergent sequence is bounded. The converse is not true.
Proof. Suppose (an ) converges to x. Then there exists N N such that |an x|
1 for all n N . Hence
|an | = |(an a) + a| |an a| + |a| 1 + |a|

n N,

so that
|an | max{1 + |a|, |a1 |, |a2 |, . . . , |aN 1 |}

n N.

To see that the converse of the theorem is not true, consider the sequence ((1)n ).
It is a bounded sequence, but not convergent.

Sequence of Real Numbers

The above theorem can be used to show that certain sequence is not convergent,
as in the following example.
EXAMPLE 1.5 For n N, let
an = 1 +

1 1
1
+ + ... + .
2 3
n

Then (an ) diverges: To see this, observe that


a2n

1 1
1
+ + ... + n
2 
3
2 

1 1 1 1
1
1 1
+
+
= 1+ +
+
+ + +
2
3 4
5 6 7 8


1
1
... +
+ ... + n
n
2 1
2
n
1+ .
2
= 1+

Hence, (an ) is not a bounded sequence, so that it diverges.

Using Theorem 1.3, the following result can be deduced


Theorem 1.4 Suppose an a and bn b as n . Then we have the following.
(i) an bn ab as n .
(ii) If bn 6= 0 for all n N and b 6= 0, then an /bn a/b as n .
Exercise 1.12 Prove Theorem 1.4.

Exercise 1.13 If (an ) converges to a and a 6= 0, then show that there exists k N
such that |an | |a|/2 for all n k, and (1/an+k ) converges to 1/a.
J

1.1.2

Monotonic sequences

We can infer the convergence or divergence of a sequence in certain cases by observing the way the terms of the sequence varies.
Definition 1.7 Consider a sequence (an ).
(i) (an ) is said to be monotonically increasing if an an+1 for all n N.
(ii) (an ) is said to be monotonically decreasing if an an+1 for all n N.
If strict inequality occur in (i) and (ii), then we say that the sequence is strictly
increasing and strictly decreasing, respectively.
Also, a monotonically increasing (respectively, monotonically decreasing) sequence is also called an increasing (respectively, a decreasing) sequence.

We shall make use of some important properties of the set R of real numbers.

Sequence and Series of Real Numbers

M.T. Nair

Definition 1.8 Let S be a subset of R. Then


(i) S is said to be bounded above if there exists b R such that x b for all
x S, and in that case b is called an upper bound of S;
(ii) S is said to be bounded below if there exists a R such that a x for all
x S, and in that case a is called a lower bound of S.

Definition 1.9 Let S be a subset of R.
(i) A number b0 R is called a least upper bound (lub) or supremum of S R
if b0 is an upper bound of S and for any upper bound b of S, b0 b.
(ii) A number a0 R is called a greatest lower bound (glb) or infimum of S R
if a0 is a lower bound of S and for any lower bound a of S, a a0 .

Thus, we have the following:
b0 R is supremum of S R if and only if b0 is an upper bound of S and if
< b0 , then is not an upper bound of S, i.e., there exists x S such that < x.
a0 R is infimum of S R if and only if a0 is a lower bound of S and if
> a0 , then is not a lower bound of S, i.e., there exists x S such that x < .
The above two statements can be rephrased as follows:
b0 R is supremum of S R if and only if b0 is an upper bound of S and for
every > 0, there exists x S such that b0 < x b0 .
a0 R is infimum of S R if and only if a0 is a lower bound of S and for
every > 0, there exists x S such that a0 x < a0 + .
Exercise 1.14 Show that supremum (respectively, infimum) of a set S R, if
exists, is unique.
J
Exercise 1.15 Prove the following:
(i) If b0 is the supremum of S, then there exists a sequence (xn ) in S which
converges to b0 .
(ii) If a0 is the infimum of S, then there exists a squence (xn ) in S which
converges to a0 .
J
EXAMPLE 1.6 (i) If S is any of the intervals (0, 1), [0, 1), (0, 1], [0, 1], then 1 is
the supremum of S and 0 is the infimum of S.
(ii) If S = { n1 : n N}, then 1 is the supremum of S and 0 is the infimum of S.
(iii) For k N, if Sk = {n N : n k}, then k is the infimum of Sk , and Sk
has no supremum.
(iv) For k N, if Sk = {n N : n k}, then k is the supremum of Sk , and Sk
has no infimum.


Sequence of Real Numbers

The above examples show the following:


The supremum and/or infimium of a set S, if exists, need not belong to S.
If S is not bounded above, then S need not have supremum.
If S is not bounded below, then S need not have infimum.
However, we have the following properties for R:
Least upper bound property: If S R is bounded above, then S has a least
upper bound. We may write this least upper bound as lub(S) or sup(S).
Greatest lower bound property: If S R is bounded below, then S has a
greatest lower bound, and we write it as glb(S) or inf(S).
Theorem 1.5 (i) Every sequence which is monotonically increasing and bounded
above is convergent.
(ii) Eery sequence which is monotonically decreasing and bounded below is convergent.
Proof. Suppose (an ) is a monotonically increasing sequence of real numbers
which is bounded above. Then the set S := {an : n N} is bounded above.
Hence, by the least upper bound property of R, S has a least upper bound, say b.
Now, let > 0 be give. Then, by the definition of the least upper bound, there
exists N N such that aN > b . Since an aN for every n N , we get
b < aN an b < b +

n N.

Thus we have proved that an b as n .


To see the last part, suppose that (bn ) is a monotonically decreasing sequence
which is bounded below. Then, it is seen that the sequence (an ) defined by an = bn
for all n N is monotonically increasing and bounded above. Hence, by the first
part of the theorem, an a for some a R. Then, bn b := a.
Note that a convergent sequence need not be monotonically increasing or monotonically decreasing. For example, look at the sequence ((1)n /n).

1.1.3

Subsequences

Definition 1.10 A sequence (bn ) is called a subsequence of a sequence (an ) if there


is a strictly increasing sequence (kn ) of natural numbers such that bn = akn for all
n N.

Thus, subsequences of a real sequence (an ) are of the form (akn ), where (kn ) is
a strictly increasing sequence natural numbers.

10

Sequence and Series of Real Numbers

M.T. Nair

For example, given a sequence (an ), the sequences (a2n ), (a2n+1 ), (an2 ), (a2n ) are
some of its subsequences. As concrete examples, (1/2n), and (1/(2n + 1)), (1/2n )
are subsequences of (1/n).
A sequence may not converge, but it can have convergent subsequences. For
example, we know that the sequence ((1)n ) diverges, but the subsequences (an )
and (bn ) defined by an = 1, bn = 1 for all n N are convergent subsequences of
((1)n ).
However, we have the following result.
Theorem 1.6 If a sequence (an ) converges to x, then all its subsequences converge
to the same limit x.
Exercise 1.16 Prove Theorem 1.6.

What about the converse of the above theorem? Obviously, if all subsequences
of a sequence (an ) converge to the same limit x, then (an ) also has to converge to
x, as (an ) is a subsequence of itself.
Suppose every subsequence of (an ) has at least one subsequence which converges
to x. Does the sequence (an ) converges to x? The answer is affirmative, as the
following theorem shows.
Theorem 1.7 If every subsequence of (an ) has at least one subsequence which converges to x, then (an ) also converges to x.
Proof. Proof is left as an exercise.
We have seen in Theorem 1.3 that every convergent sequence is bounded, but a
bounded sequence need not be convergent.
Question: For every bounded sequence, can we have a convergent subsequence?
The answer is in affirmative:
Theorem 1.8 (Bolzano-Weirstrass theorem). Every bounded sequence of real
numbers has a convergent subsequence.
Proof: For a first reading this proof can be omitted.
quence in R. For each k N, consider the set

Let (an ) be a bounded se-

Ek := {an : n k},
and let bk := sup Ek , k N. Clearly, b1 b2 . . ..
1

From the book: Mathematical Analysis: a straight forward approach by K.G. Binmore.

Sequence of Real Numbers

11

We consider the following two mutually exclusive cases:


(i) For every k N, bk Ek
(ii) There exists k N such that bk 6 Ek .
In case (i), we may write (bk ) is a monotonically decreasing subsequence of (an ),
and since (an ) is bounded, (bk ) converges.
Now, suppose that case (ii) holds, and let k N such that bk 6 Ek . Then for
every n k, there exists n0 > n such that an0 > an . Take n1 = k and n2 = n0 .
Then we have n1 < n2 and an1 < an2 . Again, since n2 Ek , there exists n3 > n2
such that an2 1 < an3 . Continuing this way, we obtain an increasing subsequence

(anj )
j=1 . Again, since (an ) is bounded, (anj )j=1 is bounded, so that it converges.
Remark 1.4 We may observe that the proof of Theorem 1.8 can be slightly modified so that we, in fact, have the following: Every sequence in R has a monotonic
subsequence.

Exercise 1.17 Prove the statement in italics in Remark 1.4.

1.1.4

Further examples

EXAMPLE 1.7 Let a sequence (an ) be defined as follows :


a1 = 1,

an+1 =

2an + 3
,
4

n = 1, 2, . . . .

We show that (an ) is monotonically increasing and bounded above.


Note that
an+1 =

2an + 3
an 3
=
+ an
4
2
4

3
an .
2

Thus it is enough to show that an 3/2 for all n N.


Clearly, a1 3/2. If an 3/2, then an+1 = an /2 + 3/4 < 3/4 + 3/4 = 3/2.
Thus, we have proved that an 3/2 for all n N. Hence, by Theorem 1.5, (an )
converges. Let its limit be a. Then taking limit on both sides of an+1 = 2an4+3 we
have
3
2a + 3
a=
i.e., 4a = 2a + 3 so that a = .
4
2

EXAMPLE 1.8 Let a sequence (an ) be defined as follows :


1
2
a1 = 2, an+1 =
an +
, n = 1, 2, . . . .
2
an
It is seen that, if the sequence converges, then its limit would be

2.

12

Sequence and Series of Real Numbers

M.T. Nair

Since a1 = 2, one may try to show that (an ) is monotonically decreasing and
bounded below.
Note that



1
2
an+1 :=
an +
an

a2n 2, i.e., an 2.
2
an

Clearly a1 2. Now,



2
1
an +
2

a2n 2 2an + 2 0,
an+1 :=
2
an

i.e., if and only if (an 2)2 0. This is true for every n N. Thus, (an ) is monotonically decreasing and bounded below, so that by Theorem 1.5, (an ) converges.

EXAMPLE 1.9 Consider the sequence (an ) with an = (1+1/n)n for all n N. We
show that (an ) is monotonically increasing and bounded above. Hence, by Theorem
1.5, (an ) converges.
Note that
an



1 n
=
1+
n
1 n(n 1) 1
n(n 1) . . . 2.1 1
+ +
= 1 + n. +
2
n
n  2!  n

  n!  n
1
1
1
1
2
= 1+1+
1
+
1
1
+
2!
n
3!
n
n




1
1
n1
+
1
1
n!
n
n
an+1 .

Also
1 n(n 1)
1 n(n 1) . . . 2.1
+ +
2
2!
n
n!
nn
1
1
1
1 + 1 + + + +
2! 3!
n!
1
1
1
1 + 1 + + 2 + + n1
2 2
2
< 3.

an = 1 + 1 +

Thus, (an ) is monotonically increasing bounded above.

Exercise 1.18 Show that (n1/n )


n=3 is a monotonically decreasing sequence.

In the four examples that follow, we shall be making use of Theorem 1.2 without
mentioning it explicitly.

Sequence of Real Numbers

13

EXAMPLE 1.10 Consider the sequence (bn ) with


bn = 1 +

1
1
1
1
+ + + +
1! 2! 3!
n!

n N.

Clearly, (bn ) is monotonically increasing, and we have noticed in the last example
that it is bounded above by 3. Hence, by Theorem 1.5, it converges.
Let (an ) and (bn ) be as in Examples 1.9 and 1.10 respectively, and let a and b
their limits. We show that a = b.
We have observed in last example that 2 an bn 3. Hence, taking limits, it
follows that a b. Notice that





1
1
1
1
2
an = 1 + 1 +
1
+
1
1
+
2!
n
3!
n
n




1
1
n1
... +
1
... 1
.
n!
n
n
Hence, for m, n with m n, we have





1
1
1
2
1
1
+
1
1
+
an 1 + 1 +
2!
n
3!
n
n

 

1
1
m1
+
1
... 1
.
m!
n
n
Taking limit as n , we get (cf. Theorem 1.2 (c))
x1+

1
1
1
1
+ + + +
= bm .
1! 2! 3!
m!

Now, taking limit as m , we get a b. Thus we have proved a = b.


The common limit in the above two examples is denoted by the letter e.

EXAMPLE 1.11 Let a > 0. We show that, if 0 < a < 1, then the sequence (an )
converges to 0, and if a > 1, then (an ) diverges to infinity.
(i) Suppose 0 < a < 1. Then we can write a = 1/(1 + r), r > 0, and we have
an = 1/(1 + r)n = 1/ (1 + nr + + rn ) < 1/(1 + nr) 0

as

n .

Hence, an 0 as n .
An alternate way: Let xn = an . Then (xn ) is monotonically decreasing and bounded
below by 0. Hence (xn ) converges, to say x. Then xn+1 = an+1 = axn ax. Hence,
x = ax. This shows that x = 0.
(ii) Suppose a > 1. Then, since 0 < 1/a < 1, the sequence (1/an ) converges to
0, so that (an ) diverges to infinity. (Why ?)


14

Sequence and Series of Real Numbers

M.T. Nair

EXAMPLE 1.12 The sequence (n1/n ) converges and the limit is 1.


Note that n1/n = 1 + rn for some sequence (rn ) of positive reals. Then we have
n = (1 + rn )n

n(n 1) 2
rn ,
2

so that rn2 2/(n 1) for all n 2. Since 2/(n 1) 0, by Theorem 1.2(c), that
rn 0, and hence by Theorem 1.2(c), n1/n = 1 + rn 1.

Remark 1.5 The proof of the result in Example 1.12 does not require the knowledge
that n1/n 1, but uses the facts that 2/(n 1) 0 (which can be shown easily)
and Theorem 1.2. But, if one is asked to show that n1/n 1, then we can give
another proof of the same by using only the definition, as follows:
Another proof without using Theorem 1.2. Let > 0 be given. To find n0 N such
that n1/n 1 < for all n n0 . Note that
n1/n 1 < n1/n < 1 + n < (1 + )n = 1 + n +

n(n 1) 2
+ . . . + n .
2

Hence

2
.
2
So, we may take any n0 N which satisfies n0 2(1 + /2 ).
n1/n 1 < if

i.e., if

n>

EXAMPLE 1.13 For any a > 0, (a1/n ) converges to 1.


If a > 1, then we can write a1/n = 1 + rn for some sequence (rn ) of positive reals.
Then we have
a = (1 + rn )n nrn so that rn a/n.
Since a/n 0, by Theorem 1.2(c), rn 0, and hence by Theorem 1.2(c), a1/n =
1 + rn 1.
In case 0 < a < 1, then 1/a > 1. Hence, by the first part, 1/a1/n = (1/a)1/n 1,
so that an 1.

Exercise 1.19 Give another proof for the result in Example 1.13 without using
Theorem 1.2.
J
EXAMPLE 1.14 Let (an ) be a bounded sequence of non-negative real numbers.
Then (1 + an )1/n 1 as n :
This is seen as follows: Let M > 0 be such that 0 an M for all n N. Then,
1 (1 + an )1/n (1 + M )1/n for all n N. By Example 1.13, (1 + M )1/n 0.
Hence the result follows by making use of part (d) of Theorem 1.2.

EXAMPLE 1.15 As an application of some of the results discussed above, consider
the sequence (an ) with an = (1+1/n)1/n , n N. We already know that lim an = 1.
n

Now, another proof for the same: Note that 1 an 21/n , and 21/n 1 as n .


Sequence of Real Numbers

15

EXAMPLE 1.16 Consider the sequence (an ) with an = (1 + n)1/n . Then an 1


as n . We give two proofs for this result.
(i) Observe that an = n1/n (1 + 1/n)1/n . We already know that n1/n 1, and
(1 + 1/n)1/n 1 as n .
(ii) Observe that n1/n (1 + n)1/n (2n)1/n = 21/n n1/n , where n1/n 1 and
1 as n .

an+1
EXAMPLE 1.17 Suppose (an ) is a sequence of positive terms such that lim
=
n an
` < 1. Then an 0.
an+1
= ` < 1, there exists q such that ` < q < 1 and N N such
Since lim
n an
an+1
that
q for all n N . Hence,
an
21/n

0 an q nN aN

n N.

Now, since q nN 0 as n , it follows that an 0 as n .

Exercise 1.20 Suppose Let (an ) is a sequence of positive terms such that
an+1
lim
= ` > 1. Then (an ) diverges to .
J
n an
EXAMPLE 1.18 Let 0 < a < 1. Then nan 0 as n .
To see this let an := nan for n N. Then we have
(n + 1)a
(n + 1)an+1
an+1
=
=
n
an
na
n
Hence, lim

n N.

an+1
= a < 1. Thus, the result follows from the last example.
an

Exercise 1.21 Obtain the the result in Example 1.18 by using the arguments in
Example 1.11.
J
(k)

(k)

Remark 1.6 Suppose for each k N, an 0, bn 1 as n , and also


(n)
(n)
an 0, bn 1 as n . In view of Theorems 1.2 and 1.4, one may think that
(2)
(n)
a(1)
n + an + + an 0

as

and
(2)
(n)
b(1)
n bn bn 1

as

n .

Unfortunately, that is not the case. To see this consider


a(k)
n =

k
,
n2

1/n
b(k)
n =k

k, n N.

16

Sequence and Series of Real Numbers


(k)

M.T. Nair

(k)

(n)

(n)

Then, for each k N, an 0, bn 1 as n , and also an 0, bn 1 as


n . But,
(2)
(n)
a(1)
n + an + + an =

1
2
n
n+1
1
+
+ + 2 =

n2 n2
n
2n
2

as

and
(2)
(n)
1/n 1/n
b(1)
2 n1/n = (n!)1/n 6 1
n bn bn = 1

as

n .

In fact, for any k N, if n k, then


(n!)1/n (k!)1/n (k nk )1/n = (k!)1/n k 1k/n = k

 k! 1/n
kk

 1/n
Since for any given k N, k kk!k
k as n , it follows that (n!)1/n 6 1. In
fact, there exists n0 N such that
(n!)1/n

k
2

n max{n0 , k}.


Thus, the sequence is (n!)1/n is unbounded.

EXAMPLE 1.19 Let an = (n!)1/n , n N. Then an 1 as n . We give two


proofs for this.
(i) Note that, for every n N,
2

1 (n!)1/n (nn )1/n = n1/n .


2

Now, since n1/n 1, we have (n!)1/n 1.


(ii) By GM-AM inequality, for n N,
(n!)1/n = (1.2. . . . .n)1/n
Thus,

n+1
1 + 2 + ... + n
=
n.
n
2
2

1 (n!)1/n n1/n .
2

Since n1/n 1 we have (n!)1/n 1.

1.1.5

Cauchy sequence

Theorem 1.9 If a real sequence (an ) converges, then for every > 0, there exists
N N such that
|an am | < n, m N.
Proof. Suppose an a as n , and let > 0 be given. Then we know that
there exists N N such that |an a| < /2 for all n N . Hence, we have
|an am | |an a| + |a am | < n, m N.
This completes the proof.

Sequence of Real Numbers

17

Definition 1.11 A a sequence (an ) is said to be a Cauchy sequence if for every


> 0, there exists N N such that
|an am | < n, m N.

Theorem 1.9 show that every convergent sequence is a Cauchy sequence. In
particular, if (an ) is not a Cauchy sequence, then (an ) does not converge to any
a R. Thus, Theorem 1.9 may help us to show that certain sequence is not
convergent. For example, see the follwing exercise.
Exercise 1.22 Let sn = 1 + 12 + . . . + n1 for n N. Then show that (sn ) is not a
Cauchy sequence. [Hint: For any n N, note that s2n sn 12 .]
J
Let us observe certain properties of Cauchy sequences.
Theorem 1.10 Let (an ) be a Cauchy sequence. The we have the following.
(i) (an ) is a bounded sequence.
(ii) If (an ) has a convergent subsequence with limit a, then the sequence (an )
itself will converge to a.
Proof. Let (an ) be Cauchy sequence and > 0 be given.
(i) Let N N be such that |an am | < for all n, m N . In particular,
|an | |an aN | + |aN | + |aN |

n N.

This proves (i).


(ii) Suppose (ank ) is a convergent subsequence of (an ), and let a = lim ank . Let
k

n0 N be such that |ank a| < for all k n0 . Hence, we have


|ak a| |ak ank | + |ank a| < 2

k max{N, n0 }.

Thus, an a as n .
In fact, the converse of Theorem 1.9 also holds:
Theorem 1.11 Every Cauchy sequence of real numbers converges.
Proof of Theorem 1.11. Let (an ) be a Cauchy sequence. By Theorem 1.10(i),
(an ) is bounded. Then, by Bolzano-Weierstrass theorem (Theorem 1.8), (an ) has a
convergent subsequence (ank ). Let a = lim ank . Now, by Theorem 1.10(ii), an a
k

as n .

18

Sequence and Series of Real Numbers

M.T. Nair

An alternate proof without using Bolzano-Weierstrass theorem (Theorem 1.8). Let


(xn ) be Cauchy sequence, and > 0 be given. Then there exists n N such that
|xn xm | < for all n, m n . In particular, |xn xn | < for all n n . Hence,
for any 1 , 2 > 0,
n max{n1 , n2 }.

xn1 1 < xn < xn2 + 2

()

From this, we see that the set {xn : > 0} is bounded above, and
x := sup{xn : > 0}
satisfies
xn < x < xn +

n n .

()

From () and (), we obtain


|xn x| <

n n .

This completes the proof.


Here is an example of a general nature.
EXAMPLE 1.20 Let (an ) be a sequence of real numbers. Suppose there exists a
positive real number < 1 such that
|an+1 an | |an an1 |

n N, n 2.

Then (an ) is a cauchy sequence. To see this first we observe that


|an+1 an | n1 |a2 a1 |

n N, n 2.

Hence, for n > m,


|an am | |an an1 | + . . . + |am+1 am |
(n2 + . . . + m1 )|a2 a1 |
m1 (1 + + . . . + nm3 )|a2 a1 |
m1
|a2 a1 |.

1
Since m1 0 as m , given > 0, there exists N N such that |an am | <
for all n, m N .

Exercise 1.23 Given a, b R and 0 < < 1, let (an ) be a sequence of real numbers
defined by a1 = a, a2 = b and
an+1 = (1 + )an an1

n N, n 2.

Show that (an ) is a Cauchy sequence and its limit is (b + a)/(1 ).

Sequence of Real Numbers

19

Exercise 1.24 Suppose f is a function defined on an interval J. If there exists


0 < < 1 such that
|f (x) f (y)| |x y|

x, y J,

then the for any a J, the sequence (an ) defined by


a1 = f (a),

an+1 := f (an )

n N,

is a Cauchy sequence. Show also that the limit of the sequence (an ) is independent
of the choice of a.
J

1.1.6

Additional exercises

1. Prove that xn l as n if and only if |xn l| 0 as n , and in


that case |xn | |l| 0 as n .
2. Establish convergence or divergence of the sequence (an ) each of the following
sequences, where an is:
(i)

n
n+1

(ii)

(1)n n
n+1

(iii)

2n
,
3n2 + 1

(iv)

2n2 + 3
.
3n2 + 1

3. Suppose (an ) is a real sequence such that an 0 as n . Show the


following:
(a) The sequence (a2n ) converges to 0.
(b) If an > 0 for all n, then the sequence (1/an ) diverges to infinity.
4. Let {xn } be a sequence defined recursively by xn+2 = xn+1 + xn , n 1 with
x1 = x2 = 1. Show that {xn } diverges to .

5. Let xn = n + 1 n for n N. Show that {xn }, {nxn } and { nxn } are


convergent. Find their limits.
P
1
6. Let xn = nk=1 n+k
, n N. Show that {xn } is convergent.
7. If (an ) converges to x and an 0 for all n N, then show that x 0 and

( an ) converges to x.
8. Prove the following:
(a) If {xn } is increasing and unbounded, then xn + as n .
(b) If {xn } is decreasing and unbounded, prove that xn as n .
9. Let a1 = 1, an+1 =
find its limit.

2 + an for all n N. Show that (an ) converges. Also,

10. Let a1 = 1 and an+1 = 41 (2an + 3) for all n N. Show that {an } is monotonically increasing and bounded above. Find its limit.

20

Sequence and Series of Real Numbers

11. Let a1 = 1 and an+1 =


its limit.

M.T. Nair

an
for all n N. Show that {an } converges. Find
1 + an

12. Prove that if (an ) is a Cauchy sequence having a subsequence which converges
to a, then (an ) itself converges to a.
13. Suppose (an ) is a sequence such that the subsequences (a2n1 ) and (a2n ) converge to the same limit, say a. Show that (an ) also converges to a.
14. Let {xn } be a monotonically increasing sequence such that {x3n } is bounded.
Is {xn } convergent? Justify your answer.
15. Show that, if a sequence (an ) has the property that an+1 an 0 as n ,
then (an ) need not converge.
16. Let a1 = 1 and an+1 = 41 (2an + 3) for all n N. Show that (an ) is monotonically increasing and bounded above. Find its limit.
17. Give an example in support of the statement: If a sequence {an } has the
property that an+1 an 0 as n , then {an } need not converge.
18. Let a > 0 and xn =
(xn ).

an an
,
an +an

n N. Discuss the convergence of the sequence

19. If 0 < a < b and an = (an + bn )1/n for all n N,then show that (an ) converges
n
to b. [Hint: Note that (an + bn )1/n = b(1 + ab )1/n .]
n
20. If 0 < a < b and an+1 = (an bn )1/2 and bn+1 = an +b
for all n N with a1 = a,
2
b1 = b, then show that (an ) and (bn ) converge to the same limit. [Hint: First
observe that an an+1 bn+1 bn for all n N.]
n bn
21. Let a1 = 1/2 and b = 1, and let an+1 = (an bn )1/2 and bn+1 = a2an +b
for all
n
n N. Show that (an ) and (bn ) converge to the same limit. [Hint: First
observe that bn bn+1 an+1 an for all n N.]

1
). Prove that
22. Let a1 = 1/2, b = 1, an = (an1 bn1 )1/2 and b1n = 12 ( a1n + bn1
an1 < an < bn < bn1 for all n N. Deduce that both the sequences {an }
and {bn } converge to the same limit , 1/2 < < 1.

23. Prove that if {an } is a Cauchy sequence having a subsequence which converges
to a, then {an } itself converges to a.

Series of Real Numbers

1.2

21

Series of Real Numbers

Definition 1.12 A series of real numbers is an expression of the form


a1 + a2 + a3 + . . . ,
or more compactly as

n=1 an ,

where (an ) is a sequence of real numbers.

The number an is called the n-th term of the series and the sequence sn :=
P
is called the n-th partial sum of the series
n=1 an .

n
X

ai

i=1

1.2.1

Convergence and divergence of series

P
Definition 1.13 A series
n=1 an is said to converge (to s R) if the sequence
{sn } of partial sums of the series converge (to s R).
P
P
If
n=1 an = s.
n=1 an converges to s, then we write
A series which does not converge is called a divergent series.

A necessary condition
P
Theorem 1.12 If
n=1 an converges, then an 0 as n . Converse does not
hold.
Proof. Clearly, if sn is the n-th partial sum of the convergent series
then
an = sn sn1 0 as n .

n=1 an ,

does not hold it is enough to observe that the series


PTo see that the converse
1
n=1 an with an = n for all n N diverges whereas an 0.
The proof of the following corollary is immediate from the above theorem.
Corollary 1.13 Suppose (an ) P
is a sequence of positive terms such that an+1 > an
for all n N. Then the series
n=1 an diverges.
The above theorem and corollary shows, for example, that the series
diverges.

n
n=1 n+1

P
1
EXAMPLE 1.21 We have
that the sequence (sn ) with sn = nk=1 k!
conP seen
1
verges. Thus, the
we have seen that the sequence
P series n=1 n! converges.
P Also,
1
(n ) with n = nk=1 k1 diverges. Hence,
diverges.

n=1 n

22

Sequence and Series of Real Numbers

M.T. Nair

P
n1 , where a, q R.
EXAMPLE 1.22 Consider the geometric series series
n=1 aq
n1
Note that sn = a + aq + . . . + aq
for n N. Clearly, if a = 0, then sn = 0 for all
n N. Hence, assume that a 6= 0. Then we have
(
na
if q = 1,
sn =
a(1q n )
if q 6= 1.
1q
Thus, if q = 1, then (sn ) is not bounded; hence not convergent. If q = 1, then we
have

a
if n odd,
sn =
0
if n even.
Thus, (sn ) diverges for q = 1 as well. Now, assume that |q| =
6 1. In this case, we
have




sn a = |a| |q|n .

1 q |1 q|
This shows that, if |q| < 1, then (sn ) converges to
not bounded, hence diverges.

a
1q ,

and if |q| > 1, then (sn ) is




Theorem 1.14 Suppose (anP


) and (bn ) are sequences such that
Pfor some k N,
a
converges
if
and
only
if
an = bn for all n k. Then
n=1 bn converges.
n=1 n
P
Proof. Suppose sn and n be the n-th partial sums of the series
n=1 an and
Pk
Pk
P
i=1 bi . Then we have
i=1 ai and =
n=1 bn respectively. Let =
sn =

n
X
i=k+1

ai =

n
X

bi = n

n k.

i=k+1

From this it follows that the sequence (sn ) converges if and only if (n ) converges.
P
P
From the above theorem it follows if
n=1 bn is obtained from
n=1 an by
omitting or adding a finite number of terms, then

an

converges

n=1

In particular,

X
n=1

an

bn

converges.

n=1

converges

an+k

converges

n=1

for any k N.
The proof of the following theorem is left as an exercise.
P
P
Theorem 1.15 Suppose
n=1 an converges to s and
n=1 bn converges to . Then
P
for every , R, n=1 ( an + bn ) converges to s + .

Series of Real Numbers

1.2.2

23

Some tests for convergence

Theorem 1.16 (Comparison test) Suppose (an ) and (bn ) are sequences of nonnegative terms, and an bn for all n N. Then,
(i)
(ii)

n=1 bn

converges =

n=1 an

diverges =

converges,

diverges.

n=1 an
n=1 bn

P
Proof.
Suppose
s
and

be
the
n-th
partial
sums
of
the
series
n
n
n=1 an and
P
n=1 bn respectively. By the assumption, we get 0 sn n for all n N, and
both (sn ) and (n ) are monotonically increasing.
(i) Since (n ) converges, it is bounded. Let M > 0 be such that n M for all
n N. Then we have sn M for all n N. Since (sn ) are monotonically increasing,
it follows that (sn ) converges.
(ii) Proof of this part follows from (i) (How?).
Corollary 1.17 Suppose (an ) and (bn ) are sequences of positive terms.
(a) Suppose ` := limn abnn exists. Then we have the following:
P
P
(i) If ` > 0, then
n=1 an converges.
n=1 bn converges
P
P
(ii) If ` = 0, then n=1 bn converges n=1 an converges.
P
P
an
(b) Suppose lim
= . Then
n=1 bn converges.
n=1 an converges
n bn
Proof. (a) Suppose limn

an
bn

= `.

(i) Let ` > 0. Then for any > 0 there exists n N such that ` < abnn < ` +
for all n N . Equivalently, (` )bn < an < (` + )bn for all n N . Had we taken
= `/2, we would get 2` bn < an < 3`
2 bn for all n N . Hence, the result follows by
comparison test.
(ii) Suppose ` = 0. Then for > 0, there exists n N such that < abnn <
for all n N . In particular, an < bn for all n N . Hence, we get the result by
using comparison test.
(b) By assumption, there exists N N such that
result follows by comparison test.

an
bn

1 for all n N . Hence the

P
1
EXAMPLE 1.23 We have already seen that the sequence (sn ) with sn = nk=1 k!
1
1
converges. Here
proof for the same fact: Note that n!
2n1
P is another
Pfor all
1
1
n N. Since n=1 2n1 converges, it follows from the above theorem that
n=1 n!
converges.


24

Sequence and Series of Real Numbers

M.T. Nair

P
1
for all n N, and since the series
n=1 n diverges,
P 1

it follows from the above theorem that the series n=1 n also diverges.
EXAMPLE 1.24 Since

1
n

1
n

Theorem 1.18 (deAlemberts ratio test) Suppose (an ) is a sequence of positive


terms such that limn an+1
an = ` exists. Then we have the following:
P
(i) If ` < 1, then the series
n=1 an converges.
P
(ii) If ` > 1, then the series n=1 an diverges.
Proof. (i) Suppose ` < q < 1. Then there exists N N such that
an+1
<q
an

n N.

In particular,
an+1 < q an < q 2 an1 < . . . < q n a1 , n N.
P
P
n
Since
n=1 an also converges.
n=1 q converges, by comparison test,
(ii) Let 1 < p < `. Then there exists N N such that
an+1
> p > 1 n N.
an
From this it follows that (an ) does not converge to 0. Hence

n=1 an

diverges.

Theorem 1.19 (Cauchys root test) Suppose (an ) is a sequence of positive terms
such that limn an 1/n = ` exists. Then we have the following:
P
(i) If ` < 1, then the series
n=1 an converges.
P
(ii) If ` > 1, then the series
n=1 an diverges.
Proof. (i) Suppose ` < q < 1. Then there exists N N such that
an 1/n < q

n N.
P n
n
Hence,
P an < q for all n N . Since the n=1 q converges, by comparison test,
n=1 an also converges.
(ii) Let 1 < p < `. Then there exists N N such that
an 1/n > p > 1

n N.

Hence, an 1 for all n N . Thus, (an ) does not converge to 0. Hence,


also diverges.

n=1 an

Remark 1.7 We remark that both dAlemberts test and Cauchy test are silent
for the case ` = 1. But, for such case, we may be able to infer the convergence or
divergence by some other means.


Series of Real Numbers


EXAMPLE 1.25 For every x R, the series
Here, an =

xn

n! .

xn
n=1 n!

25

converges:

Hence
x
an+1
=
an
n+1

n N.

Hence, it follows that limn an+1


an = 0, so that by dAlemberts test, the series
converges.



n
P
n
EXAMPLE 1.26 The series
converges: Here
n=1 2n+1
an 1/n =

n
1
< 1.
2n + 1
2

Hence, by Cauchys test, the series converges.


P
EXAMPLE 1.27 Consider the series
n=1
limn an+1
an

= 1 = limn an
sn =

n
X
k=1

1/n .


1
n(n+1) .

In this series, we see that

However, the n-th partial sum sn is given by


n

X
1
=
k(k + 1)
k=1

1
1

k k+1


=1

1
.
n+1

Hence {sn } converges to 1.

EXAMPLE 1.28 Consider the series

1
n=1 n2 .

1
1

2
(n + 1)
n(n + 1)
Since

In this case, we see that


n N.

1
n=1 n(n+1)

converges, by comparison test, the given series also converges. 


P 1
P 1
EXAMPLE 1.29 Since P
n=1 n2 converges and
n=1 n diverges, by comparison

1
test, we see that the series n=1 np converges for p 2 and diverges for p 1. 
P
1
EXAMPLE 1.30 Consider the series
n=1 np for p > 1. To discuss this example,
p
consider the function f (x) := 1/x , x 1. Then we see that for each k N,
Z k
1
1
dx
1
k 1 x k = p p = p
.
p
k
x
k
k1 x
Hence,

Z n
n
n Z k
X
X
1
dx
dx
n1p 1
1

=
=

.
p
p
kp
x
x
1

p
p

1
k1
1
k=2

k=2

Thus,
sn :=

n
X
1
1

+ 1.
p
k
p1
k=1

Hence, (sn ) is monotonically increasing and bounded above. Therefore, (sn ) converges.


26

Sequence and Series of Real Numbers

M.T. Nair

Now, we have a more general result.


Theorem 1.20 (Integral test) Suppose f (x) is a continuous,
R n non-negative and
decreasing function for x [1, ). For each n N, let an := 1 f (x)dx. Then

f (n)

converges (an )

converges.

n=1

Rn
Rk
P
Proof. First we observe that an = 1 f (x) dx = nk=2 k1 f (x) dx. Now, since
f (x) is a decreasing function for x [1, ), we have for each k N,
k 1 x k = f (k) f (x) f (k 1).
Hence, for k = 2, 3, . . .,
k

Z
f (k)

f (x) dx f (k 1)
k1

so that

n
X

f (k)

k=2

Thus,

n
X

Now, let sn :=

f (x) dx

k1

k=2

Z
f (k)

k=2
n
X

n Z
X

f (x) dx
1

n
X

f (k 1).

k=2
n
X

f (k 1).

k=2

f (k) for n N. Then from the above inequalities, together

k=1

with the fact that (sn ) is a monotonically increasing sequence, it follows that (an )
converges if and only if (sn ) converges.

1.2.3

Alternating series

P
n+1 u where (u ) is a sequence of
Definition 1.14 A series of the form
n
n
n=1 (1)
positive terms is called an alternating series.

Theorem 1.21 (Leibnizs theorem) Suppose (un ) is a sequence of positive terms
such that
n+1 for all n N, and un 0 as n . Then the alternating
P un un+1
series
(1)
un converges.
n=1
Proof. Let sn be the n-th partial sum of the alternating series
We observe that
s2n+1 = s2n + u2n+1 n N.

n+1 u .
n
n=1 (1)

Since un 0 as n , it is enough to show that (s2n ) converges (Why?). Note


that
s2n = (u1 u2 ) + (u3 u4 ) + . . . + (u2n1 u2n ),

Series of Real Numbers

27

s2n = u1 (u2 u3 ) . . . (u2n2 u2n1 ) u2n


for all n N. Since ui ui+1 0 for each i N, (s2n ) is monotonically increasing
and bounded above. Therefore (s2n ) converges. In fact, if s2n s, then we have
s2n+1 = s2n + u2n+1 s, and hence sn s as n .
P
(1)n+1
By the above theorem the series
converges. Likewise, the series
n=1
n
P (1)n+1
P (1)n+1
also converge.
n=1
n=1 2n1 and
2n
P
(1)n+1
Remark 1.8 The series
n=1 2n1 appear in the work of a Kerala mathematician Madhava presented in the year around 1550 by another Kerala mathematician
Nilakantha. The discovery of the above series is normally attributed to Leibniz and
James Gregory after nearly 300 years of its discovery.

Suppose (un ) is as in Leibnizs theorem (Theorem 1.21), and let s R be such

X
that sn s, where sn is the nth partial sum of
(1)n+1 un .
n=1

How fast (sn ) converges to s?


In the proof of Theorem 1.21, we have shown that {s2n } is a monotonically
increasing sequence. Similarly, it can be shown that {s2n1 } is a monotonically
decreasing sequence.
Since (s2n1 ) is monotonically decreasing and (s2n ) is monotonically increasing,
we have
s2n1 = s2n + u2n s + u2n ,

s s2n+1 = s2n + u2n+1 .

Thus,
s2n1 s u2n ,

s s2n u2n+1 .

Consequently,
|s sn | un+1

1.2.4

n N.

Absolute convergence

Definition 1.15 A series


verges.

n=1 an

is said to converge absolutely, if

n=1 |an |

con

Theorem 1.22 Every absolutely convergent series converges.


P
Proof. Suppose
convergent
series. Let sn and n be
n=1 an is an absolutely
P
P
the n-th partial sums of the series
a
and
|a
| respectively. Then, for
n
n
n=1
n=1
n > m, we have


X

n
n
X



|sn sm | =
an
|an | = |n m |.
j=m+1 j=m+1

28

Sequence and Series of Real Numbers

M.T. Nair

Since, {n } converges, it is a Cauchy sequence. Hence, form the above relation it


follows that {sn } is also a Cauchy sequence. Therefore, by the Cauchy criterion, it
converges.
P
an absolutely
Another proof without using Cauchy criterion. Suppose
n=1 an isP
convergent series. Let sn and n be the n-th partial sums of the series
n=1 an and
P

|a
|
respectively.
Then
it
follows
that
n
n=1
sn + n = 2pn ,
where pn is the sum of all positive terms from {a1 , . . . , an }. Since {n } converges,
it is bounded, and since pn n for all n N, the sequence {pn } is also bounded.
Moreover, {pn } is monotonically increasing. Hence {pn } converge as well. Thus,
both {n }, {pn } converge. Now, since sn = 2pn n for all n N, the sequence
{sn } also converges.
P
P
a
is
said
to
converge
conditionally
if
Definition 1.16
A
series
n
n=1 an
n=1
P
converges, but
|a
|diverges.

n
n=1
P
(1)n+1
EXAMPLE 1.31 The series
is conditionally convergent.

n=1
n
P
(1)n+1
EXAMPLE 1.32 The series
is absolutely convergent.

n=1
n2
P
(1)n+1
EXAMPLE 1.33 The series
is absolutely convergent.

n=1
n!
P
sin(n)
is absolutely convergent:
EXAMPLE 1.34 For any R, the series
n=1
n2
Note that


sin(n)
1


n2 n2 n N.
P
P sin(n)
1
Since
converges,
by
comparison
test,

2
n=1 n
n=1 n2 also converges.
EXAMPLE 1.35 The series

X
(1)n log n
is conditionally convergent. To see
n log(log n)

n=3

log n
. Since n log n log(log n) we have
this, let un =
n log(log n)
1
log n
1

n
n log(log n)
log(log n)

()

so that un 0. It can be easily seen that un+1 un . Hence, by Leibnitz


P theorem,
the given series converges. Inequality () also shows that the series
n=3 un does
not converge.

Here are two more results whose proofs are based on some advanced topics in
analysis

Series of Real Numbers

29

P
Theorem 1.23 Suppose
n=1 an is an absolutely convergent
P series and (bn ) is a
sequence obtainedPby rearranging
the
terms
of
(a
).
Then
n
n=1 bn is also absolutely
P
convergent, and
a
=
b
.
n=1 n
n=1 n
P
Theorem 1.24 Suppose
n=1 an is a conditionally convergent series. The for
every R, there exists a P
sequence (bn ) whose terms are obtained by rearranging
the terms of (an ) such that
n=1 bn = .
To illustrate the last theorem consider the conditionally convergent series
Consider the following rearrangement of this series:
1
Thus, if an =

n=1

(1)n+1
.
n

1 1 1 1 1
1
1
1
+ + +

+ .
2 4 3 6 8
2k 1 4k 2 4k

(1)n+1
n

for all n N, the rearranged series is

b3k2 =

1
,
2k 1

b3k1 =

1
,
4k 2

n=1 bn ,

b3k =

where

1
4k

for k = 1, 2, . . .. Let sn and n be the n-th partial sums of the series


P

n=1 bn respectively. Then we see that

n=1 an

and

1 1 1 1 1
1
1
1
3k = 1 + + +

 2 4 3 6 8
 2k 1  4k 2 4k

1 1
1 1 1
1
1
1
=
1
+

+ +

2 4
3 6 8
2k 1 4k 2 4k

 



1 1
1 1
1
1
=

+ +

2 4
6 8
4k 2 4k

 



1
1
1 1
1
1
=
1
+

+ +

2
2
3 4
2k 1 2k
1
=
s2k .
2
Also, we have
3k+1 = 3k +

1
,
2k + 1

3k+2 = 3k +

1
1

.
2k + 1 4k + 2

We know that {sn } converge. Let limn sn = s. Since, an 0 as n , it then


follows that
s
lim 3k = ,
k
2

s
lim 3k+1 = ,
k
2

Hence, we can infer that n s/2 as n .

s
lim 3k+2 = .
k
2

30

Sequence and Series of Real Numbers

1.2.5

Additional exercises

M.T. Nair

(These problems were prepared for the students of MA1010, Aug-Nov, 2010.)

1. Using partial fractions, prove that

X
n=1

3n 2
=1
n(n + 1)(n + 2)

2. Test the following series for convergence:

X
(n!)2
(a)
(2n)!
n=1

(d)

(g)


X
n=1

X
n=1

n
n+1

n2

X
(n!)2 n
(b)
5
(2n)!
n=1

n2

4n

1
+ 2n + 3

(e)

X
n=1

(h)

n2

X
n
(c)
n+1
n=1


1
n+1 n
n

(f )

X
(1)(n1)

n
n=1

1
2
3
n
+ 2 + 3 + ... + n + ...
2 2
2
2

1
1
1
1
(i) + + + ... +
+ ...
10
20
30
10n
1
1
1
1
(k)
+
+
+ ... +
+ ...
3
3
3
3
n+6
7
8
9
 4  9

n2
1
2
3
n
(l) +
+
+...+
+...
2
3
4
n+1

(j) 2 +

(m)

3 4
n+1
+ + ... +
+ ...
2 3
n

1 2 3
n
+ + +...+ 2
+...
2 5 10
n +1

3. Is the Leibniz Theorem applicable to the series:


1
1
1
1
1
1

+ ... +

+ ...
n1
n+1
21
2+1
31
3+1
Does the above series converge?
Justify your answer.
4. Find out whether (or not) the following series converge absolutely or conditionally:
1
1
1
1
1
+ 2 2 + 2 + ... + (1)n+1
+ ...
2
3
5
7
9
(2n 1)2
1
1
1
1
1
(b)

+ ... + (1)n
+ ...
ln 2 ln 3 ln 4 ln 5
ln n
(a) 1

5. Find the sum of the series:

1
1
1
+
+ ... +
+ ...
1.2.3 2.3.4
n.(n + 1).(n + 2)

6. Test for the convergence of the following series:

p

p
X
X
X
1
1

(a)
(b)
(c)
n4 + 1 n4 1
n!
(n + 1) n
n=1
n=1
n=1

Series of Real Numbers

(d)

(f )

1
,
(a + n)p (b + n)p

n=1
p
X
3

n3

+1n

a, b, p, q > 0

(g)

n=1

(i)

n=1

(n + 6)1/3

(j)

n=1



X
nx n
(l)
1+n
(o)

n=1

(1)n+1

(m)

2n!
n!

(log n)n

n=1

tan

n=1

 
1
n

1
n(n + 1)
n=1
 2

X
1 n
(k)
1+
n
(h)

n=1

(1)n1

n=1

n+1

(e)

31


n

(p)

n
n+1

(1)n1
p
n(n + 1)(n + 2)
n=1

X
(2)n
n=1

n=1

(n)

n2

7. Examine the following series for absolute / conditional convergence:

X
X
X
(1)n+1
(1)n+1 n
(1)n
(a)
(b)
(c)
(2n 1)2
3n1
n(log n)2
n=1

(d)

n=1

X
(1)n log n
n=1

n log log n

(e)

n=1

(1)n+1

n=1

1
n

(f )

(1)n+1

n=1

1
n2n

8. Let (an ) be a sequence


Pof non-negative numbers
Pand (akn ) be a subsequence
(an ). Show that, if n=1 an converges, then n=1 akn also converges. Is the
converse true? Why?

2
Limit, Continuity and Differentiability
of Functions

In this chapter we shall study limit and continuity of real valued functions defined
on certain sets.

2.1

Limit of a Function

Suppose f is a real valued function defined on a subset D of R. We are going to


define limit of f (x) as x D approaches a point a, not necessarily in D. First we
have to be clear about what x D approaches a point a means.

2.1.1

Limit point of a set D R

Definition 2.1 Let D R and a R. Then a is said to be a limit point of D if


there exists a sequence (an ) in D which is not eventually constant such that an a
as n .

We may recall that a sequence (an ) is said to be eventually constant if there
exists k N such that an = ak for all n k.
Theorem 2.1 A point a R is a limit point of D R if and only if for every
> 0,
(a , a + ) (D \ {a}) 6= .
Proof. Suppose a R is a limit point of D. Then we know that there exists a
sequence (an ) in D which is not eventually constant such that an a. Hence, for
every > 0, there exists N N such that an (a , a + ) for all n N . In
particular, there exists n N such that an (a , a + ) (D \ {a}).
Conversely, suppose for every > 0, (a , a + ) (D \ {a}) 6= . Then, for each
n N, taking = 1/n, there exists an D \ {a} such that an (a 1/n, a + 1/n).
Hence, 0 |an a| < 1/n for all n N, showing that an a. Clearly, (an ) is not
eventually constant.

32

Limit of a Function

33

EXAMPLE 2.1 (i) Every point in an interval is its limit point.


(ii) If I is an open interval of finite length, then both the end points of I are
limit points of I.
(iii) If D = {x R : 0 < |x| < 1}, then every point in the interval [1, 1] is a
limit point of D.
Remark 2.1 (i) For a R, an open interval of the form (a , a + ) for some
> 0 is called a neighbourhood of a, or sometimes, a -neighbourhood of a.
(ii) By a deleted neighbourhood of a point a R we mean a set of the form
D := {x R : 0 < |x a| < } for some > 0, i.e., the set (a , a + ) \ {a}. 
With the terminologies in the above remark,
a point a R is a limit point of D R if and only if every deleted neighbourhood of a contains at least one point of D.
Now, we define limit of f (x) as x approaches a.

2.1.2

Limit of a function f (x) as x approaches a

Definition 2.2 Let f be a real valued function defined on a set D R, and


let a R be a limit point of D. We say that f (x) has the limit b R as x
approaches a if for every > 0, there exists > 0 such that
|f (x) b| < whenever

x D, 0 < |x a| < ,

and in that case we write


lim f (x) = b or

xa

f (x) b as

x a.


Thus, limxa f (x) = b if and only if for every open interval Ib containing b there
exists an open interval Ia containing a such that
x Ia (D \ {a}) = f (x) Ib .
In the following, whenever we talk about limit of a function f as x
approaches a R, we assume that f is defined on a set D R and a is
a limit point of D.
Exercise 2.1 Show that, if limit of a function, if exists, is unique.

Exercise 2.2 Show that, if lim f (x) = b, then there exists a deleted neighbourhood
xa

D of a and M > 0 such that |f (x)| M for all x D .

34

Limit, Continuity and Differentiability of Functions

M.T. Nair

Remark 2.2 Suppose f is a real valued function defined on an interval I and a I.


What do we mean by the statement that
lim f (x) does not exist?
xa

If you see the definitiion of existence of a limit, then you see that, the above statement means the following:
For any b R, there exists > 0 such that for any > 0, there is atleast
one x (a , a + ) such that f (x ) 6 (b , b + ).
How do we show, by first principle, i.e., from the definition, that a function f does
not have a limit as x approaches to a particular point x0 ?
Assume that the limxx0 f (x) exists and it is equal to for some R.
Then we know that for every > 0, there exists := such that
0 < |x x0 | < = |f (x) | < .
Then, for a particular choice of , say = 0 , one must be able to find
an x0 such that |x0 x0 | < 0 but |f (x) | 0 .
Let us illustrate this by a simple example.


0, 1 x 0,
We show that
1, 0 < x 1.
lim x 0f (x) does not exist. Suppose limx0 f (x) = for some R and let
> 0 be given. Then, we know from the definition of the limit that there exists
:= such that
Let f : [1, 1] R be defined by f (x) =

x 6= 0,

x (, ) = f (x) ( , + ).

We know that, f (x) takes the values only 0 and 1 and the length of the interval
( , + ) is 2. Hence, if we take 0 such that 20 < 1, then f (x) cannot belong
to ( 0 , +0 ) for all x in a neighbourhood of 0. To see this consider the following
cases:
case (i): = 0. In this case, f (x) 6 ( 0 , + 0 ) for any x > 0.
case (ii): = 1. In this case, f (x) 6 ( 0 , + 0 ) for any x < 0.
case (iii): 6= 0, 6= 1. In this case, f (x) 6 ( 0 , + 0 ) for any x0.
Thus, we arrive at a a contradiction to our assumption.

Limit of a Function

35

Limit of a function in terms of a sequences


Let a be a limit point of D R and f : D R.
Suppose lim f (x) = b. Since a is a limit point of D, we know that there exists
xa

a sequence (xn ) in D which is not eventually constant such that xn a. Does


f (xn ) b? The answer is yes. In fact, we have more!
Theorem 2.2 If lim f (x) = b, then for every sequence (xn ) in D such that xn a,
xa

we have f (xn ) b.
Proof. Suppose lim f (x) = b. Let (xn ) be a sequence in D such that xn a. Let
xa

> 0 be given. We have to show that there exists n0 N such that |f (xn ) b| <
for all n n0 .
Since lim f (x) = b, we know that there exists > 0 such that
xa

x D \ {a}, |x a| < = |f (x) b| < .

()

Also, since xn a, there exists n0 N such that |xn a| < for all n n0 . Hence,
from (), we have |f (xn ) b| < for all n n0 .
What about the converse of the above theorem? The converse is also true, in a
slight restricted sense.
Theorem 2.3 If for every sequence (xn ) in D which is not eventually constant, the
sequence (f (xn )) converges to b, then lim f (x) = b.
xa

Proof. Suppose for every sequence (xn ) in D which is not eventually constant,
the sequence (f (xn )) converges to b. Assume for a moment that f does not have the
limit b as x approaches a. Then, by the definition of the limit, there exists 0 > 0
such that for every > 0, there exists at least one x D such that
x 6= a

and |x a| < ,

but |f (x ) b| > 0 .

In particular, for every n N, there exists xn D \ {a} such that


xn 6= a

and |xn a| <

1
,
n

but |f (xn ) b| > 0 .

Thus, (xn ) is not eventually constant such that xn a but f (xn ) 6 b. Thus we
arrive at a contradiction to our hypothesis.
Remark 2.3 The advantage of Theorem 2.2 is that, if we can find a sequence (xn )
in D such that xn a, but (f (xn )) does not converge, then we can assert that
lim f (x) does not exist. Similarly, by Theorem 2.3, we can determine lim f (x) if
n

we are able to show convergence of (f (xn )) to some b for any arbitrary (not for a
specific) non-eventually constant sequence (xn ) in D which converges to a.


36

Limit, Continuity and Differentiability of Functions

2.1.3

Some properties

M.T. Nair

The following two theorems can be proved using Theorems 2.2 and 2.3, and the
results on convergence of sequences of real numbers.
Theorem 2.4 We have the following.
(i) If lim f (x) = b and lim g(x) = c, then
xa

xa

lim [f (x) + g(x)] = b + c,

xa

(ii) If lim f (x) = b and b 6= 0, then f (x)


xa
and
1
lim
=
xa f (x)

lim f (x)g(x) = bc.

xa

6= 0 in a deleted neighbourhood of a
1
.
b

Theorem 2.5 If f and g have the same limit b as x approaches a, and if h is a


function such that f (x) h(x) g(x) for all x deleted neighbourhood of a, then
lim h(x) = b.
xa

The following two corollaries are immediate from Theorem 2.4.


Corollary 2.6 If lim f (x) = b, lim g(x) = c, and c 6= 0, then g is nonzero in a
xa
xa
deleted neighbourhood of c and
b
f (x)
= .
xa g(x)
c
lim

Corollary 2.7 If lim f (x) = b, lim g(x) = c and f (x) g(x) for all x in a deleted
xa
xa
neighbourhood of a, then b c.
Theorem 2.8 Suppose lim f (x) = b and lim g(y) = c. Then lim g(f (x)) = c.
xa

yb

xa

Proof. Let > 0 be given. Then there exists 1 > 0 such that
|y b| < 1 = |g(y) c| < .
Also, let 2 > 0 be such that
|x a| < 2 = |f (x) b| < 1 .
Hence,
|x a| < 2 = |f (x) b| < 1 = |g(f (x)) c| < .
This completes the proof.
Exercise 2.3 Write details of the proof of Theorem 2.4 and Corollary 2.6 and
Corollary 2.7.
J

Limit of a Function

37

Exercise 2.4 Suppose is a function defined in a neighbourhood of a point x0


such that lim (x) = x0 . If f is also a function defined in a neighbourhood of x0
xx0

and lim f (x) exists, then prove that lim f ((x)) exists and
xx0

xx0

lim f ((x)) = lim f (x).


xx0

xx0

2.1.4

Some Examples

EXAMPLE 2.2 If f (x) is a polynomial, say f (x) = a0 + a1 x + . . . + ak xk , then


for any a R,
lim f (x) = f (a).
xa

We obtain this by using Theorem 2.4.


Let us show the same by using the definition, i.e., using arguments: Let
b = f (a) and let > 0 be given. We have to find > 0 such that |x a| < =
|f (x) b| < . Note that
f (x) f (a) = a1 (x a) + a2 (x2 a2 ) + . . . + ak (xk ak ),
where
xn an = (x a)[xn1 + xn2 a + . . . + xan2 + an1 ].
Now, suppose |x a| < 1. Then we have |x| < 1 + |a| so that
|xnj aj1 | < (1 + |a|)n1
and hence,
|xn an | < |x a|n(1 + |a|)n1 .
Thus, |x a| < 1 implies


|f (x) f (a)| |x a| |a1 | + |a2 |2(1 + |a|) + . . . + |ak |k(1 + |a|)k1 ,
Therefore, taking := |a1 | + |a2 |2(1 + |a|) + . . . + |ak |k(1 + |a|)k1 , we have
|f (x) f (a)| < whenever

|x a| < := min{1, /}.




EXAMPLE 2.3 Let D = R \ {2} and f (x) =

x2 4
x2 .

Then lim f (x) = 4.


x2

Note that, for x 6= 2,


f (x) =

(x + 2)(x 2)
= (x + 2).
x2

Hence, for > 0, |f (x) 4| < whenever |x 2| < := .

38

Limit, Continuity and Differentiability of Functions

M.T. Nair

EXAMPLE 2.4 Let D = R \ {0} and f (x) = x1 . Then lim f (x) does not exist. To
x0

see this consider the sequence (xn ) with xn = 1/n for n N. Then we have xn 0
but (f (xn ) diverges to infinity. Therefore, by Theorem 2.2, lim f (x) does not exist.
x0

Alternatively, for any b R,


|f (x) b| |f (x)| |b| > 1

whenever

|f (x)| > 1 + |b|.

But,
|f (x)| > 1 + |b| |x| <

1
.
1 + |b|

Thus, for any b R,


|f (x) b| > 1

whenever

|x| <

1
.
1 + |b|

Thus, we have proved that it is not possible to find a > 0 such that |f (x) b| < 1
for all x with |x| < .

EXAMPLE 2.5 We show that (i) lim sin(x) = 0 and (ii) lim cos(x) = 1.
x0

x0

From the graph f the function sin x, it is clear that


0<x<

= 0 < sin x < x.


2

Also,

< x < 0 = 0 < | sin x| < |x|.


2
Hence, from Theorem 2.5, we have lim | sin x| = 0. Thus, lim sin(x) = 0.

x0

x0

Also, since cos x = 1 2 sin2 (x/2) and lim sin(x/2) = 0, Theorem 2.4(i) implies
x0

lim cos x = 1.

x0

sin x
= 1.
x0 x
It can be seen, using the graph of sin x that

EXAMPLE 2.6 We show that lim

0<x<

= sin x < x < tan x.


2

0<x<

sin x
= cos x <
< 1.
2
x

Hence,

Since

sin(x)
x

sin x
x

and cos(x) = cos x, it follows that


0 < |x| <

sin x

= cos x <
< 1.
2
x
sin x
= 1.
x0 x

Therefore, by Theorem 2.4(iv) and Example 2.5(ii), we have lim

Limit of a Function

39

Now a general example.


Exercise 2.5 Let f : R R be such that f (x + y) = f (x) + f (y). Suppose lim f (x)
x0

exists. Prove that lim f (x) = 0 and lim f (x) = f (c) for every c R.
xc

x0

Hint: Use the facts that f (2x) = 2f (x), Theorem 2.8 and f (x) f (c) = f (x c). J
Exercise 2.6 Suppose is a function defined in a neighbourhood I0 of a point x0
such that
x I0 , |x x0 | < r = |(x) x0 | < r
r > 0.
If f is also a function defined in a neighbourhood of x0 and lim f (x) exists, then
xx0

prove that lim f ((x)) exists and limxx0 f ((x)) = limxx0 f (x).

xx0

2.1.5

Left limit and right limit

Definition 2.3 Let f be a real valued function defined on a set D R, and let
a R be a limit point of D.
(i) We say that f (x) has the left limit b R as x approaches a from left
if for every > 0, there exists > 0 such that
|f (x) b| < whenever

x D, a < x < a.

(ii) We say that f (x) has the right limit b R as x approaches a from
right if for every > 0, there exists > 0 such that
|f (x) b| < whenever

x D, a < x < a + .


If f (x) has the left limit b R as x approaches a from left we write


lim f (x) = b or

xa

f (x) b as

x a ,

and if f (x) has the right limit b R as x approaches a from right we write
lim f (x) = b or

xa+

f (x) b as

x a+ .

The following theorem can be proved easily.


Theorem 2.9 Let f be a real valued function defined on a set D R, and let a R
be a limit point of D. Then lim f (x) exists if and only if lim f (x) and lim f (x)
xa

xa

exist and lim f (x) = lim f (x), and in that case


xa

xa+

lim f (x) = lim f (x) = lim f (x).

xa

xa

xa+

xa+

40

Limit, Continuity and Differentiability of Functions

M.T. Nair

In view of the above theorem, if (i) lim f (x) does not exist or (ii) lim f (x)
xa

xa+

does not exist or (iii) both lim f (x) and lim f (x) exist but lim f (x) 6= lim f (x),
xa

xa

xa+

xa+

then lim f (x) does not exist.


xa

Exercise 2.7 Give examples to illustrate the above statements.

2.1.6

Limit at and at

Definition 2.4 Suppose a function f is defined on an interval of the form (a, ) for
some a > 0. Then we say that f has the limit b as x and write lim f (x) = b,
x
if for every > 0, there exits M > a such that
|f (x) b| < whenever

x > M.


Definition 2.5 Suppose a function f is defined on an interval of the form (, a)


for some a < 0. Then we say that f has the limit b as x and write
lim f (x) = b, if for every > 0, there exits M < a such that

|f (x) b| < whenever

x < M.


Exercise 2.8 Prove the following:


1. If f is defined in (a, ) with a > 0, then the function g(x) := f (1/x) is defined
on a (0, 1/a), and
lim f (x) exists lim g(x) exists

x0

and in that case lim f (x) = lim g(x).


x

x0

2. If f is defined in (, a) with a < 0, then the function g(x) := f (1/x) is


defined on a (1/a, 0), and
lim f (x) exists lim g(x) exists

x0

and in that case lim f (x) = lim g(x).


x

x0

J
Definition 2.6
that

1. lim f (x) = + if for every M > 0, there exists > 0 such


xa

0 < |x a| < = f (x) > M.

Limit of a Function

41

2. lim f (x) = if for every M > 0, there exists > 0 such that
xa

0 < |x a| < = f (x) < M.


3.

lim f (x) = + if for every M > 0, there exists > 0 such that

x+

x > = f (x) > M.


4.

lim f (x) = if for every M > 0, there exists > 0 such that

x+

x > = f (x) < M.


5.

lim f (x) = + if for every M > 0, there exists > 0 such that

x < = f (x) > M.


6.

lim f (x) = if for every M > 0, there exists > 0 such that

x < = f (x) < M.



For the next example, we require the following definition.
Definition 2.7 For positive numbers a and b, and p, q N, we define
ap/q := (ap )1/q .
If b is a positive real number and (bn ) is s sequence of positive rational numbers
such that bn b, then we define
ab := lim abn .
n

(Of course, one has to show that this limit exists; which is true!).


1 n
exists, and we denoted it by e. Now
EXAMPLE 2.7 Recall that lim 1 +
n
n
we show that

1 x
lim 1 +
= e.
x
x
Let > 0 be given. We have to find an M > 0 N such that

1 x
e< 1+
< e + whenever x > M.
()
x
Now, we can see that, for every n N, if x R is such that n x n + 1, then
1+

1
1
1
1+ 1+
n+1
x
n

42

Limit, Continuity and Differentiability of Functions

so that


1+

Thus is is same as

M.T. Nair

1 n 
1 x 
1 n+1
1+
1+
.
n+1
x
n

1 x
n 1 +
n ,
x

where

n := 1 +

1 n+1
1 1 
1+
,
n+1
n+1


1
1 n 
n := 1 +
1+
.
n
n

We know that n e and n e as n . Therefore, there exists n0 N such


that for all n n0 ,
e < n < e + ,

e < n < e + .

Now, take M = n0 and let x > M . Take n n0 such that n + 1 x n. For this
n and x, we have

1 x
e < n 1 +
n < e + .
x
Thus, we obtained an M > 0 such that

1 x
e< 1+
< e + whenever x > M.
x
Thus, we have proved ().

Using the arguments used in the above example, we obtain a more general result.
Theorem 2.10 Suppose (n ) and (n ) are sequences of positive real numbers and
f is a (real valued) function defined on (0, ) having the following property: For
n N, x R,
n < x < n + 1 = n f (x) n .
If (n ) and (n ) converge to the same limit, say b, then lim f (x) = b.
x

2.1.7

Additional Exercises

x
= 1.
x3 4x 9

x,
if x < 1,
2. Show that the function f defined by f (x) =
does not
1 + x, if x 1
have the limit as x 1.

3 x, if x > 1,
1,
if x = 1, Find lim f (x). Is it f (1)?
3. Let f be defined by f (x) =
x1

2x,
if x, 1.
1. Using the definition of limit, show that lim

4. Let f be defined on a deleted neighbourhood D0 of a point x0 and lim f (x) =


xx0

b. If b 6= 0, then show that there exists > 0 such that f (x) 6= 0 for every
x (x0 , x0 + ) D0 .

Continuity of a Function

5. Let f be defined by f (x) =

43

1, if x Q,
Show that
0, if x 6 Q.

(i) lim f (x) does not exist, and


x0

(ii) lim xf (x) = 0.


x0

6. Suppose lim f (x) = and lim g(x) = b. Show that lim g(f (x)) = b.
x

7. Let f : (0, ) R be such that lim f (x) = b. Show that lim f (x1 ) = b.
x0

2.2
2.2.1

Continuity of a Function
Definition and some basic results

Definition 2.8 Let f be a real valued function defined on an interval I and a I.


The f is said to be continuous at a if lim f (x) = f (a).

xa

Definition 2.9 Let f be a real valued function defined on an interval I. Then f is


said to be continuous on I if f is continuous at every a I.

Thus, f is continuous at a I if and only if for every > 0, there exists > 0
such that
|f (x) f (a)| < whenever x I, |x a| < .
By Theorems 2.2 and 2.3, we have the following:
Theorem 2.11 A function f : I R is continuous at a I if and only if for every
sequence (xn ) in I with xn a, we have f (xn ) f (a).
Also, using Theorem 2.4, we obtain the following.
Theorem 2.12 Suppose f and g are defined on an interval I and both f and g are
continuous at a I. Then we have the following.
(i) f + g and f g are continuous at a.
(ii) If g(a) 6= 0, then there exists 0 > 0 such that g(x) 6= 0 for every x I with
|x a| < 0 and f /g is continuous at a.
From Theorem 2.8, we have the following.
Theorem 2.13 Suppose f is continuous at a point x0 and g is continuous at the
point y0 := f (x0 ). Then g f is continuous at x0 .
The following property of a continuous function is worth noticing.

44

Limit, Continuity and Differentiability of Functions

M.T. Nair

Theorem 2.14 Suppose f is a continuous function defined on an interval I and


x0 I is such that f (x0 ) > 0. Then there exists > 0 such that f (x) > 0 for all
x I (x0 , x0 + ).
Proof. Suppose the the conclusion in the theorem does not hold. Then for every
> 0, there exists x I (x0 , x0 + ) such that f (x) 0. In particular,
for each n N, taking n = 1/n, there exists xn I (x0 1/n, x0 + 1/n) such
that f (xn ) 0. Thus, we have xn x0 as n . Hence, by Theorem 2.11,
f (xn ) f (x0 ). Since f (xn ) 0 for all n N, we have f (x0 ) 0, which contradicts
the assumption that f (x0 ) > 0.

2.2.2

Some examples

EXAMPLE 2.8 If f (x) is a polynomial, say f (x) = a0 + a1 x + . . . + ak xk , then f


is continuous on R

EXAMPLE 2.9 For given a R, let f (x) = |x a|, x R. Then f is continuous
at every x0 R. To see this, note that
|f (x) f (x0 )| = ||x a| |x0 a|| |x x0 .
Hence, for every > 0, we have
|x x0 | < = |f (x) f (x0 )| < .

EXAMPLE 2.10 Let f (x) =
continuous on R.

x2 4
x2

for x R \ {2} and f (2) = 4. Then f is




EXAMPLE 2.11 The functions f and g defined by f (x) = sin x and g(x) = cos x
for x R are continuous at 0.
Note that for x, y R,
sin x sin y = 2 sin

x y 
2

cos

x + y 
2

so that
| sin x sin y| |x y|

x, y R.

Hence, for every > 0 and for every x0 R,


|x x0 | < = | sin x sin x0 | < .
Thus, f (x) = sin x is continuous at every x R. Since cos x1 2 sin2 (x/2), x R,
it also follows that g(x) = cos x is continuous at every x R.

EXAMPLE 2.12 Let f (x) =
at every point in R.

sin x
x

for x 6= 0 and f (0) = 1. Then f is continuous




Continuity of a Function

45

EXAMPLE 2.13 Let f (x) = x1 for x 6= 0. Then there does not exist a continuous
function g defined on R such that g(x) = f (x) for all x 6= 0.

EXAMPLE 2.14 The function f defined by f (x) = 1/x, x 6= 0 is continuous at
every x0 6= 0:
Note that for x 6= 0, x0 6= 0,


1

1 = |x x0 | 2|x x0 |
x x0
|xx0 |
|x20 |

whenever

|x x0 |

|x0 |
2

since |x| = |x0 (x0 x)| |x0 | |x0 x|. Hence, for any > 0,


2

1
1 < whenever |x x0 | < := min{ x0 , x0 }.
x x0
2
2
Thus, f is continuous at every x0 6= 0.

EXAMPLE 2.15 Let f be defined by f (x) = 1/x on (0, 1]. Then there does not
exist a continuous function g on [0, 1] such that g(x) = f (x) for all x (0, 1].
Suppose g is any function defined on [0, 1] such that g(x) = f (x) for all x (0, 1].
Then we have 1/n 0 but g(1/n) = f (1/n) = n . Thus, g(1/n) 6 g(0).


EXAMPLE 2.16 The function f defined by f (x) = x, x 0 is continuous at


every x0 0:
Let > 0 be given. First consider the point x0 = 0. Then we have

|f (x) f (x0 )| = x < whenever |x| < 2 .


Thus, f is continuous at x0 = 0. Next assume that x0 > 0. Since |x x0 | =

( x + x0 )| x x0 |, we have

|x x0 |
|x x0 |
| x x0 | =
.

x0
x + x0
Thus,

| x x0 | < whenever

|x x0 | < := x0 .


More generally, we have the following example.


EXAMPLE 2.17 Let k N. Then the function f defined by f (x) = x1/k , x 0
is continuous at every x0 0:
Let > 0 be given. First consider the point x0 = 0. Then we have
|f (x) f (x0 )| = x1/k < whenever

|x| < k .

Thus, f is continuous at x0 = 0. Next assume that x0 > 0. Let y = x1/k and


1/k
y0 = x0 . Since
y k y0k = (y y0 )(y k1 + y k2 y0 + . . . + yy k2 + y0k1 ),

46

Limit, Continuity and Differentiability of Functions

M.T. Nair

so that
1/k

x x0 = (x1/k x0 )(y k1 + y k2 y0 + . . . + yy k2 + y0k1 ).


Hence,
1/k

|x1/k x0 | =

|x x0 |
|x x0 |

.
k1
y k1 + y k2 y0 + . . . + yy k2 + y0
y0k1

Thus,
1/k

|x1/k x0 | < whenever

11/k

|x x0 | < := y0k1 = x0

Thus, f is continuous at every x0 > 0.

.


EXAMPLE 2.18 For a rational number r, let f (x) = xr for x > 0. Then using
the above example together with Theorem 2.13, we see that f is continuous at every
x0 > 0.

We know that given r R, there exists a sequence (rn ) of rational numbers such
that rn r. For n N, let fn (x) = xrn , x > 0. Since each fn is continuous for
x > 0, one may enquire whether the function f defined by f (x) = xr is continuous
for x > 0.
First of all how do we define the xr for x > 0?
We shall discuss this issue in the next subsection, where we shall introduce two
important classes of functions, namely, exponential and logarithm functions. In fact,
our discussion will also include, as special cases, the Examples 2.16 - 2.18.

2.2.3

Exponential and logarithm functions

We have already come across expression such as ab for a > 0 and b R, though we
have not proved existence of such numbers, and also defined a number denoted by



X
1 1/n
1
e as the limit of the sequence 1 +
or the series
. Now, we formally
n
n!
n=0
define the following functions:
Exponential function: ex , x R.
Natural logarithm function: ln x, x > 0.
Exponential function: ax , x R for a given a > 0.
Logarithm function with base a > 0:

loga x, x > 0.
P
xn
First, we observe that for every x R, the series
n=0 n! converges absolutely.
This can be seen by using the ratio test. This series plays a very significant role in
mathematics.

Continuity of a Function

47

Definition 2.10 For x R, let


exp(x) :=

X
xn
n=0

n!

The function exp(x), x R, is called the exponential function.

From the above definition, it is clear that


exp(0) = 1

and

exp(1) = e.

In order to derives some of the important properties of the function exp(x), the
student is urged to do the following exercise.
P
P
Exercise 2.9 Suppose that series
n=0 bn are absolutely convergent.
n=0 an and
Then, the series

n
X
X
ak bnk
()
cn with cn :=
n=0

k=0

P
P
is absolutely convergent. Further, show that if =
n=0 bn , then
n=0 an and =
P

c
=
.
n=0 n
P
The
series
Pn=0 cn defined in () is called the Cauchy product of the series
P
J
n=0 cn .
n=0 cn and
Using the conclusion in the above exercise, it can be proved that
exp(x + y) = exp(x) exp(y)

x, y R.

()

Exercise 2.10 Prove () above.

We observe the following properties:


exp(x) =

1
exp(x)

x R. In particular, since exp(x) > 0 for x 0, we

have
exp(x) > 0

x R.

[This follows from ()]


exp(x) > 1 x > 0

and

exp(x) = 1 x = 0.

[From the definition, x > 0 implies exp(x) > 1. Next, suppose x 0. If x = 0, then
exp(x) = exp(0) = 1. If x < 1, then taking y = x, we have y > 1, and hence from
the first part, exp(y) > 1, i.e., 1/ exp(x) = exp(x) > 1 so that exp(x) < 1. Hence,
exp(x) > 1 x > 0. From this, we get exp(x) = 1 x = 0.]
x > y exp(x) > exp(y).
[x > y x y > 0 exp(x y) > 1. But, exp(x y) = exp(x)/ exp(y).]

48

Limit, Continuity and Differentiability of Functions


exp(kx) = [exp(x]k

x R, k Z. In particular, taking x = 1 and x = 1,


exp(k) = ek

k Z.

Since e = exp(1) = exp(k/k) = [exp(1/k]k


exp(1/k) = e1/k
exp(m/n) = em/n

M.T. Nair

k N, we have
k N.

m, n N. Hence,
exp(r) = er

r Q.

We know that every real number is a limit of a sequence of rational numbers.


Thus, if r R, there exits a sequence (rn ) of rational numbers that rn r. So, it
is natural to define
er = lim ern
n

provided the above limit exists. Thus, our next attempt is to show that the function
exp(x), x R, is continuous.
In view of these observations, we use the following
Proposition 2.15 The following results hold.
(i) exp(x) as x .
(ii) exp(x) 0 as x .
Proof. For x 0, we have
exp(x) = 1 + x +

X
xn
n=2

n!

1 + x.

From this (i) follow. To see (ii), let x < 0 and y = x. Then y > 0 so that by (ii),
exp(x) = exp(y) =

1
0
exp(y)

as

y .

Thus, exp(x) 0 as x .

NOTATION: For brevity of expression, we shall use the notation ex for exp(x).
Theorem 2.16 The function exp(x) is continuous on R

Continuity of a Function

49

Proof. Let x, x0 R. Then we have


ex ex0 = ex0 (exx0 1) = ex0

X
(x x0 )n

n!

n=1

= ex0 (x x0 )

X
(x x0 )n1
n=1

n!

Thus, if |x x0 | 1, then

X
1
= ex0 (e 1)|x x0 |.
|e e | e |x x0 |
n!
x

x0

x0

n=1

Hence, for every > 0,


|ex ex0 | < whenever

|x x0 | < min{1, /[ex0 (e 1)]}

so that ex is a continuous function for x R.


Theorem 2.17 The function ex is bijective from R to (0, ).
Proof. First we observe that, for x1 , x2 in R
ex2 ex1 = ex1 [ex2 x1 1].
Thus,
ex2 = ex1 ex2 x1 = 1 x1 = x2 ,
showing that the function x 7 ex is one-one.
Next, to show the function is onto, let y (0, ). Since, by Proposition 2.15,
ex 0

as

ex as

x ,

x ,

by intermediate value property, there exists x R such that ex = y.


Definition 2.11 For b > 0, the unique a R such that ea = b is called the natural
logarithm b, and it is denoted by ln b. The function
ln x,

x > 0,

is called the natural logarithm function.

Definition 2.12 For a > 0 and b R, we define


ab := eb ln a .

Remark 2.4 We note that ln e = 1 so that if a = e, then the Definition 2.12
matches with Definition 2.10.

Theorem 2.18 Let a > 0. Then the function ax is continuous and bijective from
R to (0, ).

50

Limit, Continuity and Differentiability of Functions

M.T. Nair

Proof. Note that for x R, ax := ex ln a . Hence, the result is a consequence of


Theorems 2.16 and 2.17, and the Definition 2.12, and using the fact that composition
of two continuous functions is continuous.
Definition 2.13 Let a > 0. For c > 0, the unique b R such that ab = c is called
the logarithm of c to the base a, and it is denoted by loga c. The function
loga x,

x > 0,

is called the logarithm function.

We observe that following.


For y R,

y = ln x ey = x.

For a > 0 and y R,


For a > 0 and x > 0,

y = loga x ay = x.
loga x =

ln x
.
ln a

Exercise 2.11 For a > 0, b > 0, show that logb a loga b = 1.

Theorem 2.19 The functions ln x and loga x for a > 0 are continuous on (0, ).
Proof. Let x, x0 belong to the interval (0, ), and let y = ln x and y0 = ln x0 .
Then we have ey = x and ey0 = x0 . Assume, without loss of generality that x > x0 .
Since ea > 1 if and only if a > 0, we have y > y0 , and hence
x x0 = ey ey0 = ey0 (eyy0 1) = ey0

X
(y y0 )n
n=1

n!

ey0 (y y0 ).

Hence,
|y y0 | ey0 |x x0 |.
Thus, for > 0, we have |y y0 | < whenever x x0 | < ey0 , ln x is continuous on
(0, ). Since loga x = ln x/ ln a, the function loga x is also continuous on (0, ).
Theorem 2.20 For r R, the function f defined by f (x) = xr is continuous at
every point x (0, ).
Proof. For r R and x > 0, we have xr = er ln x . Hence, the result follows from
Theorem 2.19 and Theorem 2.13.
Remark 2.5 Often, the notation log x is used for the natural logarithm function
instead of ln x.


Continuity of a Function

2.2.4

51

Some properties of continuous functions

Recall that a subset S of R is said to be bounded if there exists M > 0 such that
|s| M for all s S, and set which is not bounded is called an unbounded set.
Recall that if S is a bounded subset of R, then S has infimum and supremum.
Exercise 2.12 Let S R. Prove the following:
(i) Suppose S is bounded, and say := inf S and := sup S. Then there exist
sequences (sn ) and (tn ) in S such that sn and tn .
(ii) S is unbounded if and only if there exists a sequence (sn ) in S which is
unbounded.
(iii) S is unbounded if and only if there exists a sequence (sn ) in S such that
|sn | as n .
(iv) If (sn ) is a sequence in S which is unbounded, then there exists a subsequence
(skn ) of (sn ) such that |skn | as n .
(v) If (sn ) is a sequence in S such that |sn | as n , and if (skn ) is
J
subsequence of (sn ), then |skn | as n .
Definition 2.14 A real valued function defined on a set D R is said to be a
bounded function if the set {f (x) : x D} is is bounded. A function is said to
be an unbounded function if it is not bounded.

The following can be easily deduced from the definition:
A function f : D R is bounded if and only if there exists M > 0 such that
|f (x)| M for all x D.
A function f : D R is not unbounded if and only if there exists a sequence
(xn ) D such that the |f (xn )| as n .
Theorem 2.21 Suppose f is a real valued function defined on a closed and bounded
interval [a, b]. Then f is a bounded function.

Proof. Assume for the time being that f : [a, b] R is not a bounded function.
Then for every n N, there exists a sequence (xn ) in [a, b] such that |f (xn )|
as n . Since (xn ) is a bounded sequence, by Bolzano-Weierstrass property of
R, there exists a subsequence (xkn ) of (xn ) such that xkn x for some x [a, b].
Therefore, by continuity of f , f (xkn ) f (x). In particular, (f (xkn ) is a bounded
sequence. This is a contradiction to the fact that |f (xn )| as n .
Thus, we have proved that continuous function cannot be unbounded.

52

Limit, Continuity and Differentiability of Functions

M.T. Nair

Attaining max f and min f


Theorem 2.22 Suppose f is a continuous real valued function defined on a closed
and bounded interval [a, b]. Then there exists x0 , y0 in [a, b] such that
f (x0 ) = := inf{f (x) : x [a, b]},

f (y0 ) = := sup{f (x) : x [a, b]}.

Proof. By the definition of infimum and supremum, there exist sequences (xn )
and (yn ) in [a, b] such that f (xn ) and f (yn ) as n . Since (xn )
and (yn ) are bounded sequences, there exist subsequences (xkn ) and (ykn ) of (xn )
and (yn ), respectively, such that xkn x and ykn y for some x, y in [a, b]. By
continuity of f , f (xkn ) f (x) and f (ykn ) f (y) as n . But, we already have
f (xkn ) and f (ykn ) . Hence, = f (x) and = f (y).
Remark 2.6 By Theorem 2.22, we say that the infimum and supremum of a continuous real valued function f defined on a closed and bounded interval [a, b] are
attained at some points in [a, b], and in that case, we write
inf{f (x) : x [a, b]} = min f (x),
axb

sup{f (x) : x [a, b]} = max f (x).


axb

The conclusion in the above theorem need hold if the domain of the function is not
of the form [a, b] or if f is not continuous. For example, f : (0, 1] R defined by
f (x) = 1/x for x (0, 1] is continuous, but does not attain supremum. Same is the
case if g : [0, 1] R is defined by
 1
x , x (0, 1],
g(x) =
1, x = 0.
Thus, neither continuity nor the fact that the domain is a closed and bounded
interval cannot be dropped. This does not mean that the conclusion in the theorem
does not hold for all such functions! For example f : [0, 1) R defined by

0, x [0, 1/2),
f (x) =
1, x [1/2, 1).
Then we see that neither f is continuous, nor its domain of the form [a, b]. But, f
attains both its maximum and minimum.

Intermediate value theorem
Suppose f is a continuous real valued function f defined on a closed and bounded
interval [a, b], and
:= min f (x),
:= max f (x).
axb

axb

Clearly,
f (x)

x [a, b].

Continuity of a Function

53

Now, the question is whether every value between and is attained by the function. The answer is in affirmative. In fact we have the following general theorem,
known as Intermediate value theorem1
Theorem 2.23 (Intermediate value theorem) Suppose f is a continuous real
valued function defined on an interval I. Suppose x1 and x2 are in I such that
f (x1 ) < f (x2 ), and c is such that f (x1 ) < c < f (x2 ). Then there exists x0 lying
between x1 and x2 such that f (x0 ) = c.
Proof. Without loss of generality assume that x1 < x2 . Let
S = {x [x1 , x2 ] : f (x) < c}.
Then S is non-empty (since x1 S) and bounded above (since x x2 for all x S).
Let
:= sup S.
Then there exists a sequence (an ) in S such that an . Note that [x1 , x2 ].
Hence, by continuity of f , f (an ) f (). Since f (an ) < c for all n N, we have
f () c. Note that 6= b (since f () c < f (x2 )).
Now, let (bn ) be a sequence in (, x2 ) such that bn . Then, again by
continuity of f , f (bn ) f (). Since bn > , bn 6 S and hence f (bn ) c. Therefore,
f () c. Thus, we have prove that there exists x0 := such that f (x0 ) c f (x0 )
so that f (x0 ) = c.
The following two corollaries are immediate consequences of immediate from the
above theorem.
Theorem 2.24 Let f be a continuous function defined on an interval. Then range
of f is a an interval.
Corollary 2.25 Suppose f is a continuous real valued function defined on an interval I. If a, b I satisfy a < b and f (a)f (b) < 0, then there exists x0 I such
that
a x0 b and f (x0 ) = 0.

2.2.5

Additional Exercises

1. Suppose f : [a, b] R is continuous. If c (a, b) is such that f (c) > 0, and


if 0 < < f (c), then show that there exists > 0 such that f (x) > for all
x (c , c + ) [a, b].
2. Let f : R R satisfy the relation f (x + y) = f (x) + f (y) for every x, y R.
If f is continuous at 0, then show that f is continuous at every x R, and in
that case f (x) = xf (1) for every x R.
1

Proof taken from the book: A Course in Calculus and Real Analysus by S.R. Ghorpade and
B.V. Limaye (IIT Bombay), Springer, 2006.

54

Limit, Continuity and Differentiability of Functions

M.T. Nair

3. There does not exist a continuous function f from [0, 1] onto R Why?
4. Find a continuous function f from (0, 1) onto R.
5. Suppose f : [a, b] [a, b] is continuous. Show that there exists c [a, b] such
that f (c) = c.
6. There exists x R such that 17x19 19x17 1 = 0 Why?
7. If p(x) is a polynomial of odd degree, then there exists at least one R such
that p() = 0.
8. Suppose f : R R is continuous such that f (x) 0 as |x| . Prove that
f attains either a maximum or a minimum.
9. Suppose f : [a, b] R is continuous such that for every x [a, b], there exists
|f (x)|
a y [a, b] such that |f (y)|
. Show that there exists [a, b] such
2
that f () = 0.
1
10. Suppose f : [a, b] [a, b] is continuous such that there |f (x) f (y)| |x y|
2
for all x, y [a, b]. Show that there exists [a, b] such that f () = .

2.3

Differentiability of functions

Definition 2.15 Suppose f is a (real valued) function defined on an open interval


I and x0 I. Then f is said to be differentiable at x0 if
lim

xx0

f (x) f (x0 )
x x0

exists, and in that case the value of the limit is called the derivative of f at x0 .
The derivative of f at x0 , if exists, is denoted by
f 0 (x0 )

or

df
(x0 ).
dx


Exercise 2.13 Show that f : I R is differentiable at x0 I if and only if


(x0 )
limh0 f (x0 +h)f
exists.
J
h
Exercise 2.14 Suppose f is defined on an open interval I and x0 I. Show that
f is differentiable at x0 I if and only if there exists a continuous function (x)
such that
f (x) = f (x0 ) + (x)(x x0 ),
and in that case (x0 ) = f 0 (x0 ).

Differentiability of functions

55

Exercise 2.15 Let be as in Exercise 2.14. Then f is differentiable at x0 , if and


only if for every sequence (xn ) in I \{x0 } which converges to x0 , the sequence f (xn )
converges, and in that case f 0 (x0 ) = limn f (xn ).
J
Exercise 2.16 Suppose f and g defined on I are differentiable at x0 I and
R. Show that the functions (x) := f (x) + g(x) and (x) := f (x), x I are
differentiable at x0 , and
0 (x0 ) = f 0 (x0 ) + g 0 (x0 ),

0 (x0 ) = 0 (x0 ).
J

2.3.1

Some properties of differentiable functions

Theorem 2.26 (Differentiability implies continuity) Suppose f defined on I


is differentiable at x0 I. Then f is continuous at x0 .
Proof. Note that
f (x0 + h) f (x0 ) =

f (x0 + h) f (x0 )
h f 0 (x0 ).0 = 0
h

as h 0.

Thus, f is continuous at x0 .
For the following theorem, we may recall that if is continuous at a point x0 and
(x0 ) 6= 0, then there exists an open interval I0 containing x0 such that (x) 6= 0
for all x I0 .
Theorem 2.27 (Products and quotient rules) Suppose f and g defined on I
are differentiable at x0 I. Then the function (x) := f (x)g(x) is differentiable at
x0 , and
0 (x0 ) = f 0 (x0 )g(x0 ) + f (x0 )g 0 (x0 ).
()
If g(x0 ) 6= 0, then the function (x) := f (x)/g(x) is differentiable at x0 , and
0 (x0 ) =

g(x0 )f 0 (x0 ) f (x0 )g 0 (x0 )


.
[g(x0 )]2

()

Proof. Note that


(x0 + h) (x0 ) = f (x0 + h)g(x0 + h) f (x0 )g(x0 )
= [f (x0 + h) f (x0 )]g(x0 + h) + f (x0 )[g(x0 + h) g(x0 )]
so that
(x0 + h) (x0 )
h

f (x0 + h) f (x0 )
g(x0 + h) g(x0 )
g(x0 + h) + f (x0 )
h
h
f 0 (x0 )g(x0 ) + f (x0 )g 0 (x0 ) as h 0.
=

56

Limit, Continuity and Differentiability of Functions

M.T. Nair

Hence, is differentiable at x0 , and


0 (x0 ) = f 0 (x0 )g(x0 ) + f (x0 )g 0 (x0 ).
Also, since
(x0 + h) (x0 ) =
=

f (x0 + h)g(x0 ) f (x0 )g(x0 + h)


g(x0 + h)g(x0 )
[f (x0 + h) f (x0 )]g(x0 ) f (x0 )[g(x0 + h) g(x0 )]
,
g(x0 + h)g(x0 )

we have
(x0 + h) (x0 )
h

f (x0 +h)f (x0 )


g(x0 )
h

0)
f (x0 ) g(x0 +h)g(x
h
g(x0 + h)g(x0 )
0
f (x0 )g(x0 ) f (x0 )g 0 (x0 )
as h 0.
[g(x0 )]2

Thus, is differentiable at x0 , and 0 (x0 ) =

g(x0 )f 0 (x0 )f (x0 )g 0 (x0 )


.
[g(x0 )]2

Theorem 2.28 (Composition rule) Suppose f is defined on an open interval I


and x0 I, and g is defined in an open interval containing y0 := f (x0 ). Let
= g f.
Then we have the following.
(i) Suppose f is differentiable at x0 and g is differentiable at y0 . Then is
differentiable at x0 and
0 (x0 ) = g 0 (y0 )f 0 (x0 ).
(ii) Suppose is differentiable at x0 , g is differentiable at y0 and g 0 (y0 ) 6= 0. Then
f is differentiable at x0 and
f 0 (x0 ) =

0 (x0 )
.
g 0 (y0 )

(iii) Suppose is differentiable at x0 , f is differentiable at x0 and f 0 (x0 ) 6= 0. Then


g is differentiable at y0 and
g 0 (y0 ) =

0 (x0 )
.
f 0 (x0 )

Proof. For x 6= x0 , let


(x) :=

(g f )(x) (g f )(x0 )
.
x x0

Differentiability of functions

57

Let (xn ) be a sequence in I \ {x0 } which converges to x0 . Then, taking yn := f (xn ),


n N, and y0 = f (x0 ), we have
(xn ) =
=
=

(g f )(xn ) g f )(x0 )
xn x0
g(yn ) g(y0 )
xn x0
g(yn ) g(y0 ) f (xn ) f (x0 )

.
yn y0
xn x0

(i) Suppose f is differentiable at x0 . Now, since xn x0 we have, by continuity


of f at x0 , yn y0 . Therefore,
f (xn ) f (x0 )
f 0 (x0 ),
xn x0

g(yn ) g(y0 )
g 0 (y0 ).
yn y0

Thus, (xn ) g 0 (y0 )f 0 (x0 ) showing that g f is differentiable at x0 and


(g f )0 (x0 ) = g 0 (y0 )f 0 (x0 ).
(ii) Suppose g f is is differentiable at x0 and g 0 (y0 ) 6= 0. Then we have
(xn ) (g f )0 (x0 ) and
f (xn ) f (x0 )
=
xn x0

(xn )
g(yn )g(y0 )
yn y0

Hence, f is differentiable at x0 and f 0 (x0 ) =

(g f )0 (x0 )
.
g 0 (y0 )

(g f )0 (x0 )
.
g 0 (y0 )

(iii) Proof of this part is analogous to the proof of (ii). Hence, we omit the
details.
Remark 2.7 The part (ii) and (iii) of Theorem 2.28 is not available in standard
books on Calculus. I found it useful while discussing derivative of logarithm function
in next section (Section 2.3.2).

Exercise 2.17 Prove part (iii) of Theorem 2.28.

2.3.2

Some Examples

We shall assume that the students are familiar with the following:
For c R, if f (x) = c, x R, then f 0 (x) = 0

x R.

If f (x) = x, x R, then f 0 (x) = 1 x R.


If f (x) = sin x, x R, then f 0 (x) = cos x

x R.

58

Limit, Continuity and Differentiability of Functions

M.T. Nair

If f (x) = cos x = f 0 (x) = sin x.


From these, using theorems in the last subsection, we obtain the following:
For n N, if f (x) = xn , x R, then f 0 (x) = nxn1

x R.

If f (x) = cos x = 1 2 sin2 (x/2), x R, then f 0 (x) = sin x

x R.

If f (x) = tan x for x D := {x R : cos x 6= 0}, then f 0 (x) = sec2 x

x D.

EXAMPLE 2.19 The function ex is differentiable for every x R and


(ex )0 = ex

x R.

We note that for h 6= 0,

ex+h ex
ex
ex X hn
ex = (eh 1 h) =
.
h
h
h
n!
n=2

Now, if |h| 1, then |h|n |h|2 for all n {2, 3, . . .}. Thus,

x+h

x
X

e
1

e
x
e ex |h|
= ex |h|(e 2).
|h| 1 =
h
n!
n=2

From this we obtain that ex is differentiable at x and its derivative is ex .

EXAMPLE 2.20 For a > 0, the function ax is differentiable for every x R and
(ax )0 = ax ln a

x R.

By the composition rule in Theorem 2.28,


(ax )0 = (ex ln a )0 = ex ln a ln a = ax ln a.

EXAMPLE 2.21 The function ln x is differentiable for every x > 0, and
(ln x)0 =

1
,
x

x > 0.

To see this, let f (x) = ln x and g(x) = ex . Then we have g(f (x) = x for every x > 0.
Since g f is differentiable, g is differentiable, and g 0 (y) = ey 6= 0 for every y R,
by Theorem 2.28, f is differentiable for every x > 0 and we have g 0 (f (x))f 0 (x) = 1.
Thus,
1 = eln x (ln x)0 = x(ln x)0
so that (ln x)0 = 1/x.

Differentiability of functions

59

EXAMPLE 2.22 For a > 0, the function loga x is differentiable for every x > 0,
and
1
(loga x)0 =
,
x > 0.
x ln a
We know that
ln x
loga x =
.
ln a
1
Hence, (loga x)0 =
for every x > 0.

x ln a
EXAMPLE 2.23 For r R, let f (x) = xr for x > 0. Then f is differentiable for
every x > 0 and
f 0 (x) = rxr1 ,
x > 0.
By the composition rule in Theorem 2.28,
f 0 (x) = (er ln x )0 = er ln x

r
xr r
=
= rxr1 .
x
x


Exercise 2.18 Prove the following.


(i) The function ln |x| is differentiable for every x R with x 6= 0, and
(ln |x|)0 =

1
,
x

x 6= 0.

(ii) For a > 0, the function loga |x| is differentiable for every x R with x 6= 0,
and
1
(loga |x|)0 =
,
x 6= 0.
x ln a
J

2.3.3

Maxima and minima

Recall from Theorem 2.22 that if f : [a, b] R is a continuous function, then there
exists x0 , y0 in [a, b] such that
f (x0 ) f (x) f (y0 )

x [a, b].

In this case, we write


f (x0 ) = min f (x)
axb

and f (y0 ) = max f (x).


axb

Definition 2.16 A (real valued) function f defined on an interval I (of finite or


infinite length) is said to attain
(a) global maximum at a point x1 I if f (x1 ) f (x) for all x I, and

60

Limit, Continuity and Differentiability of Functions

M.T. Nair

(b) global minimum at a point x2 I if f (x2 ) f (x) for all x I.


The function f is said to attain global extremum at a point x0 I if f attains
either global maximum or global minimum at x0 .

Thus, a continuous fiunction f defined on a closed and bounded interval I attain
global maximum and global minimum at some points in I.
In Remark 2.6 we have seen that a function f defined on an interval I need not
attain maximum or minimum if either I is not closed and bounded or if f is not
continuous. However, maximum or minimum can attain in a subinterval. To take
care of these cases, we introduce the following definition.
Definition 2.17 A (real valued) function f defined on an interval I (of finite or
infinite length) is said to attain
(a) local maximum at a point x1 I if there exists > 0 such that
f (x1 ) f (x)

x I (x1 , x1 + ),

(b) local minimum at a point x2 if if there exists > 0 such that


f (x2 ) f (x)

x I (x2 , x2 + ).

The function f is said to attain local extremum at a point x0 I if f attains


either local maximum or local minimum at x0 .

Remark 2.8 It is conventional to omit the adjective local in local maximum, local
minimum and local extremum. Thus when we say a function has maximum at a
point x0 , we generally mean a local maximum at x0 . Similar comments apply to
minimum and extremum.

Theorem 2.29 (Necessary condition) Suppose f is a continuous function defined on an interval I having local extremum at a point x0 I. If x0 is an interior
point of I (i.e., x0 is not an end point of I) and f is differentiable at x0 , then
f 0 (x0 ) = 0.
Proof. Suppose f attains local maximum at x0 which is an interior point of I.
Then there exists > 0 such that (x0 , x0 + h) I and f (x0 ) f (x0 + h) for all
h with |h| < . Hence, for all h with |h| < ,
f (x0 + h) f (x0 )
0 if h < 0,
h
f (x0 + h) f (x0 )
0 if h > 0.
h
Taking limit as h 0, we get f 0 (x0 ) 0 and f 0 (x0 ) 0 so that f 0 (x0 ) = 0.
By analogous arguments, it can be shown that if f attains minimum at a point
y0 (a, b), then f 0 (y0 ) = 0.

Differentiability of functions

61

Remark 2.9 A function can have more than one maximum and minimum. For
example, consider
f (x) = sin(4x),
[0, ].
We see that f has maximum value 1 at /8 and 5/8, and has minimum value 1
at 3/8 and 7/8.

Remark 2.10 (a) In view of Theorem 2.29, if a function f is differentiable at an
interior point x0 of an interval I and f 0 (x0 ) 6= 0, then f can not have local maximum
or local minimum at x0 .
(b) It is to be observed that in order to have a maximum or minimum at a point
x0 , the function need not be differentiable at x0 . For example
f (x) = 1 |x|,

|x| 1,

has a maximum at 0 and


g(x) = |x|,

|x| 1,

has a minimum at 0. Both f and g are not differentiable at 0.


(c) Also, if a function is differentiable at a point x0 and f 0 (x0 ) = 0, then it is not
necessary that it has loal maximum or local minimum at x0 . For example, consider
f (x) = x3 ,

|x| < 1.

In this example, we have f 0 (0) = 0. Note that f has neither local maximum nor
local minimum at 0.

Definition 2.18 Suppose f is defined n an interval I and x0 is an interior point
of I. If f 0 (x0 ) exists and f 0 (x0 ) = 0 or if f 0 (x0 ) does not exist, then x0 is called a
critical point of f .

In Section 2.3.6 we shall give some sufficient conditions for existence of local
exrema of functions. Now, let us derive some important consequences of Theorem
2.29.

2.3.4

Some important theorems

Rolles theorem
Theorem 2.30 (Rolles theorem) Suppose f is a continuous function defined on
a closed and bounded interval [a, b] such that it is differentiable at every x (a, b).
If f (a) = f (b), then there exists c (a, b) such that f 0 (c) = 0.
Proof. Let g(x) = f (x) f (a). Then we have g(a) = 0 = g(b), and g 0 (x) = f 0 (x)
for every x (a, b).

62

Limit, Continuity and Differentiability of Functions

M.T. Nair

We know that g attain maximum and minimum at some points x1 and x2 ,


respectively, in [a, b], i.e., there exists x1 , x2 in [a, b] such that
g(x2 ) g(x) g(x1 )

x [a, b].

If g(x1 ) = g(x2 ), then g is a constant function and hence g 0 (x) = 0 for all x [a, b].
Hence, assume that g(x2 ) < g(x1 ). Then, either g(x1 ) 6= 0 or g(x2 ) 6= 0. Assume
that g(x2 ) 6= 0, so that x2 6= a and x2 6= b. Hence, by Theorem 2.29, g 0 (x2 ) = 0.
Thus, f 0 (x2 ) = 0.
Similarly, if g(x1 ) 6= 0, then we shall arrive at f 0 (x1 ) = 0.
Exercise 2.19 Show that between any two roots of the equation ex cos x 1 = 0,
there is at least one root of the equation ex sin x 1 = 0.
J
Lagranges mean value theorem
As a corollary to Rolles theorem we obtain the following.
Theorem 2.31 (Lagranges mean value theorem) Suppose f is a continuous
function defined on a closed and bounded interval [a, b] such that it is differentiable
at every x (a, b). Then there exists c (a, b) such that
f (b) f (a) = f 0 (c)(b a).
Proof. Let
f (b) f (a)
(x a),
x [a, b].
ba
Note that is continuous on [a, b], differentiable in (a, b), (a) = 0 = (b), and
(x) := f (x) f (a)

f (b) f (a)
,
x (a, b).
ba
By Rolles theorem (Theorem 2.30), there exists c (a, b) such that 0 (c) = 0.
Thus, f (b) f (a) = f 0 (c)(b a).
0 (x) := f 0 (x)

EXAMPLE 2.24 Let f be continuous on [a, b] and differentiable at every point in


(a, b). Suppose there exists c R such that
f 0 (x) = c

x (a, b).

Then there exists b R such that


f (x) = c x + b

x [a, b].

In particular, f 0 (x) = 0 for all x (a, b), then f is a constant function.


To see this consider x0 (a, b). Then for any x [a, b], there exists x between
x0 and x such that
f (x) f (x0 ) = f 0 (x )(x x0 ) = c(x x0 ).
Hence, f (x) = f (x0 ) + c(x x0 ). Thus, f (x) = c x + b with b = f (x0 ) c x0 .

Differentiability of functions

63

Cauchys generalized mean value theorem


Suppose f and g are continuous functions on [a, b] which are differentiable on (a, b).
Suppose further that g 0 (x) 6= 0 for all x (a, b). Then, by Lagranges mean value
theorem, there exist c1 , c2 in (a, b) such that
f (b) f (a)
f 0 (c1 )
= 0
.
g(b) g(a)
g (c2 )
Question is whether we can assert the existence of a single point c (a, b) such
that
f (b) f (a)
f 0 (c)
= 0 .
g(b) g(a)
g (c)
Answer is in affirmative as the following theorem shows.
Theorem 2.32 (Cauchys generalized mean value theorem) Suppose f and
g are continuous functions on [a, b] which are are differentiable at every point in
(a, b). Suppose further that g 0 (x) 6= 0 for all x (a, b). Then, there exists c (a, b)
such that
f 0 (c)
f (b) f (a)
= 0 .
g(b) g(a)
g (c)
Proof. First note that from the assumption on g, using Mean value theorem,
g(b(6= g(a). Now, let
(x) := f (x) f (a)

f (b) f (a)
[g(x) g(a)],
g(b) g(a)

x [a, b].

Note that is continuous on [a, b], differentiable in (a, b), (a) = 0 = (b), and
0 (x) := f 0 (x)

f (b) f (a) 0
g (x),
g(b) g(a)

x (a, b).

By Rolles theorem (Theorem 2.30), there exists c (a, b) such that 0 (c) = 0. This
completes the proof.
Exercise 2.20 Let 0 < a < b. Show that for every n N, a <
[Hint: take f (x) = xn+1 and g(x) = xn .]

n[bn+1 an+1 ]
< b.
(n + 1)[bn an ]
J

If f is defined in a closed interval [a, b] and x0 = a or x0 = b, then by limxx0 f (x)


we mean limxx+ f (x) if x0 = a and limxx f (x) if x0 = b.
0

64

Limit, Continuity and Differentiability of Functions

M.T. Nair

LHospitals rules
Theorem 2.33 (LHospitals rule)2 Suppose f and g are continuous functions
on [a, b] which are differentiable at every point in (a, b), except possibly at x0 [a, b].
f 0 (x)
f (x)
Suppose f (x0 ) = 0 = g(x0 ) and lim 0
exists. Then lim
exists and
xx0 g (x)
xx0 g(x)
lim

xx0

f (x)
f 0 (x)
= lim 0
.
g(x) xx0 g (x)

f 0 (x)
exists, there exists a deleted neighbourhood D0 of x0
xx0 g 0 (x)
such that g 0 (x) 6= 0 for x D0 [a, b]. By Cauchys generalized mean value theorem
(Theorem 2.32), for every x D0 , there exists x between x and x0 such that
Proof. Since lim

f (x) f (x0 )
f 0 (x )
f (x)
=
= 0
.
g(x)
g(x) g(x0 )
g (x )
f 0 (x )
f 0 (x)
exists,
lim
exists and it is equal to
xx0 g 0 (x )
xx0 g 0 (x)
f 0 (x)
f (x)
f (x)
f 0 (x)
lim 0
(See Exercise 2.4). Thus, lim
exists and lim
= lim 0
.
xx0 g (x)
xx0 g(x)
xx0 g(x)
xx0 g (x)
This completes the proof.
Since |x x0 | < |x x0 | and lim

The following theorem is proved by modifying the arguments in the proof of


Theorem 2.36 .
Theorem 2.34 (LHospitals rule) Suppose f and g are continuous functions on
[a, b] which are differentiable at every point in (a, b), except possibly at x0 [a, b].
f (x)
f 0 (x)
Suppose lim f (x) = 0 = lim g(x) and lim 0
exists. Then lim
exists
xx0
xx0 g(x)
xx0
xx0 g (x)
and
f (x)
f 0 (x)
= lim 0
.
lim
xx0 g(x)
xx0 g (x)

f (x) if x 6= x0
g(x) if x 6= x0
and g(x) =
. Then,
0
if x = x0
0
if x = x0
the result is obtained from Theorem 2.36 by taking f and g in place of f and g,
respectively.
Proof. Let f(x) =

The following theorem is proved using the above theorem by using the change
of variable x 7 y := 1/x.
2
LHospital is pronounced as Lopital. The rule is named after the 17th-century French mathematician Guillaume de lHospital, who published the rule in his book Analyse des Infiniment Petits
pour lIntelligence des Lignes Courbes (i.e., Analysis of the Infinitely Small to Understand Curved
Lines) (1696), the first textbook on differential calculus.

Differentiability of functions

65

Theorem 2.35 (LHospitals rule) Suppose f and g are differentiable at every


f 0 (x)
point in (a, ) for some a > 0. Suppose lim f (x) = 0 = lim g(x) and lim 0
x
x
x g (x)
f (x)
exists. Then lim
exists and
x g(x)
f (x)
f 0 (x)
= lim 0
.
x g(x)
x g (x)
lim

Proof. Let f(y) = f (1/y) and g(y) = g(1/y) for 0 < y < 1/a. We note that
lim f (x) = 0 = lim g(x) lim f(y) = 0 = lim g(y).

y0

y0

Also, since
f0 (y) = [f (1/y)]0 = f 0 (1/y)(1/y 2 ),
we have

f 0 (x)
x g 0 (x)
lim

exists

g0 (y) = [g(1/y)]0 = g 0 (1/y)(1/y 2 ),


f0 (y)
y0 g
0 (y)

lim

exists.

Hence, applying Theorem 2.36 to f, g instead of f, g, we obtain the result.


The following theorem also holds.
Theorem 2.36 (LHospitals rule) Suppose f and g are continuous functions on
[a, b] which are differentiable at every point in (a, b), except possibly at x0 [a, b].
f 0 (x)
f (x)
Suppose lim f (x) = = lim g(x) and lim 0
exists. Then lim
exists
xx0
xx0
xx0 g (x)
xx0 g(x)
f (x)
f 0 (x)
and lim
= lim 0
.
xx0 g(x)
xx0 g (x)
Proof. First consider the case of := lim

xx0

f 0 (x)
6= 0. In this case, since
g 0 (x)

lim f (x) = = lim g(x) lim (1/f (x)) = 0 = lim (1/g(x)),

xx0

xx0

xx0

the result follows from Theorem 2.35 by interchanging the roles of f and g.
To consider the general case where is not necessarily zero, let , x R such
that |x x0 | < | x0 | and sufficiently close to x0 such that (x) 6 g(). This is
guaranteed because, g 0 (x) 6= 0 for x sufficiently close to x0 . Then there exists x,
lying between and x such that have
h
i
f ()
0
1

f (x)
f (x, )
f (x) f ()
f (x)
h
i
=
=
0
g()
g (x,)
g(x) g()
g(x) 1
g(x)

66

Limit, Continuity and Differentiability of Functions

so that
f (x)
=
g(x)

f 0 (x, )
g 0 (x,)

1
h
1

f ()
f (x)
g()
g(x)

M.T. Nair

i
i.

Now, for each such we have






g()
f ()
= 1 = lim 1
.
lim 1
xx0
xx0
f (x)
g(x)
Also, since |x, x0 | < | x0 |, it can be seen using Lemma ?? that
lim

x0

f 0 (x, )
f 0 (x)
=
lim
.
xx0 g 0 (x)
g 0 (x,)

h
1
x, )
h
Hence, using (, ) arguments, it can be shown that lim 0
x0 g (x,)
1
f (x)
exists and
so that lim
xx0 g(x)
i
h
f ()
0
1

f (x)
f (x, )
f (x)
f 0 (x)
i = lim 0
h
lim
= lim 0
.
xx0 g(x)
xx0 g (x)
x0 g (x,)
1 g()
f 0 (

f ()
f (x)
g()
g(x)

i
i exists

g(x)

This completes the proof.


Exercise 2.21 Fill gaps in the proof of the above theorem.

Remark 2.11 The cases


(i)

lim f (x) = 0 = lim g(x),

(ii) lim f (x) = = lim g(x)


xx0

xx0

can be treated analogously to the cases already discussed in the above theorems. 
Taylors formula
Now, we state another important formula in calculus.
Theorem 2.37 (Taylors formula) Suppose f is defined and has derivatives
f (1) (x), f (2) (x), . . . , f (n+1) (x) for x in an open interval I and x0 I. Then, for
every x I, there exists x between x and x0 such that
f (x) = f (x0 ) +

n
X
f (j) (x0 )
j=1

j!

(x x0 )j +

f (n+1) (x )
(x x0 )n+1 .
(n + 1)!

Equivalently, for h with x0 + h I, there exists x between x and x0 such that


f (x0 + h) = f (x0 ) +

n
X
f (j) (x0 )
j=1

j!

hj +

f (n+1) (x ) n+1
h
.
(n + 1)!

Differentiability of functions

67

Proof. Let x I with x 6= x0 , and let


Pn (t) = f (x0 ) +

n
X
f (j) (x0 )

j!

j=1

(t x0 )j ,

t I.

Then Pn is a polynomial of degree n, Pn (x0 ) = f (x0 ) and


Pn(j) (x0 ) = f (j) (x0 ),

j {1, . . . , n}.

Now, let
g(t) = f (t) Pn (t) (x)(t x0 )n+1 ,

t I,

where
(x) :=

f (x) Pn (x)
.
(x x0 )n+1

Note that, by this choice of (x), we have g(x0 ) = 0 and g(x) = 0. Also, we have
g (1) (x0 ) = 0,

g (2) (x0 ) = 0,

...,

g (n) (x0 ) = 0.

Hence, by Rolles theorem, there exists x1 between x0 and x such that g 0 (x1 ) = 0.
Again, by Rolles theorem, there exists x2 between x0 and x1 such that g 00 (x2 ) = 0.
Continuing this, there exists x := xn+1 between x0 and xn such that g (n+1) (x ) = 0.
But,
g (n+1) (t) = f (n+1) (t) Pn(n+1) (t) (x)(n + 1)! = f (n+1) (t) (x)(n + 1)!.
Thus, we have
(x) =

f (n+1) (x )
,
(n + 1)!

so that
f (x) = f (x0 ) +

n
X
f (j) (x)
j=1

j!

(x x0 )j +

f (n+1) (x )
(x x0 )n+1 .
(n + 1)!

Thus the proof is complete.3


Definition 2.19 In the Taylors formula (Theorem 2.37, the term
Rn (x) :=

f (n+1) (x )
(x x0 )n+1
(n + 1)!

is called the remainder term in the formula.


3

This proof adapted from S. Ghorpade & B.V. Limaye: A Course in Calculus and Analysis,
Springer, 2006

68

Limit, Continuity and Differentiability of Functions

M.T. Nair

We observe that if f is infinitely differentiable and if


|Rn (x)| 0

as

for every x I, then


f (x) = f (x0 ) +

X
f (n) (x0 )

j!

n=1

(x x0 )n ,

x I.

()

Definition 2.20 If f can be represented as a series as in (), for all x in a neighbourhood of x0 , then such a series is called the Taylors series of f around the
point x0 .

A natural question that one may ask is:
Doe every infinitely differentiable function in a neighbourhood of x0 has
a Taylors series expansion?
Unfortunately, the answer is negative. For example, if we define

f (x) =

e1/x , x 6= 0,
0,
x = 0,

then it can be seen that f (0) = 0 and f k) (0) = 0 for all k N. Thus, f does not
have the Taylors series expansion around the point 0,
EXAMPLE 2.25 Using Taylors formula, we shall show that
sin x =

X
(1)n x2n+1

(2n + 1)!

n=0

x R.

For this, let f (x) = sin x and x0 = 0. Since f is infinitely differentiable, and
f 2j (0) = 0,

f 2j1 (0) = (1)j

j N,

we have
f (x) = f (x0 ) +

2n+1
X
j=1

= f (x0 ) +
= f (x0 ) +

f (j) (0) j f (2n+2) (x ) 2n+2


x +
x
j!
(2n + 2)!

n
X
f (2j+1) (0)
j=0
n
X
j=0

(2j + 1)!

x2j+1 +

f (2n+2) (x ) 2n+2
x
(2n + 2)!

(1)j 2j+1 f (2n+2) (x ) 2n+2


x
+
x
(2j + 1)!
(2n + 2)!

Differentiability of functions

69

Also, since | sin x| 1, we have


f (2n+2) ( )x2n+2
|x|2n+2


x
0


(2n + 2)!
(2n + 2)!

as

n .

Therefore,
n

h
X
(1)j 2j+1 i

x
0
f (x) f (x0 ) +
(2j + 1)!

as

j=0

and hence, sin x =

X
(1)n x2n+1
n=0

(2n + 1)!

x R.

Exercise 2.22 Suppose f is infinitely differentiable in an open interval I and x0 I.


Further, suppose that there exists M > 0 such that
|f (k) (x)| M

x I,

k N {0}.

Then show that


f (x) = f (x0 ) +

X
f (n) (x0 )
n=1

j!

(x x0 )n ,

x I.

J
Exercise 2.23 Using Taylors formula, prove the following:

(i) cos x =

X
(1)n x2n

(2n)!

n=0

(ii) ex =

X
xn
n=0

n!

for all x R.

for all x R.

(iii)

X
1
=
xn for all x with |x| < 1.
1x
n=0

(iv) tan

x=

X
(1)n x2n+1
n=0

2n + 1

for all x R.

Deduce Madhava-Gregory series:

X (1)n
=
.
4
2n + 1
n=0

70

Limit, Continuity and Differentiability of Functions

2.3.5

Increasing and decreasing functions

M.T. Nair

Definition 2.21 Let f be a function on a set D R. Then f is said to be


(i) monotonically increasing or increasing on D if
x, y D,

x y = f (x) f (y),

(ii) strictly increasing on D if


x, y D,

x < y = f (x) < f (y),

(iii) monotonically decreasing or decreasing on D if


x, y D,

x y = f (x) f (y).

(iv) strictly decreasing on D if


x, y D,

x < y = f (x) > f (y).




Theorem 2.38 Let f be continuous on [a, b] and differentiable on (a, b). Then
(i) f is increasing iff f 0 (x) 0 for all x (a, b).
(ii) f is decreasing iff f 0 (x) 0 for all x (a, b).
(iii) f is strictly increasing if f 0 (x) > 0 for all x (a, b).
(iv) f is strictly decreasing if f 0 (x) < 0 for all x (a, b).
Proof. (i) Suppose f is increasing and x (a, b). Note that
f (x + h) f (x)
f (x + h) f (x)
= lim
.
+
h0
h
h
h0

f 0 (x) = lim
Since,

f (x+h)f (x)
h

0 for h > 0, from the above equality we obtain f 0 (x) 0.

To see the converse and (iii), let x1 , x2 [a, b] with x1 < x2 . Then, by mean
value theorem, theorem there exists (x1 , x2 ) such that
f (x2 ) f (x1 ) = f 0 ()(x2 x1 ).
Hence, if f 0 (x) 0 (respectively, f 0 (x) > 0) for every x (a, b), then f (x1 ) f (x2 )
(respectively, f (x1 ) < f (x2 )). Thus, (i) and (iii) are proved. Similar arguments will
lead to the proof of (ii) and (iv).

Differentiability of functions

71

EXAMPLE 2.26 Consider the function f (x) = x4 for x R. Then we have


f 0 (x) = 4x3 for all x R. Note that
f 0 (x) > 0 x > 0

and f 0 (x) < 0

x < 0.

Hence,
f is strictly increasing on (0, ), and
f is strictly decreasing on (0, ).

2.3.6

More about local maxima and local minima

Theorem 2.39 (A sufficient condition) Suppose f is continuous on an interval


I and x0 is an interior point of I. Further suppose that f is differentiable in a
deleted nbd of x0 .
(i) If there exists an open interval I0 I containing x0 such that
f 0 (x) > 0

x I0 , x < x0

and

f 0 (x) < 0

x I0 , x > x0 ,

then f has loal maximum at x0 .


(ii) If there exists an open interval I0 I containing x0 such that
f 0 (x) < 0

x I0 , x < x0

and

f 0 (x) > 0

x I0 , x > x0 ,

then f has local minimum at x0 .


Proof. (i) Let x I0 . Then, by mean value theorem, there exists x between x0
and x such that
f (x) f (x0 ) = f 0 (x )(x x0 ).
By assumption,
x < x0 = f 0 (x ) > 0

and x > x0 = f 0 (x ) < 0.

Hence, in both the cases, we have f (x) < f (x0 ) so that has maximum at x0 . Thus,
(i) is proved.
Similar arguments will lead to the proof of (ii).
EXAMPLE 2.27 Consider
f (x) = x4 ,

g(x) = 1 x4 ,

|x| < 1.

Then f 0 (x) = 4x3 is negative for x < 0 and positive for x > 0. Hence, by Theorem
2.39, f has local minimum at 0. Also, g 0 (x) = 4x3 is positive for x < 0 and
negative for x > 0. Hence, by Theorem 2.39, g has local maximum at 0.


72

Limit, Continuity and Differentiability of Functions

M.T. Nair

Remark 2.12 The conditions given in Theorem 2.39 are cannot be dropped. For
example, consider f (x) = x3 , x R. Then we see that f 0 (x) = 3x2 > 0 for positive
x and negative x. Note that f does not have extremum at 0.

Theorem 2.40 (Another sufficient condition) Suppose f is defined on an interval I and x0 is an interior point of I. Further, suppose that f continuously twice
differentiable in a neighbourhood of x0 and f 0 (x0 ) = 0. Then we have the following:
(i) If f 00 (x0 ) < 0, then f has local maximum at x0 .
(ii) If f 00 (x0 ) > 0, then f has local minimum at x0 .
Proof. By Taylors theorem, there exists an open interval I0 containing x0 such
that for every x I0 , there exists x between x0 and x such that
f (x) f (x0 ) = f 0 (x0 )(x x0 ) +

f 00 (x )
f 00 (x )
(x x0 )2 =
(x x0 )2 .
2
2

()

(i) Suppose f 00 (x0 ) < 0. Since f 00 is continuous in a nbd of x0 , there exists an


open interval I1 containing x0 such that for all x I1 ,
f 00 (x)

f 00 (x0 )
.
2

In particular, from (), we obtain


f (x) f (x0 ) =

f 00 (x )
(x x0 )2 < 0
2

x I1 .

Thus, f has a maximum at x0 .


(ii) Suppose f 00 (x0 ) > 0. Then, we obtain reverse of the inequalities in the proof
of (i), and arrive the conclusion that f has a minimum at x0 .
Remark 2.13 The conditions given in Theorem 2.40 are only sufficient conditions.
There are functions f for which non of the conditions (i) and (ii) are satisfied at a
point x0 , still f can have local extremum at x0 . For example, consider
f (x) = x4 ,

g(x) = 1 x4 ,

|x| < 1.

Then f 0 (0) = 0 = g 0 (0), f has local minimum at 0 and g has local maximum at 0.
But, f 00 (0) = 0 = g 00 (0).

Remark 2.14 How to identify critical points and extreme points of a function?
1. Suppose f is defined on an open interval I.
(a) Find those points at which either f is not differentiable or f 0 vanish.
These points are the critical points of f .

Differentiability of functions

73

(b) Suppose f 0 (x0 ) = 0.


i. If f 0 (x) has the same sign for x on both side of x0 , then f does not
have an extremum at x0 . Otherwise,
ii. use the test for maximum or minimum as given in Theorem 2.39.
2. Suppose f is continuous on [a, b] and differentiable on (a, b).
(a) f can have maximum or minimum only the at the end points of [a, b] or
at those points in (a, b) at which f 0 vanishes.
(b) Use the tests as in Theorem 2.39 or Theorem 2.40.


2.3.7

Additional exercises

1. Prove that the function f (x) = |x|, x R is not differentiable at 0.


2. Consider a polynomial p(x) = a0 + a1 x2 + . . . + an xn with real coefficients
a1 a2
an
a0 , a1 , . . . , an such that a0 + + + . . . +
= 0. Show that there exists
2
3
n+1
x0 R such that p(x0 ) = 0.
[Note that the conclusion need not hold if the condition imposed on the coefficients is dropped. To see this, consider p(x) = 1 + x2 .]
3. Let I and J be open intervals and f : I J be bijective and differentiable at
every x0 I. If f 0 (x0 ) 6= 0, then show that the inverse function f 1 : J I
is also differentiable at x0 and and (f 1 )0 (x0 ) = 1/f 0 (x0 ).
4. Using Taylors theorem, show that
(1 + x)n = 1 + nx +

n(n 1) 2 n(n 1)(n 2) 3


x +
x + . . . + xn .
2!
3!

5. Show that there exists no differentiable


 function f : [0, 1] R which is differ0, if 0 < x < 1/2,
entiable on (0, 1) such that f 0 (x) =
1, if 1/2 x < 1.
[Hint: Use Example 2.24 in the interval [0, 1/2] and [1/2, 1] taking x0 = 1/2,
and show that the resulting function f is not differentiable at x0 = 1/2.]
6. Suppose f is differentiable on (0, ) and lim f 0 (x) = 0.
x

Prove that lim [f (x + 1) f (x)] = 0.


x

3
Definite Integral

3.1

Introduction

In school you must have come across the definition of integral of a function f :
[a, b] R between the limits a and b, i.e.,
Z b
f (x)dx
a

as the number g(b) g(a), where g : [a, b] R is such that its derivative is f .
One immediate question one raises would be the following:
Given any function f : [a, b] R, does there exist a differentiable function
g : [a, b] R such that g 0 (x) = f (x)?
Obviously, it is not necessary to have such a function g (See the exercise below).

1, 1 x 0,
. Show that there
Exercise 3.1 Consider the function f (x) =
1, 0 x 1
does not exist a differentiable function g : [1, 1] R such that g 0 (x) = f (x) for all
x (0, 1).
Another point one recalls from school is that if g : [a, b] R is a differentiable
function, then g 0 (x) has a geometric meaning, namely, it represents the slope of the
tangent to the graph of g at the point x.
Rb
Do we have a geometric meaning to the integral a f (t)dt?
We answer both the above questions affirmatively for a certain class of functions
by giving a geometric definition of the concept of integral.
Suppose f : [a, b] R is a bounded function. Our attempt is to associate a
number to such a function such that, in case f (x) 0 for x [a, b], then is the
area of the region, say Rf , bounded by the graph of f , x-axis, and the ordinates at
a and b. We may not succeed to do this for all bounded functions f .
Suppose, for a moment, f (x) 0 for all x [a, b]. Let us agree that we have
some idea about what the area under the graph of f . Then it is clear that
m(b a) area(Rf ) M (b a),
74

(3.1.1)

Lower and Upper Sums

75

where m = inf x[a,b] f (x) and M = supx[a,b] f (x). Thus, we get estimates for
area(Rf ). To get better estimates, let us consider a point c such that a < c < b.
Then we have
m1 f (x) M1 x [a, c];

m2 f (x) M2 x [c, b],

where
m1 = inf f (x),
x[a,c]

m2 = inf f (x),
x[c,b]

M1 = sup f (x),

M2 = sup f (x).

x[a,c]

x[c,b]

Then it is obvious that


m1 (c a) + m2 (b c) area(Rf ) M1 (c a) + M2 (b c).

(3.1.2)

Since
m(b a) = m(c a) + m(b c) m1 (c a) + m2 (b c),
M (b a) = M (c a) + M (b c) M1 (c a) + M2 (b c),
we can infer that the estimates in (3.1.2) are better than those in (3.1.1). We can
improve these bounds by taking more and more points in [a, b]. This is the basic
idea of Riemann integration.

3.2

Upper and Lower Sums

Let f : [a, b] R be a bounded function, and let P be a partition of [a, b], i.e., a
finite set P = {x0 , x1 , x2 , . . . , xk } of points in [a, b] such that
a = x0 < x1 < x2 < . . . < xk = b.
We shall denote such partitions by P = {xi }ki=0 also.
Corresponding to the partition P = {xi }ki=0 , and the function f , we associate
two numbers:
L(P, f ) :=

k
X

mi xi ,

U (P, f ) :=

i=1

k
X

Mi xi ,

i=1

where, for i = 1, . . . , k, xi = xi xi1 , and


mi = inf{f (x) : xi1 x xi },

Mi = sup{f (x) : xi1 x xi }.

Definition 3.1 The quantities L(P, f ) and U (P, f ) are called lower sum and
upper sum, respectively, of the function f associated with the partition P .

76

Definite Integral

M.T. Nair

Note that if f (x) 0 for all x [a, b], then L(P, f ) is the total area of the
rectangles with lengths mi and widths xi , and U (P, f ) is the total area of the
rectangle with lengths Mi and widths xi for i = 1, . . . , k. Thus, it is intuitively
clear that the required area, say , under the graph of f must satisfy the relation:
L(P, f ) U (P, f )
for all partitions P of [a, b].
Throughout this chapter, functions defined on [a, b] are considered to be bounded
functions, and P denotes the set of all partitions of [a, b]. Thus we have
L(P, f ) U (P, f )

P P.

NOTATION: If P and Q are partitions of [a, b], then we denote by P Q the


partition obtained by taking all points in P and Q with the condition that common
points would be taken only once.
Exercise 3.2 For P, Q P, let Pe = P Q. Prove that
(i) L(P, f ) L(Pe, f )

and U (Pe, f ) U (Q, f ),

(ii) L(P, f ) U (Q, f ).

We observe that
m(b a) L(P, f ) U (P, f ) M (b a)

P P,

where m = inf{f (x) : a x b} and M = sup{f (x) : a x b}. In view


of this, the set {L(P, f ) : P P} is bounded above by M (b a) and the set
{U (P, f ) : P P} is bounded below by m(b a), where m = inf{f (x) : a x b}
and M = sup{f (x) : a x b}. Hence,
f := sup{L(P, f ) : P P},

f := inf{U (P, f ) : P P}.

exist.
Exercise 3.3 Show that L(P, f ) f f U (Q, f ) for all P, Q P.
Hint: Exercise 3.2.

3.3

Integrability and Integral

Definition 3.2 If there exists a unique such that


L(P, f ) U (P, f )

P P,

then we say that f is Riemann integrable on [a, b], and this is called the
Riemann integral of f , and it is is denoted by
Z b
f (x) dx.
a

Integrability and Integral

77

Remark 3.1 In the due course, Riemann integral will be simply referred to as
integral

The proof of the following theorem is left as an exercise.
Theorem 3.1 A bounded function f : [a, b] R is integrable if and only if f = f .
Definition 3.3 The quantities f and f are known as lower integral and upper
integral, respectively, and they are usually denoted by
Z b
Z b
f (x)dx, respectively.
f (x)dx and
a


Remark 3.2 Not all functions are integrable! For example, consider f : [a, b] R
defined by

0, x Q,
f (x) =
1, x 6 Q.
For this function we have L(P, f ) = 0 and U (P, f ) = b a for any partition P of
[a, b]. Thus, in this case f = 0, f = b a, and hence, f is not integrable.

Theorem 3.2 Suppose f is integrable on [a, b], and m, M are such that
m f (x) M
Then
Z
m(b a)

x [a, b].

f (x) dx M (b a).
a

In particular, if M0 > 0 is such that |f (x)| M0 for all x [a, b], then
Z b




f (x) dx M0 (b a).

a

Proof. We know that for any partition P on [a, b],


Z b
m(b a) L(P, f )
f (x) dx U (P, f ) M (b a).
a

Hence the result.


Definition 3.4 Suppose f : [a, b] R is integrable. Then we define
Z a
Z b
f (x) dx :=
f (x) dx.
b

Also, for any function function f : [a, b] R, we define


Z
f (x) dx := 0
[a, b].

78

Definite Integral

M.T. Nair

EXAMPLE 3.1 Let f (x) = c for all x [a, b], for some c R. Then we have
L(P, f ) = c(b a),

U (P, f ) = c(b a)

Hence
c(b a) = L(P, f ) f f U (P, f ) = c(b a).
Rb
Thus, f is integrable and a f (x)dx = c(b b).

EXAMPLE 3.2 Let f (x) = x for all x [a, b]. Let


xi = a + i

(b a)
,
n

i = 0, 1, . . . , n.

Then Pn = {xi }ni=0 is a partition of [a, b]. In this case we have


mi = xi1 ,

Mi = xi ,

xi =

ba
n

for i = 1, . . . , n.

Hence
n
X

n 
X


(b a) (b a)
,
L(Pn , f ) =
xi1 xi =
a + (i 1)
n
n
i=1
i=1

n
n 
X
X
(b a) (b a)
U (Pn , f ) =
xi xi =
a+i
.
n
n
i=1

i=1

It is see that
(b a)2
n(n + 1)
+ (b a)2
,
n
2n2
(b a)2 n(n + 1)
U (Pn , f ) = a(b a) +
n2
2
L(Pn , f ) = a(b a)

as n . Thus, both {L(Pn , f )} and {U (Pn , f )} converge to the same limit


(b2 a2 )/2. Now, since
L(Pn , f ) f f U (Pn , f ) n N,
Rb
the function f , is integrable and a x dx = (b2 a2 )/2.

3.3.1

Some necessary and sufficient conditions for integrability

In Example 3.2, what we have showed is that for a sequence (Pn ) of partitions,
the sequences {U (Pn , f )} and {L(Pn , f )} converge to the same point. We shall see
(Corollary 3.4 below) that this is true in general.
Theorem 3.3 A bounded function f : [a, b] R is Riemann integrable if and only
if for every > 0, there exists a partition P of [a, b] such that
U (P, f ) L(P, f ) < .

Integrability and Integral

79

Proof. Suppose f is integrable and let be its integral. Then we have f = f .


Let > 0 be given. By the definition of f and f , there exists partitions P 0 and
P 00 of [a, b] such that
f < L(P 0 , f ) < f + ,

f < U (P 00 , f ) < f + .

Let P = P 0 P 00 . Then we have


f < L(P 0 , f ) L(P, f ) U (P, f ) U (P 00 , f ) < f + .
Since f = = f , we have
< L(P, f ) U (P, f ) < + .
This proves that U (P, f ) L(P, f ) < .
Conversely, suppose for every > 0, there exists a partition P of [a, b] such
that U (P , f ) L(P , f ) < . Since L(P , f ) f f U (P , f ) we obtain that
f f < for every > 0. This shows that f = f . Thus, f is integrable.
Corollary 3.4 A bounded function f : [a, b] R is Riemann integrable if and only
if there exists a sequence (Pn ) of partitions of [a, b] such that
U (Pn , f ) L(Pn , f ) 0

as

and in that case


Z
L(Pn , f )

Z
f (x) dx

and

U (Pn , f )

f (x) dx
a

as n .
Proof. The proof follows from Theorem 3.3. However, we give details:
Suppose f is integrable. Then, by Theorem 3.3, for each n N, there exists a
partition Pn of [a, b] such that
U (Pn , f ) L(Pn , f ) <

1
.
n

Thus, U (Pn , f ) L(Pn , f ) 0 as n .


Conversely, suppose there exists a sequence (Pn ) of partitions of [a, b] such that
U (Pn , f ) L(Pn , f ) 0 as n . Since
L(Pn , f ) f f U (Pn , f )

n N,

we obtain that f = f . Thus, f is integrable.


We use Corollary 3.4 to prove the following theorem.

80

Definite Integral

M.T. Nair

Theorem 3.5 Let f and g be integrable over [a, b]. Then f + g is integrable and
Z

f (x) dx.

f (x) dx +

[f (x) + g(x)]dx =

Proof. By Corollary 3.4, there exists sequences of partitions (Pn0 ) and (Pn00 ) of
[a, b] such that
U (Pn0 , f ) L(Pn0 , f ) 0,

U (Pn00 , g) L(Pn00 , g) 0

as n . For each n N, let Pn = Pn0 Pn00 . Then we have


L(Pn0 , f )

Z
L(Pn , f )

f (x)dx U (Pn , f ) U (Pn0 , f ),

L(Pn00 , f ) L(Pn , f )

g(x)dx U (Pn , f ) U (Pn00 , g).

Hence,
U (Pn , f ) L(Pn , f ) 0,

U (Pn , g) L(Pn , g) 0

()

as n . We note that for every y [c, d] [a, b],


inf f (x) + inf g(x) f (y) + g(y) sup f (x) + sup g(x).

cxd

cxd

cxd

cxd

Hence,
L(Pn , f ) + L(Pn , g) L(Pn , f + g) U (Pn , f + g) U (Pn , f ) + U (Pn , g).
Hence, from (),
U (Pn , f + g) L(Pn , f + g) 0

as

n .

Thus, by Corollary 3.4, f + g is integrable on [a, b]. Writing


an := L(Pn , f ) + L(Pn , g),

bn := U (Pn , f ) + U (Pn , g),

() implies
Z
an

g(x)dx bn ,

f (x)dx +
a

Z
an

[f (x) + g(x)]dx bn .
a

Note that bn an 0 as n . Thus, we obtain


Z b
Z b
Z b
[f (x) + g(x)]dx =
f (x) dx +
f (x) dx.
a

This completes the proof.

()

Integrability and Integral

81

Theorem 3.6 Suppose f is integrable on [a, c] and [c, b]. Then f is integrable on
[a, b], and
Z b
Z c
Z b
f (x) dx.
f (x) dx +
f (x) dx =
c

Proof. Let f1 = f |[a,c] , f2 = f |[c,b] . Let > 0 be given. Since f1 and f2 are
integrable, there exist partitions P1 and P2 of [a, c] and [c, b] respectively such that

U (P1 , f1 ) L(P1 , f1 ) < ,


2

U (P2 , f2 ) L(P2 , f2 ) < .


2

Suppose P = P1 P2 . Then, it can be seen that


L(P, f ) = L(P1 , f1 ) + L(P2 , f2 ),

U (P, f ) = U (P1 , f1 ) + U (P2 , f2 ).

Hence,
U (P, f ) L(P, f ) = [U (P1 , f1 ) L(P1 , f1 )] + [U (P2 , f2 ) L(P2 , f2 )] < .
Thus f is integrable. Since
Z
L(P, f )

f (x)dx U (P, f ),
a

Z
L(P1 , f1 ) + L(P2 , f2 )

f (x)dx U (P1 , f1 ) + U (P2 , f2 ),

f (x)dx +
a

it follows that

Z
c

Z c

Z b
Z b



f (x) dx +
f (x) dx
f (x) dx < .

a

This is true for all > 0. Hence the final result.


Theorem 3.7 If f is integrable on [a, b] such that f (x) 0 for all x [a, b], then
Rb
a f (x) dx 0. More generally, suppose f and g are integrable on [a, b] such that
f (x) g(x) for all x [a, b]. Then
Z

Z
f (x) dx

g(x) dx.
a

Rb
Proof. Since L(P, f ) a f (x)dx for every partition P on [a, b], and since
Rb
L(P, f ) 0 by the the assumption on f , we obtain a f (x)dx 0. The general
case follows from the above by applying the above result for the function defined
by (x) = g(x) f (x), x [a, b].
Corollary 3.8 Suppose f is integrable on [a, b]. Then
Z b
Z b



f (x)dx
|f (x)|dx.

a

82

Definite Integral

M.T. Nair

Exercise 3.4 Prove Corollary 3.8.


Definition 3.5 Let f : [a, b] R be a function, P = {xi : i = 0, 1, . . . , k} be a
partition of [a, b], and T = {ti : i = 1, . . . , k} be such that xi1 ti xi for all
i = 1, . . . , k. Such a set T is called a set of tags on P or simply a tag. The number
S(P, f, T ) :=

k
X

f (ti )xi

i=1

is called the Riemann sum for f corresponding to the partition P and the set T
of tags on P .
We may observe that for any partition P and for any set T of tags on P ,
L(P, f ) S(P, f, T ) U (P, f ).
This observation together with Corollary 3.4 gives the following result.
Theorem 3.9 If (Pn ) is a sequence of partitions of [a, b] such that
U (Pn , f ) L(Pn , f ) 0

as

n ,

then f is integrable, and


Z
S(Pn , f, Tn )

f (x) dx,
a

where Tn is any set of tags on Pn , n N.


We have the following characterization.
Theorem 3.10 A bounded function f : [a, b] R is Riemann integrable if and
only if there exists such that for every > 0, there exists a partition P of [a, b]
satisfying
|S(P, f, T ) | < for every tag T on P.
Proof. Let > 0 be given.
Suppose f is Riemann integrable. Then, by Theorem 3.3, there exists a partition
P of [a, b] such that U (P, f ) L(P, f ) < . Since L(P, f ) S(P, f, T ) U (P, f ) for
every tag T on P , we obtain
|S(P, f, T ) | < for every tag T on P.
Conversely, let P be a partition of [a, b] be such that
< S(P, f, T ) < + for every tag T on P.
Let
Tn0 = {t0i,n : i = 1, . . . , k},

Tn00 = {t00i,n : i = 1, . . . , k}

()

Integral of Continuous Functions

83

be tags on P : a = x0 < x1 < . . . < xk = b such that


t0i,n mi ,

t00i,n Mi

as

n .

Then
S(P, f, Tn0 ) L(P, f ),

S(P, f, Tn00 ) U (P, f )

as

n .

Also, from (), we have


< S(P, f, Tn0 ) < + ,

< S(P, f, Tn00 ) < +

n N.

Now, taking limit, we have


< L(P, f ) < + ,

< U (P, f ) < + .

Therefore, U (P, f ) L(P, f ) < . By Theorem 3.3, f is integrable.

3.4

Integral of Continuous Functions

Definition 3.6 Let P = {xi : i = 0, 1, . . . , k} be a partition of [a, b]. Then the


quantity max{xi xi1 : i = 1, . . . , n} is called the mesh of the partition P and it
is denoted by (P ), i,e.,
(P ) := max{xi xi1 : i = 1, . . . , k}.

Theorem 3.11 Suppose f : [a, b] R is a continuous function. Then f is integrable. In fact, we have the following:
(i) For every > 0 there exists a > 0 such that for every partition P of [a, b]
with (P ) < , we have
U (P, f ) L(P, f ) < .
(ii) Suppose (Pn ) is a sequence of partitions of [a, b] such that (Pn ) 0 as
n . Then the sequences {U (Pn , f )}, {L(Pn , f )} {S(Pn , f, Tn )} converge
Rb
to the same limit a f (x) dx, where Tn is any set of tags on Pn for each n N.
The main property of a continuous function f : [a, b] R that is required to
prove the first part is its uniform continuity.
Definition 3.7 A real valued function f defined on an interval I is said to be
uniformly continuous on I if for every > 0, there exists > 0 such that
x, y I,

|x y| < = |f (x) f (y)| < .




84

Definite Integral

M.T. Nair

Clearly, every uniformly continuous function is continuous. But, the converse is


not true. To see this, consider the function
f (x) =

1
,
x

0 < x < 1.

Clearly, f is continuous on I := (0, 1). Now, let > 0 be given. For x, y (0, 1),
|f (x) f (y)| <

|x y|
< |x y| < xy.
xy

Thus, for f to be uniformly continuous, there must exist a > 0 such that
|xy|, which is not possible. In particular, we are not in a position to choose a
independent of the points x, y.
To see this fact, more clearly, consider xn = 1/n and yn = 1/(n + 1). Then we
know that |xn yn | 0 as n and |f (xn ) f (yn )| = 1 for all n N. Hence,
the condition in the definition of uniform continuity is not satisfied if we take < 1.
The above kind of situation will not arise if the interval I is closed and bounded.
More specifically, we have the following theorem.
Theorem 3.12 Every real valued continuous function defined on a closed and
bounded interval is uniformly continuous.
Proof. Suppose f : [a, b] R is continuous. We have to show that for every
> 0, there exists > 0 such that
|x y| < = |f (x) f (y)| < .
Suppose this is not true. Then there exists 0 > 0 such that for every > 0, there
are x, y such that
|x y| < but |f (x) f (y)| 0 .
()
Taking = 1/n, there exists xn , yn such that
|xn yn | < 1/n

but |f (xn ) f (yn )| 0 .

Since (xn ) is a bounded sequence, it has a convergent subsequence, say xkn c


for some c R. Since |xkn ykn | 0, we also have the convergence ykn c.
Now, by the continuity of f , we have f (xkn ) f (c) and f (ykn ) f (c). Thus,
|f (xkn ) f (ykn )| 0. This is a contradiction to ().
Proof of Theorem 3.11. Let f : [a, b] R be a continuous function, and let
> 0 be given. Let P : a = x0 < x1 < x2 . . . < xk = b be a partition of [a, b]. Then
U (P, f ) L(P, f ) =

k
X
i=1

(Mi mi )(xi xi1 ).

Integral of Continuous Functions

85

Since f is continuous on the closed interval [a, b], there exists i , i in [xi1 , xi ] such
that Mi = f (i ), mi = f (i ) for i = 1, . . . , k. Hence,
k
X

U (P, f ) L(P, f ) =

[f (i ) f (i )](xi xi1 ).

i=1

Again, since f is uniformly continuous on [a, b], there exists > 0 such that
|f (t) f (s)| < /(b a)

whenever |t s| < .

Hence, if we take P such that (P ) < , then we have


U (P, f ) L(P, f ) =

k
X
[f (i ) f (i )](xi xi1 ) < .
i=1

Therefore, by Theorem 3.3, f is integrable, and the proof of (i) is also over.
Next, suppose (Pn ) is a sequence of partitions such that (Pn ) 0 as n .
Let N N be such that (Pn ) < for all n N . Then, it follows by (i) above that
U (Pn , f ) L(Pn , f ) < for all n N , showing that U (Pn , f ) L(Pn , f ) 0 as
n . Now, the concludions in (ii) follows from the observations
Z
L(Pn , f )

f (x)dx U (Pn , f ),
a

L(Pn , f ) S(Pn , f, Tn ) U (Pn , f )


for all n N.
Exercise 3.5 Suppose f : [a, b] R is a continuous function. Show that for every
> 0 there exists a > 0 such that for every partition P of [a, b] with (P ) < , we
have
Rb
(a) a f (x)dx L(P, f ) <
Rb
(b) U (P, f ) a f (x)dx <
Rb
(c) |S(P, f, T ) a f (x)dx| < for any tag T on P .
Remark 3.3 The property (c) in the above exercise is written as
Z
lim S(P, f, T )
(P )0

f (x)dx.
a

86

Definite Integral

M.T. Nair

EXAMPLE 3.3 Let f (x) = ex for all x [a, b]. Then, f is continuous. Let
(b a)
,
n

hn =

xi = a + ihn ,

ti = xi1 ,

i = 1, . . . , n.

Then with Pn = {xi }ni=1 and T = {ti }ni=1 , we have (Pn ) 0, and
S(Pn , f, Tn ) = hn

n
X

ea+(i1)hn = hn ea

i=1

n
X

n(i1) = hn ea

i=1

nn 1
,
n 1

where n = ehn . Since nn = eba , we have


S(Pn , f, Tn ) = hn ea
Since limn

ehn 1
hn

nn 1
hn
hn
= [eb ea ] hn
.
= ea [eba 1] hn
n 1
e 1
e 1

= 1, we have S(Pn , f, Tn ) = ea [eba 1] = eb ea .

Remark 3.4 It can be also shown that if a bounded function f : [a, b] R is


piecewise continuous, i.e., there are at most a finite number of points in [a, b] at
which f is discontinuous, then f is integrable. For a proof of this, see Theorem 3.21
in the appendix (Section 3.7).


3.5

Some Properties

For continuous functions f and g, we can give a simpler proof for Theorem 3.5, as
in the following.
Theorem 3.13 Suppose f and g are continuous functions on [a, b]. Then
b

Z
a

[f (x) + g(x)] dx =
f (x) dx +
g(x) dx
a
a
Z b
Z b
c f (x) dx = c
f (x) dx c R.
a

Proof. Suppose (Pn ) is a sequence of partitions on [a, b] such that (Pn ) 0 as


n . For each n N, let Tn be a set of tags on Pn . Since f , g are continuous,
the functions f + g and c f are also continuous for every c R, and
Z b
Z b
S(Pn , f, Tn )
f (x) dx,
S(Pn , g, Tn )
g(x) dx,
a

Z
S(Pn , f + g, Tn )

[f (x) + g(x)] dx.


a

Now, since
S(Pn , f + g, Tn ) = S(Pn , f, Tn ) + S(Pn , g, Tn ),
S(Pn , c f, Tn ) = c S(Pn , f, Tn )

c R,

Some Properties

87

it follows that
Z

a
b

g(x) dx,

f (x) dx +

[f (x) + g(x)] dx =
Z

f (x) dx

c f (x) dx = c

c R.

This completes the proof.


Theorem 3.14 (Mean-value theorem) Suppose f is continuous on [a, b]. Then
there exists [a, b] such that
Z b
1
f (x) dx = f ().
ba a
Proof. Since f is continuous, we know that there exist u, v [a, b] such that
f (u) = m := min f (x) and f (v) = M := max f (x). Hence, by Theorem 3.2,
Z b
1
f (u)
f (x) dx f (v).
ba a
Hence, by intermediate value theorem, there exists [a, b] such that
Z b
1
f (x) dx = f ().
ba a
Hence the result.
Theorem 3.15 (Generalized mean-value theorem) Suppose f and g are continuous on [a, b] where g(x0 ) 6= 0 for some x0 [a, b] and g(x) 0 for all x [a, b].
Then there exists [a, b] such that
Z b
Z b
f (x)g(x) dx = f ()
g(x)dx.
a

Proof. Let m := inf f (x) and M = sup f (x). Then we have


Z b
Z b
Z b
m
g(x)dx
f (x)g(x) dx M
g(x)dx.
a

Since g(x0 ) 6= 0 for some x0 [a, b] and g(x) 0 for all x [a, b], it follows that
Rb
a g(x)dx > 0. Hence,
Rb
f (x)g(x) dx
m aRb
M.
a g(x)dx
Therefore, by the intermediate value theorem, there exists [a, b] such that
Rb
f (x)g(x) dx
f () = a R b
.
g(x)dx
a
Hence the result.

88

Definite Integral

3.6

Some Results

3.6.1

M.T. Nair

First fundamental theorem

Theorem 3.16 Suppose f is continuous on [a, b], and for x [a, b], let
Z x
f (t)dt.
(x) =
a

Then is differentiable and 0 (x) = f (x)

x (a, b).

Proof. Let x (a, b) and h > 0 be such that x + h [a, b]. Then we have
x+h

Z
(x + h) (x) =

Z
f (t)dt

Z
f (t)dt =

x+h

f (t)dt.
x

By mean-value theorem, there exists h in the interval with endpoints x, x + h such


that
Z
1 x+h
f (t)dt = f (h ).
h x
Since f is continuous at x, we have f (h ) f (x) as h 0. Hence
lim

(x + h) (x)
= f (x).
h

lim

(x + h) (x)
= f (x).
h

h0+

Similarly, we have
h0

Thus, 0 (x) exists and 0 (x) = f (x).


Definition 3.8 The function in Theorem 3.16 is called the indefinite integral.


3.6.2

Second fundamental theorem

Definition 3.9 A function g is called an anti-derivative or primitive of f if g


is differentiable and g 0 (x) = f (x) for all x [a, b].

By the
R x Theorem 3.16, if f is a continuous function, then the indefinite integral
(x) = a f (t)dt is an anti-derivative of f .
We may observe that if g1 and g2 are anti-derivatives of f , then g10 (x) = f (x) =
for all x [a, b], i.e., g10 (x) g20 (x) = 0 for all x [a, b]. Hence, it follows that
g1 g2 is a constant. Thus, in view of the above theorem it follows that if g is an
anti-derivative of f , and if is the indefinite integral of f , then g(x) = (x) + c for
g20 (x)

Some Results

89

all x [a, b] and for some constant c R. Hence, in view of the above theorem, we
have
Z b
f (t)dt.
g(b) g(a) = (b) (a) =
a

Thus we have proved the following theorem.


Theorem 3.17 (Second fundamental theorem) Suppose f is continuous on
[a, b] and suppose that g is an anti-derivative of f . Then
Z b
f (t)dt = g(b) g(a).
a

The conclusion of the above theorem is also known as Newton-Leibniz formula.


The difference g(b) g(a) is usually written as [g(x)]ba .
Here is another proof for the above theorem.
An alternate Proof. Let g be an antiderivative of f , i.e., g is differentiable and
g 0 = f . Let P : a = x0 < x1 < . . . < xn = b be any partition of [a, b]. Then by
Lagranges mean value theorem, there exists i [xi1 , xi ] such that
g(xi ) g(xi1 ) = g 0 (i )(xi xi1 ) = f (i )(xi xi1 ).
Hence,

n
n
X
X
g(b) g(a) =
[g(xi ) g(xi1 )] =
f (i )(xi xi1 ).
j=1

j=1

Pn

Since the Riemann sum j=1 f (i )(xi xi1 ) is constant for any partition P , it
Rb
P
follows that nj=1 f (i )(xi xi1 ) = a f (x)dx. Thus,
Z

f (x)dx = g(b) g(a).


a

This completes the proof.


Remark 3.5 For giving the alternate proof above, we have not used the continuity
of f , but the fact that f is an anti-derivative, i.e., derivative of a function g. It is
known that, a function can be an anti-derivative without being continuous.

The following theorem shows that integral of a continuous function is a Riemann
sum.
Theorem 3.18 Suppose f is a continuous function.Then for every partition P of
[a, b], there exists a set T of tags on P such that
Z b
S(P, f, T ) =
f (x) dx.
a

90

Definite Integral

M.T. Nair

Proof. Let P = {xi : i = 1, . . . , k} be a partition of [a, b]. Since f is continuous,


by mean value theorem (Theorem 3.14), there exists i [xi1 , xi ] such that
Z xi
f (x) dx = f (i )(xi xi1 ),
i = 1, . . . , k.
xi1

Hence, taking T = {i : i = 1, . . . , k},


S(P, f, T ) =

k
X

f (i )(xi xi1 ) =

i=1

k Z
X
i=1

xi

f (x) dx.

f (x) dx =

xi1

This completes the proof.

3.6.3

Applications of fundamental theorem

Theorem 3.19 (Product formula) Suppose f and g are continuous functions on


[a, b], and let G be an anti-derivative of g. If f is differentiable on [a, b], then
b

Z
a

f (x)g(x) dx = [f (x)G(x)]ba

f 0 (x)G(x) dx.

Proof. Recall that if u and v are differentiable, then


(uv)0 = u0 v + uv 0 .
Hence,
Z

[u(x)v(x)]0 dx =

u0 (x)v(x) dx +

u(x)v 0 (x) dx.

Using fundamental theorem,


[u(x)v(x)]ba

Z
=

u (x)v(x) dx +
a

u(x)v 0 (x) dx.

Thus,
Z

u(x)v 0 (x) dx = [u(x)v(x)]ba

u0 (x)v(x) dx.

Now, taking f (x) = u(x) and v(x) = G(x), we obtain the required formula.
Theorem 3.20 (Change of variable formula) Suppose : [, ] R is a
differentiable function such that () = a and () = b. Then, for any continuous
function f : [a, b] R,
Z

f (x) dx =
a

f ((t)) 0 (t)dt.

Some Results

91

Proof. Let F be an anti-derivative of f , i.e., such that F 0 (x)f (x). Then taking
G(t) = F ((t)) for t [, ], we have
G0 (t) = F 0 ((t)) 0 (t) = f ((t)) 0 (t),

t [, ].

Hence, by fundamental theorem,


Z

f ((t)) (t)dt =

G0 (t)dt = G() G() = F (()) F (()).

Hence,
Z

f ((t)) (t)dt = F (b) F (a) =

f (x) dx.
a

This completes the proof.


The following examples have been worked out by knowing the antiderivatives of
certain functions.
EXAMPLE 3.4 For k 0,
b

xk+1
x dx =
k+1


b
=
a

bk+1 ak+1
.
k+1


EXAMPLE 3.5 For 6= 0,


Z

e
a

ex
dx =

b
=
a

eb ea
.
k+1

92

Definite Integral

3.7

Appendix

M.T. Nair

Theorem 3.21 Suppose f : [a, b] R is bounded and piecewise continuous,i.e.,


there are at most a finite number of points in [a, b] at which f is discontinuous.
Then f is integrable.
Proof. Let > 0 be given. We have to show that there exists a partition P such
that U (P, f ) L(P, f ) < .
Suppose that c (a, b) such that f is continuous on [a, c) and (c, b]. Let > 0 be
such that c + < b and c > a. Let f1 , f2 , f3 be restrictions of f to the intervals
[a, c ], [c + , b] and [c , c + ], respectively. Since f is continuous on [a, c ]
and [c + , b], f is integrable on these intervals, so that there exist partitions P1 on
[a, c ] and P2 on [c + , b] such that

U (P1 , f1 ) L(P1 , f1 ) < ,


3

U (P2 , f2 ) L(P2 , f2 ) < .


3

Assume that > 0 is so small that (M m) < /6. Then for any partition P3 on
[c , c + ], we have

U (P3 , f3 ) L(P3 , f3 ) < .


3
Now, for the partition P = P1 P2 P3 on [a, b], we have
U (P, f ) = U (P1 , f1 ) + U (P2 , f2 ) + U (P3 , f3 ),
L(P, f ) = L(P1 , f1 ) + L(P2 , f2 ) + L(P3 , f3 ).
Hence,
U (P, f ) L(P, f ) < .
Thus, f is integrable. The case of more than one (but finite number of) points of
discontinuity can be handled analogously.

4
Improper Integrals

Recall that we defined definite integral of a function f for the case when f is a
bounded function defined on a closed interval [a, b]. In case the assumptions on f
are not satisfied, then can we still have a notion of integral? We discuss a few such
cases.

4.1

Definitions

Rt
Definition 4.1 Suppose f is defined on [a, ). If (t) := a f (x) dx exists for
every t > a, and if limt (t) exists, then we define the improper integral of f
over [a, ) as
Z t
Z
f (x) dx = lim
f (x) dx.
t a

Rb
Definition 4.2. Suppose f is defined on (, b]. If (t) := t f (x) dx exists for
every t < b, and if limt (t) exists, then we define the improper integral of
f over (, b] as
Z b
Z b
f (x) dx = lim
f (x) dx.
t t

Rc
Definition 4.3. Suppose f is defined on R := (, ). If f (x) dx and
R
c f (x) dx exists for some c R, then we define the improper integral of f
over (, ) as
Z
Z c
Z
f (x) dx =
f (x) dx +
f (x) dx.

Rt
We may observe that existence of limt t f (x) dx does not, in general, imply
R
existence of f (x) dx. To see this, consider the following example:
Rt
Let f (x) = x for every x R. Then we have t f (x) dx = 0 for every t R,
Rc
R
but the integrals f (x) dx and c f (x) dx do not exist.
93

94

Improper Integrals

M.T. Nair

Next we consider the case when f is defined on an interval J of finite length, but
limxx0 |f (x)| = , where x0 either belongs to J or it is an end point of J.
Rb
Definition 4.4. Suppose f is defined on (a, b]. If t f (x) dx exists for every
Rb
t (a, b), and if lim0 a+ f (x) dx exists, then we define the improper integral
of f over (a, b] as
Z b
Z b
f (x) dx.
f (x) dx = lim
0 a+

Rt
Definition 4.5. Suppose f is defined on [a, b). If a f (x) dx exists for every
R b
t (a, b), and if lim0 a f (x) dx exists, then we define the improper integral
of f over [a, b) as
Z b
Z b
f (x) dx = lim
f (x) dx.
0 a

Rc
Definition 4.6. Suppose f is defined on [a, c) and (c, b]. If a f (x) dx and
Rb
c f (x) dx exist, then we define the improper integral of f over [a, b] as
Z c
Z b
Z b
f (x) dx =
f (x) dx +
f (x) dx.
a

Now we combine the situations in Definitions 4.1, 4.2 with Definitions 4.4, 4.5,
to consider improper integrals over intervals of the form (a, ) and (, b).

Rt
Definition
4.7.
Suppose
f
is
defined
on
(a,
).
If
the
integrals
a f (x) dx and
R
f
(x)
dx
exist
as
improper
integrals
for
every
t
>
a,
then
we
define
the
improper
t
integral of f over (a, ) as
Z
Z t
Z
f (x) dx =
f (x) dx +
f (x) dx.
a

Rt
Definition. Suppose f is defined on (, b). If the integrals f (x) dx and
Rb
t f (x) dx exist as improper integrals for every t < b, then we define the improper
integral of f over (, b) as
Z b
Z t
Z b
f (x) dx =
f (x) dx +
f (x) dx.

In case an improper integral exists (resp. does not exists), then we also say that
the improper integral converges (resp. diverges).

Definitions

4.1.1

Examples
Z

1
dx. Note that
x

EXAMPLE 4.1 Consider the improper integral


1
t

1
dx = [ln x]t1 = ln t as t .
x

Hence,

R
1

1
x

dx diverges.


Z

1
dx Note that
x2

EXAMPLE 4.2 Consider the improper integral


1
t

Z
1

Hence,

95

R
1

1
x2

1
1
dx =
2
x
x

t
=1
1

1
1
t

as

t .

dx converges.


Z

EXAMPLE 4.3 For p 6= 1, consider the improper integral


1

case, we have
t

Z
1

1
dx. In this
xp

 p+1 t
1
x
tp+1 1
dx
=
=
.
xp
p + 1 1
p + 1

Note that,
p>1

1
tp+1 1

p + 1
p1

and
p<1

as

tp+1 1
as
p + 1

t ,

t ,

Hence,

Z
1

1
dx
xp

converges for p > 1,


diverges for p 1.

Z

EXAMPLE 4.4 For p 6= 1, consider the improper integral


0

we have
Z

1
xp+1
dx
=
xp
p + 1


1
=

1
dx. In this case,
xp

1 p+1
.
p + 1

Note that,
p>1

and
p<1

p+1 1
as
p + 1
p+1 1
1

p + 1
1p

0,

as 0,

96

Improper Integrals

M.T. Nair

Hence,
Z

1
dx
xp

converges for p < 1,


diverges for p 1.


EXAMPLE 4.5 Let a < b and < 1. Then


We observe that for a < t < b,
Z t
a

dx
=
(b x)

Rb

dx
a (bx)

ba

bt

converges:

du
.
u

Now,
Z

lim

tb a

dx
exists
(b x)

ba

lim

du

tb bt u
Z ba

lim

exists

du
exists
u

< 1.

Rb

Exercise 4.1 Suppose f 0 and the integral a f (x)dx exists for every x [a, b)
Rb
and limxb (b x) f (x) exists for some < 1. Then a f (x)dx exists.
[Hint: Observe that for any > 0, there exists x0 [a, b) such that the number
+
:= limxb (b x) f (x) satisfies 0 f (x) (bx)
for all x [x0 , b).]

4.2

Integrability by Comparison

We state a result which will be useful in asserting the existence of certain improper
integral by comparing it with certain other improper integral.
Suppose J is either an interval of finite or infinite length. Suppose f defined on
J, except possibly at one point (It can be generalized to the case when f is not be
defined at some finite number of point in J). We denote the improper integral of f
over J by
Z
f (x) dx,
J

and say the the improper integral over


R J converges whenever it exists, and otherwise,
we say that the improper integral J f (x) dx diverges.
For example, if J = [a, b], then f may not be defined at a or at b or at some
point c (a, b), and the corresponding improper integrals, by definitions are
Z
lim

ta t

Z
f (x) dx,

lim

tb a

Z
f (x) dx,

lim

tc

Z
f (x) dx + lim

tc+

f (x) dx.
t

Integrability by Comparison

97

Theorem 4.1 Suppose J is above, and f and g are defined on J.


(i) If 0 f (x) g(x) for all x J, and
exists.
(ii) If

4.2.1

R
J

|f (x)| dx exists, then

R
J

J g(x) dx exists, then

R
J

f (x) dx

f (x) dx exists.

Examples

EXAMPLE 4.6 Since




sin x
1


xp xp ,

cos x
1


p p
x
x

it follows from Example 4.3 and Theorem ??(ii) that the improper integrals
Z
Z
sin x
cos x
dx
and
dx
p
x
xp
1
1
converge for all p > 1.
R x
R
In fact 1 sin
xp dx and 1
example.


cos x
xp

dx converge for all p > 0 as we see in the next

EXAMPLE 4.7 Let p > 0. Then for t > 0,



t
Z t
Z t
1
1
sin x
dx =
( cos x) p
cos x dx
p
p
p+1
x
x
1 x
1
1


Z t
cos t
cos x
= cos 1 p p
dx.
p+1
t
1 x
By the result in Example 4.6,
Z
cos x
dx converges for all p > 0.
xp+1
1
R sin x
t
Also, cos
tp R 0 as t . Hence, 1
xp dx converges for all p > 0. Similarly, we

x
see that 1 cos
dx
converges
for
all
p
> 0.

xp
EXAMPLE 4.8 Since



sin x sin x 1
1

=

xp x xp1 xp1 ,

cos x
1


p p
x
x

it follows from Example 4.4 above and Theorem 4.1(ii)that


Z 1
sin x
dx converges for all p < 2,
xp
0
Z 1
cos x
dx converges for all p < 1.
xp
0


98

Improper Integrals

M.T. Nair

EXAMPLE 4.9 Observe that


sin x
sin x 1
sin 1
=
p1
p
p1
x
x x
x
Since

R1

1
0 xp1

x (0, 1].

dx diverges for p 1 1, i.e., for p 2, it follows that


Z 1
sin x
dx diverges for all p 2,
xp
0


EXAMPLE 4.10 From Examples 4.8, 4.9, 4.7,


Z
sin x
dx converges for
xp
0

0 < p < 2.


4.2.2

Gamma and Beta Functions

Gamma and Beta Functions are certain improper integrals which appear in many
applications.
Gamma function
We show that for x > 0, the improper integral
Z
(x) :=
tx1 et dt
0

converges. The function (x), x > 0, is called the gamma function.


R1
Note that for tx1 et tx1 for all t > 0, and 0 tx1 dt converges for x > 0.
Hence, by Theorem 4.1,
Z 1
tx1 et dt converges for x > 0.
0

tx1 et

R
Also, we observe that
0 as t , and 1 t2 dt converges. Hence, by
2
t
R
Theorem 4.2, 1 tx1 et dt converges. Thus,
Z
Z 1
Z
x1 t
x1 t
(x) :=
t e dt =
t e dt +
tx1 et dt
0

converges for every x > 0.


Beta function
We show that for x > 0, y > 0, the improper integral
Z 1
(x, y) :=
tx1 (1 t)y1 dt
0

Integrability Using Limits

99

converges. The function (x, y) for x > 0, y > 0 is called the beta function.
Clearly, the above integral is proper for x 1, y 1. Hence it is enough to
consider the case of 0 < x < 1, 0 < y < 1. In this case both the points t = 0 and
t = 1 are problematic. hence, we consider the integrals
Z 1
Z 1/2
x1
y1
tx1 (1 t)y1 dt.
t (1 t)
dt,
1/2

We note that if 0 < t 1/2, then (1t)y1 21y so that tx1 (1t)y1 21y tx1 .
R 1/2
R 1/2
Since 0 tx1 dt converges it follows that 0 tx1 (1 t)y1 dt converges. To deal
with the second integral, consider the change of variable u = 1 t. Then
Z 1/2
Z 1
x1
y1
uy1 (1 u)x1 du
t (1 t)
dt =
0

1/2

which converges by the above argument. Hence,


Z 1
(x, y) :=
tx1 (1 t)1y dt,

x > 0, y > 0

converges for every x > 0, y > 0.

4.3

Integrability Using Limits

Now some more results which facilitate the assertion of convergence/divergence of


improper integrals, whose proofs follow from the definition of limits.
Rb
Theorem 4.2 Suppose f (x) 0, g(x) 0 for all x [a, ), a f (x)dx and
Rb
f (x)
a g(x)dx exists for every b > a. Suppose further that g(x) ` as x .
R
R
(i) If ` 6= 0, then a f (x)dx converges a g(x)dx converges.
R
R
(ii) If ` = 0, then a g(x)dx converges a f (x)dx converges.
Proof. (i) Suppose ` 6= 0. Then ` > 0, and for > 0 with ` > 0, there exists
x0 a such that
f (x)
`<
< ` + x x0 .
g(x)
Hence
(` )g(x) < f (x) < (` + )g(x) x x0 .
R
Rx
Consequently, x0 f (x)dx converges iff x0 g(x)dx converges. As a 0 f (x)dx and
R x0
a g(x)dx exist, the result in (i) follows.
R

(ii) Suppose ` = 0. Then for > 0, there exists x0 a such that


f (x)
< x x0 .
g(x)

100

Improper Integrals

M.T. Nair

R
Thus, f (x) < g(x) for all x x0 . Hence, convergence of x0 g(x)dx implies the
R
convergence of x0 f (x)dx. From this the result in (ii) follows.

4.4

Additional Exercises
Z


sin

1. Does
1

1
x2


dx converge?



[ Hint: Note that sin x12 x12 .]
Z
cos x
dx converge?
2. Does
x(log x)2
2


cos x
[Hint: Observe x(log
x(log1 x)2 and use the change of variable t = log x.]
x)2
Z

3. Does
0

sin2 x
dx converge?
x2
2

[Hint: Observe sinx2 x x12 for x 1 and


extension on [0, 1].]
Z 1
sin x
4. Does
dx converge?
x2
0


x
[Hint: Observe sin
= sinx x x1 sin1 1 .]
x2
Z

5. Does

0 < x 1 has a continuous

f (x)dx 0 as a, b .

f (x)dx exists implies


a0

sin2 x
,
x2

Ra
Rb
Rb
[Hint: Note that a f (x)dx = a0 f (x)dx a0 f (x)dx 0 as a, b .]
Z
2
6. Does
ex dx converge?
0
2

[Hint: Note that ex is continuous on [0, 1], and ex


Z
sin(log x)
7. Does
dx converge?
x
2

1
x2

for 1 x .]

[Hint: Use the change of variable t = log x, and the fact that
diverges.]
Z 1
8. Does
ln xdx converge?
0

[Hint: Use the change of variable t = log x.]

log 2 sin t dt

5
Geometric and Mechanical
Applications of Integrals

5.1
5.1.1

Computing Area
Using Cartesian Coordinates

Suppose a curve is given by an equation


a x b,

y = f (x),

where f : [a, b] R is a continuous function such that f (x) 0 for all x [a, b].
Then, the area under the curve, i.e., the area of the region bounded by the graph
of f , the x-axis, and the ordinates at x = a and x = b, is
Z b
k
X
lim
f (i )xi =
y dx.
(P )0

j=1

Suppose the curve is given in parametric form:


x = (t),

y = (t),

, t ,

such that a = (), b = (). Then the area under the curve takes the form
Z
(t)0 (t)dt.

If f takes both positive and negative values, but changes sign only at a finite
number of points, then the area bounded by the curve, the x-axis, and the ordinates
at x = a and x = b, is given by
Z b
|f (x)| dx.
a

Suppose f : [a, b] R and g : [a, b] R are continuous functions such that


f (x) g(x) for all x [a, b]. Then the area of the region bounded by the graphs of
f and g, and the ordinates at x = a and x = b is given by
Z b
k
X
lim
[g(i ) f (i )]xi =
[g(x) f (x)] dx.
(P )0

j=1

101

102

Geometric and Mechanical Applications of Integrals

5.1.2

M.T. Nair

Using Polar Coordinates

Suppose a curve is given in polar coordinates as


,

= (),

where : [, ] R is a continuous function. Then the the area of the region


bounded by the graph of and the rays = and = is is given by
k
X
1

lim
(P )0

5.1.3

j=1

[(i )i ](i ) =

k
X
1

lim
(P )0

j=1

[(i )]2 i =

1
2

2 d.

Some Examples

Using cartesian coordinates and parametrization


EXAMPLE 5.1 We find the area bounded by the cures defined by

y = x,
y = x2 , x 0 :
Note that the points of intersection of the curves are at x = 0 and x = 1. Also,

x x2 for 0 x 1. Hence, the required area is


1

"

xx


2

x3/2 x3
dx =

3/2
3

#1
0

1
= .
3

EXAMPLE 5.2 We find the area bounded by the ellipse


x = a cos t,

y = b sin t,

0 t 2 :

The required area is


Z
4

y dx = 4
0

/2

/2

(1 cos 2t dt = ab.

(b sin t)(asint) dt = 2ab


0

EXAMPLE 5.3 We find the area bounded by one arch of the cycloid
x = a(t sin t),

y = a(1 cos t).

One arch of the cycloid is obtained by varying t over the interval [0, 2]. Thus, the
required area is
Z 2a
Z 2
Z 2
0
y dx =
y(t)x (t) dt =
a2 (1 cos t)2 dt = 3a2 .
0

Computing Arc Length

103

Using polar coordinates


EXAMPLE 5.4 We find the area bounded by a circle of radius a. Without loss
of generality assume that the centre of the circle is the origin. Then, the circle can
be represented in polar coordinates as
= a.
Hence the required area is
1
2

2 d = a2 .

EXAMPLE 5.5 We find the area bounded by the lemniscate

= a cos 2.
The required area is
#
" Z
Z /4
1 /4 2
cos 2d = a2 .
d = a2
2
2 /4
/4

5.2
5.2.1

Computing Arc Length


Using Cartesian Coordinates

Suppose a curve is given by and equation


a x b,

y = f (x),

where f : [a, b] R is a continuous function which is differentiable except possibly


at a finite number of points. Then the length of the curve is given by
A :=

lim
(P )0

k p
X
(xi xi1 )2 + (yi yi1 )2 ,
j=1

where
yi yi1 = f (xi ) f (xi1 ) = f 0 (i )xi ,
for some i [xi1 , xi ], i = 1, . . . , k. Hence,
A :=

lim
(P )0

lim
(P )0

lim
(P )0

Z
=

k p
X
(xi xi1 )2 + (yi yi1 )2
j=1
k p
X
(xi )2 + [f 0 (i )xi ]2
j=1
k p
X
1 + [f 0 (i )]2 xi
j=1

s
1+

dy
dx

2
dx.

104

Geometric and Mechanical Applications of Integrals

M.T. Nair

If the curve y = f (x), a x b, is given in parametric form:


x = (t),
then
Z
A=

1+
a

t ,

y = (t),

dy
dx

2

s

dx =

d
dt

2


+

d
dt

2
dt.

Remark 5.1 Curves in parametric form are assumed to be piecewise smooth, i.e.,
having unique tangents except possible at a finite number of points. Note that
if a curve is given in parametric form as x = (t), y = (t) with t b,
then it has unique tangent at (x0 , y0 ) if 0 (t0 ), 0 (t0 ) exists, where t0 is such that
(x0 = (t0 ), y0 = (t0 ), and |0 (t0 )|2 + | 0 (t0 )|2 6= 0.


5.2.2

Using Polar Coordinates

Suppose a curve is given in polar coordinates as


,

= (),

where : [, ] R is a continuous function. Since


x = cos ,
we have
Z

y = sin ,

s

A=

dx
d

2


+

dy
d

2
d.

Note that

dx
dy
= 0 cos + ( sin ),
= 0 sin + cos .
d
d
Hence, it follows that
s
Z  2  2
dx
dy
A =
+
d
d
d

Z p
=
2 + 02 d.

5.2.3

Examples

Using cartesian coordinates


EXAMPLE 5.6 We find the length of the circumference of a circle of radius a.
Without loss of generality assume that the centre of the circle is the origin,i.e.,
the circle is given by x2 + y 2 = a2 . The required length is
s
 2
Z a
p
dy
1+
dx,
y = a2 x2 .
L := 4
dx
0

Computing Arc Length

105

Thus,
a

L := 4a
0

dx
= 2a.
x2

a2

Using parametric form


EXAMPLE 5.7 Now we find the length of the circle when it is represented by the
equations
x = a cos ,

0 2.

y = a sin ,

The required length is


Z

/2

s

L := 4
0

Z
=

dx
d

2


+

dy
d

2
d

/2 p

a2 sin2 + a2 cos2 d = 2a.

EXAMPLE 5.8 Let us find the length of the ellipse


x = a cos ,

y = b sin ,

0 2.

The required length is


Z

/2

s

L := 4
0

Z
=

dx
d

2


+

dy
d

2
d

/2 p

a2 sin2 + b2 cos2 d

Z
=

/2 p

a2 (1 cos2 ) + b2 cos2 d

Z
=

/2 p

a2 (a2 b2 ) cos2 d

Z
=

4a

/2 p

1 2 cos2 d,

where = a ab . The above integral is not expressible in standard form unless


= 1, i.e., unles b = a in which case the ellipse is the circle. But, the integral can
be approximately computed numerically.
EXAMPLE 5.9 We find the length of the astroid: x = a cos3 t, y = a sin3 t.

106

Geometric and Mechanical Applications of Integrals

M.T. Nair

The required length is


/2

s

dx
d

L := 4
0


+

dy
d

2
d

/2 p

Z
=

2

9a2 cos4 t sin2 t + 9a2 sin4 t cos3 t d

4
0

/2 p

Z
=

cos2 t sin2 dt = 6a.

12a
0

Using polar coordinates


EXAMPLE 5.10 We find the length of the cardioid = a(1 + cos ).
R 2 p
The required length is L := 0
2 + 02 d. Since
02 = a2 sin2 ,

2 = a2 (1 + cos )2 ,
we have
L=

Z
2a

1 + cos d = 4a

5.3

Z
0





cos d = 8a.

2

Computing Volume of a Solid

Suppose that a three dimensional object, a solid, lies between two parallel planes
x = a and x = b. Let (x) be the area of the cross section of the solid at the point x,
with cross section being parallel to the yz-plane. We assume that the function (x),
x [a, b] is continuous. Now, consider a partition P : a = x0 < x1 < . . . < xk = b
of the interval [a, b]. Then the volume of the solid is given by
lim
(P )0

k
X

Z
(i )xi =

(x) dx.
a

j=1

EXAMPLE 5.11 Let us compute the volume of the solid enclosed by the ellipsoid
x2 y 2 z 2
+ 2 + 2 = 1.
a2
b
c
For a fixed x [a, a], the boundary of the cross section at x is given by the equation
y2 z2
x2
+
=
1

,
b2
c2
a2
i.e.,
y2
z2
+
= 1,
(x)2 (x)2

where

x2
(x) = b 1 2 ,
a

r
(x) = c 1

x2
.
a2

Computing Volume of a Solid of Revolution

107

Hence,


x2
(x) = (x)(x) = bc 1 2 ,
a
and the required volume is
Z



x2
4
1 2 dx = abc.
a
3
a

Z
(x) dx = bc

V :=

In particular, volume of the solid bounded by the sphere x2 + y 2 + z 2 = a2 is 43 a3 .

5.4

Computing Volume of a Solid of Revolution

Suppose a solid is obtained by revolving a curve y = f (x), a x b, with x-axis


as axis of revolution. We would like to find the volume of the solid.
In this case the area of cross section at x is given by
(x) = y 2 = [f (x)]2 ,

a x b.

Hence, the volume of the solid of revolution is


b

Z
V :=

(x) dx =
a

y 2 dx.

EXAMPLE 5.12 Let us compute the volume of the solid of revolution of the curve
y = x2 about x-axis for a x a. The required volume is
Z

a
2

V :=

y dx =
a

2
x4 dx = a5 .
5
a

EXAMPLE 5.13 We compute the volume of the solid of revolution of the catenary
y=


a  x/a
e
+ ex/a
2

about x-axis for 0 x b. The required volume is


Z

V :=

y dx =
0

Z
2

a2 b  2x/a
a2  x/a
x/a
e
+e
dx =
e
+ e2x/a + 2 .
4
4 0

We see that
V :=

 b3
a3  2b/a
e
e2b/a +
.
8
8

108

5.5

Geometric and Mechanical Applications of Integrals

M.T. Nair

Computing Area of Surface of Revolution

Suppose a solid is obtained by revolving a curve y = f (x), a x b, with x-axis


as axis of revolution. We would like to find the area of the surface of the solid.
The required area is

A :=

lim
(P )0

k
X

2f (i )si ,

j=1

where P : a = x0 < x1 < . . . < xk = b is a partition of the interval [a, b], and
p
1 + [f 0 (i )]2 xi ,

si :=

i = 1, . . . , k.

Thus

A=

lim
(P )0

k
X

Z b
p
y
2f (i ) 1 + [f 0 (i )]2 xi = 2

s
1+

j=1

dy
dx

2
dx.

EXAMPLE 5.14 We find the surface of revolution of the parabola y 2 = 2px,


0 x a for p > 0. The required area is
Z
A = 2

1+

dy
dx

2
dx

r
Z ap
p

= 2
2px 1 +
dx = 2 p
p + 2x dx
2x
0
0

a

i
2 p h
2
3/2 1
= 2 p (2x + p)
=
(2a + p)3/2 p3/2 .
3
2 0
3
Z

5.6

ap

Centre of Gravity

Suppose A1 , A2 , . . . , An are material particles on the plane at coordinates (x1 , y1 ), (x2 , y2 ), . . . , (xn , yn )
and masses m1 , m2 , . . . mn respectively. Then the centre of gravity of the system
of these particles is at the point A = (xC , yC ), where
Pn
xi mi
xC := Pi=1
,
n
i=1 mi

Pn
yi mi
yC := Pi=1
.
n
i=1 mi

Now we attempt to define the centre of gravity of a material line and material
planar region enclosed by certain curves.

Centre of Gravity

5.6.1

109

Centre of gravity of a material line in the plane

Suppose a curve L is given by the equation y = f (x), a x b. We assume


that this curve is a material line. Suppose the density of the material at the point
X = (x, y) is (X). This density is defined as follows: Suppose M (X, r) is the mass
of an arc of the line containing the point X with length r. Then the density of the
material at the point x is defined by
(X) := lim

r0

M (X, r)
.
r

Now, in order to find the centre of gravity of L, we first consider a partition P :


a = x0 < x1 < . . . < xk , and take points i = [xi1 , xi ], i = 1, . . . , n. Then we take
the the centre of gravity of the system of material points at (1 , f (1 ), (2 , f (2 ), . . . , (k , f (k )
as
Pn
Pn
f ( ) s
i i si
i=1
Pn i i i .
,
yC (P ) := i=1
xC (P ) = Pn
i=1 i si
i=1 i si
Here, si is the length of the arcs joining (xi1 , yi1 ) to (xi , yi ), and i is the density
at the point (i , f (i ). Here yi = f (xi ). Note that i si is the approximate mass
of the arc joining (xi1 , yi1 ) to (xi , yxi ). Now, the centre of gravity of L is at
(xC , yC ), where
Pn
Pn
i i si
f ( ) s
i=1
Pn i i i .
xC = lim Pi=1
,
y
:=
lim
C
n

s
(P )0
(P
)0
i
i=1 i
i=1 i si
Assuming that the function (X) := (x, f (x)) is continuous on [a, b], we see that
r
r
 2
 2
Rb
Rb
dy
dy
x(x,
y)
1
+
y(x,
y)
1
+
dx
dx
dx
dx
a
a
r
r
xC =
,
y
=
.
C
 2
 2
Rb
Rb
dy
dy
dx
dx
a (x, y) 1 + dx
a (x, y) 1 + dx

Example. We find the centre of gravity of the semi-circlular arc x2 + y 2 = a2 ,


y 0, assuming
that the density of the material is constant. In this case, y =

2
2
f (x) := a x , so that it follows that
s
 2
dy
a
1+
=
.
2
dx
a x2
Hence, since (x, y) is constant,

xC = 0

 2
dy
y
1
+
dx
dx
a
2a
r
yC =
=
.


2

Ra
dy
1 + dx dx
a
Ra

110

5.6.2

Geometric and Mechanical Applications of Integrals

M.T. Nair

Centre of gravity of a material planar region

Next we consider the centre of gravity of a material planar region bounded by


two curves
y = f (x),

y = g(x),

with

f (x) g(x)

a x b.

Suppose that the density of the material at the point X is (X). This density is
defined as follows: Suppose M (X, r) is the mass of the circular region S(X, r)
with centre at x and radius r > 0, and (X, r) is the area of the same circular
region. Then the density of the material at the point x is defined by
M (X, r)
.
r0 (X, r)

(X) := lim

Now, in order to find the centre of gravity of , we first look at the following special
case: Suppose is a rectangle given by a1 x b1 , a2 y b2 . Then we can
infer that the centre of gravity of such rectangle is located at the point


a1 + b1 a2 + b2
,
.
2
2
Taking the above obervation into account, we consider a partition P : x0 < x1 <
. . . < xk of the interval [a, b], and consider the rectangular strips:
Ri :

xi1 x xi ,

f (i ) y g(i ),

i = 1, . . . , k,

where i = xi12+xi , i = 1, . . . , k. If is the (constant) density of the material, then


the mass of the rectangular strip Ri is
mi = [g(i ) f (i )]xi ,

i = 1, . . . , k.

Assuming that the mass of the rectangular strip Ri is concentrated at its mid-point:


f (i ) + g(i )
Xi : i ,
,
2
we consider the centre of gravity of the system of material points at Xi as
Pn f (i )+g(i )
Pn
mi
i mi
i=1
xC,P := Pn
,
yC,P := i=1Pn 2
.
i=1 mi
i=1 mi
Now the centre of gravity of is defined as
xC =

lim xC,P ,

(P )0

yC =

lim yC,P ,

(P )0

i.e.,
xC

Pn
i [g(i ) f (i )]xi
=
lim Pi=1
n
(P )0
i=1 [g(i ) f (i )]xi
Rb
x[g(x) f (x)] dx
= Ra b
a [g(x) f (x)] dx

Moment of Inertia

111

Pn
yC

1
i=1 2 [f (i + g(i )][g(i ) f (i )]xi
Pn
i=1 [g(i ) f (i )]xi

lim
(P )0

1
2

Rb

+ g(x)][g(x) f (x)] dx
Rb
a [g(x) f (x)] dx

a [f (x

EXAMPLE 5.15 We find the coordinates of the centre of gravity of a segment of


a parabola y 2 = a x cut off by the straight line x = a.
In this case

f (x) = a x,

g(x) =

a x,

0 x a.

Hence the coordinates of the centre of gravity are


Rb
2 a x a x dx
3
= Rb
xC = R b
= a.
5
a [g(x) f (x)] dx
a 2 a x dx
Rb
1 a [f (x + g(x)][g(x) f (x)] dx
= 0.
yC =
Rb
2
[g(x) f (x)] dx
Rb
a

x[g(x) f (x)] dx

5.7

Moment of Inertia

Suppose there are n material points in the plane. Let their masses be m1 , m2 , . . . mn
respectively. Suppose that these points are at distances d1 , . . . , dn from a fixed point
O. Then the moment of inertia of the system of these points with respect to the
point O is defined by the quantity:
IO :=

n
X

d2i mi .

i=1

If O is the origin, and (x1 , y1 ), (x2 , y2 ), . . . , (xn , yn ) are the points, then
IO :=

n
X

(x2i + yi2 )mi .

i=1

5.7.1

Moment of inertia of a material line in the plane

Suppose a curve L is given by the equation y = f (x), a x b. We assume


that this curve is a material line. Suppose the density of the material at the point
X = (x, y) is (X).
Now, in order to find the moment of inertia of L, we first consider a partition
P : a = x0 < x1 < . . . < xk , and take points i = [xi1 , xi ], i = 1, . . . , n. Then

112

Geometric and Mechanical Applications of Integrals

M.T. Nair

we consider the moment of inertia of the system of material points at (1 , i ), i =


1, . . . , n. Here, i = f (i ), i = 1, . . . , n.
IO,P :=

n
X

(i2 + i2 )mi .

i=1

Thus,

n
X

IO,P :=

(i2 + i2 )i si .

i=1

Here, si is the length of the arcs joining (xi1 , yi1 ) to (xi , yi ), and i is the
density at the point (i , i ). Note that i si is the approximate mass of the arc
joining (xi1 , f (xi1 ) to (xi , f (xi ). Now, assuming that the functions f (x) and
(x) := (x, f (x)) are continuous on [a, b], the moment of inertial of L with respect
to O is
IO =

lim IO,P

(P )0

n
X

lim
(P )0

(i2 + i2 )i si

i=1

b
2

(x + y )(x, y)


1+

5.7.2

dy
dx

2
dx.

Moment of inertia of a circular arc with respect to the centre

Suppose the given curve is a circular arc: = a, . Following the arguments


in the above paragraph, we compute the moment of inertia using polar coordinates:
The moment of inertia, in this, case is given by
IO :=

lim
(P )0

n
X

d2i mi ,

i=1

where di = a, mi = i ai , for i = 1, . . . , n, so that


IO =

n
X

lim
(P )0

a2 i [ai ] = a3

()d.

i=1

Here, () is the point density. If () = , a constant, then


IO = a

()d = ( )a3 .

In particular, M.I of the circle = a, 0 2, is


IO = 2a3 .

Additional Exercises

5.7.3

113

Moment of inertia of a material sector in the plane

The region is R : 0 a, with constant density . To find the M.I. of


R, we partition it by rays and circular arcs:
P :

= 0 < 1 < 2 < . . . < n = ,

Q:

0 = 0 < 1 < 2 < . . . < m = a.

Consider the elementary region obtained by the above partition:


Rij : j1 j a,

i1 i i .

Assume that the the mass of this region Rij is concentrated at the point (
j , i ),

where j [j1 , j ], i [i1 , i ]. Then the MI of the material point at (


j , i )
is mij d2ij where mij is the mass of the region Rij which is approximately equal to
[
j i j ], and dij = j . Thus the MI of the sub-sector i1 i is defined
by
lim
(Q)0

n
X

mij d2ij

lim
(Q)0

j=1

(Q)0

j=1
n
X

lim
Z

n
X
[
j i j ] 2j


3j j i

j=1


d i
3

a4
i .
4

From this, it follows that, the moment of inertia of the sector is


lim
(P )0

m
X
a4
i=1

i =

( )a4
.
4

In particular, moment of inertia of a circular disc is


a4
M a2
=
,
2
2
where M = a2 is the mass of the disc.
Exercise 5.1 If M is the mass of a right circular homogeneous cylinder with base
2
radius a, then show that its moment of inertia is M2a .

5.8

Additional Exercises

1. Find the area of the portion of the circle x2 + y 2 = 1 which lies inside the
parabola y 2 = 1 x.

114

Geometric and Mechanical Applications of Integrals

M.T. Nair

[Hind: Area enclosed by the circle in the second and third quadrant and
the area enclosed by the
parabola in the first and fourth quadrant. The the
R1
required area is 2 + 2 0 1 x dx.
Ans: 2 + 43 . ]
2. Find the area common to the cardioid = a(1 + cos ) and the circle =

3a
2 .

[Hind: The points of intersections of the given


are given by 1+cos = 32 ,
h curves
i
R
R
/3 3a 2
1
2 (1 + cos )2 d .
d
+
a
i.e., for = 3 . Hence the required area is 2 12 0
2
2 /3
Ans:

7
4

9 3
8 .

3. For a, b > 0, find the area included betwee the parabolas y 2 = 4a(x + a) and
y 2 = 4b(b x).
[Hind: Points of intersection of the curves is given byh a(x + a) = b(b x), i.e.,
i

Rb p
R ba p
2 a2
= ba; y = 2 ab. The required area is 2 a
4a(x + a) + ba 4b(b x) dx .
x = b a+b

Ans: 38 ab(a + b).]


4. Find the area of the loop of the curve r2 cos = a2 sin 3
[Hint:r = 0 for = 0 and = /3, and r is maximum for = /6. The area
R /3 2
is 0 r2 d. ]
5. Find the area of the region bounded by the curves x y 3 = 0 and x y = 0.
[Hint:
Points of intersections of the curves are at x = 0, 1, 1. The area is
R 1 1/3
2 0 (x x)dx. Ans: 1/2 ]
6. find the area of the region that lies inside the circle r = a cos and outside
the cardioid r = a(1 cos ).
[Hint: Note that the circle is the one with centre at (0, a/2) and radius a/2.
R /3
The curves intersect at = /3. The required area is /3 (r12 r22 )d,

3
where r1 = a cos , r2 = a(1 cos ). Ans: a3 (3 3 ) ]
7. Find the area of the loop of the curve x = a(1 t2 ), y = at(1 t2 ) for
1 t 1.
[Hint: y = 0 for t {1, 0, 1}, and y negative for 1 t 0 and positive for
0 t 1. Also, Ry 2 = x2 (a
R 0x)/a so that the curve is symmetric w.r.t. the
a
x-axis. Area is 2 0 ydx = 2 1 y(t)x0 (t)dt. Ans: 8a2 /15 ]
8. Find the length of an arch of the cycloid x = a(t sin t), y = a(1 cos t).
[Hint: The curve cuts the x-axisR at p
x = a and x = 2a for t = 0 and t = 2
2
respectively. Thus the length is 0
[x0 (t)]2 + [y 0 (t)]2 dt. Ans: 8a. ]

Additional Exercises

115

9. For a > 0, find the length of the loop of the curve 3a y 2 = x(x a)2 .
[Hint: The curve cuts the x-axis at x = a,rand the curve is symmetric w.r.t.
 2
Ra
dy
the x-axis. Thus the required area is 2 0 1 + dx
dx. Note that 6ayy 0 =
(x a)(3x a), so that 1 + y 02 =
10. Find the length of the curve r =
[Hind: ` :=

(3x+a)2
12ax .

2
1+cos ,

Ans:

4a

.
3

0 /2.

R /4
R /2 p
r2 + [r0 ]2 d = 2 0 sec3 d. Ans: 2 + ln( 2 + 1). ]
0

11. Find the volume of the solid obtained by revolving the curve y = 4 sin 2x,
0 x /2, about y-axis.
[Hint: writing y = 4 sin 2x, 0 x /4 and y = 4 sin 2u, /4 u /2, the
R4
R /2
R /4
required volume is 0 (u2 x2 )dy = /4 u2 (8 cos 2u)du 0 x2 (8 cos 2x)dx.
Also, note that the curve is symmetric w.r.t. the line x = /4. Hence, the
R /4
required volume is given by 0 [( 4 x)2 x2 ]dy. Ans: 2 2 .]
12. Find the area of the surface obtained by revolving a loop of the curve 9ax2 =
y(3a y)2 about y-axis.
r
 2
R 3a
[Hind: x = 0 iff y = 0 or y = 3a. The required area is 2 0 x 1 + dx
dx.
dy
Ans: 3a2 . ]
13. Find the area of the surface obtained by revolving about x-axis, an arc of the
catenary y = c cosh(x/c) between x = a and x = a for a > 0.
Ra
Ra p
[Hind: The area is 2 a y 1 + y 02 dx = 2 c a cosh2 xc dx. Ans: c [2a +
c sinh 2a
c ]. ]
14. The lemniscate 2 = a2 cos 2 revolves about the line = 4 . Find the area of
the surface of the solid generated.
R /4 p
[Hind: The required surface is 22 /4 h 2 + 02 d, where h := sin

2
= a cos 3 so that 2 + 02 = cosa 2 . Ans: 4a2 . ]


,

15. Find the volume of the solid generated by the cardioid = a(1 + cos ) about
the initial line. [Ans: 83 . ]

6
Sequence and Series of Functions

6.1
6.1.1

Sequence of Functions
Pointwise Convergence and Uniform Convergence

Let J be an interval in R and let F(J) be the set of all real valued functions defined
on F .
Definition 6.1 By a sequence of functions on J we mean a map F : N F(J).
If fn := F (n) for n N, then we denote F by (fn ), and call (fn ) as a sequence of
functions.

Definition 6.2 (a) We say that the sequence (fn ) of functions on J converges
pointwise on J if for each x J, the sequence (fn (x)) of real numbers converges.
(b) Suppose (fn ) converges pointwise on J. Let f : J R be defined by
f (x) = limn fn (x), x J. Then we say that (fn ) converges to f pointwise
on J, and f is the pointwise limit of (fn ), and in that case we may also write
fn f

pointwise on

J.


Thus, (fn ) converges to f pointwise on J if and only if for every > 0 and for
each x J, there exists N N (depending, in general on both and x) such that
|fn (x) f (x)| < for all n N .
Exercise 6.1 Pointwise limit of a sequence of functions is unique.
EXAMPLE 6.1 Consider fn : R R defined by
fn (x) =

sin(nx)
,
n

xR

and for n N. Then we see that for each x R,


|fn (x)|

1
n
116

n N.

Sequence of Functions

117

Thus, (fn ) converges pointwise to f on R, where f is the zero function on R, i.e.,


f (x) = 0 foe very x R.

Suppose (fn ) converges to f pontwise on J. As we have mentioned, it can happen
that for > 0, and for each x J, the number N N satisfying |fn (x) f (x)| <
n N depends not only on but also on the point x. For instance, consider the
following example.
EXAMPLE 6.2 Let fn (x) = xn for x [0, 1] and for n N. Then we see that for
0 x < 1, fn (x) 0, and fn (1) 1 as n . Thus, (fn ) converges pointwise to
a function f defined by

0, x 6= 1,
f (x) =
1, x = 1.
In particular, (fn ) converges to the zero function on [0, 1). We show that this
convergence is not uniform.
Suppose for every > 0, there exists N N such that |xn f (x)| < for all
n N and x [0, 1). In particular, |xN | < for all for all x [0, 1), so that taking
limit as x 1, we get 1 . This is impossible, if we had taken < 1.

For > 0, if we are able to find an N N which does not vary as x varies
over J such that |fn (x) f (x)| < for all n N , then we say that (fn ) converges
uniformly to f on J. Following is the precise definition of uniform convergence of
(fn ) to f on J.
Definition 6.3 Suppose (fn ) is a sequence of functions defined on an interval J.
We say that (fn ) converges to a function f uniformly on J if for every > 0
there exists N N (depending only on ) such that
|fn (x) f (x)| <

n N

and x J,

and in that case we write


fn f

uniformly on

J.


We observe the following:


If (fn ) converges uniformly to f , then it converges to f pointwise as well.
Thus, if a sequence does not converge pointwise to any function, then it can not
converge uniformly.
If (fn ) converges uniformly to f on J, then (fn ) converges uniformly to f on
every subinterval J0 J.
Example 6.2 shows that a pointwise convergent sequence of functions need not
converge uniformly. Another way of seeing the nonuniform convergence of the sequence (fn ) in Example 6.2 is as follows: If we have uniform convergence, then for

118

Sequence and Series of Functions

M.T. Nair

any > 0, there exists N N such that |xn | < for all n N and for all x [0, 1).
In particular, we have
|xN | <
x [0, 1).
Hence, taking limit as x 1, we would get 1 < , which is not possible if we had
chosen < 1.
Here is another example of a sequence of functions which converges pointwise
but not uniformly.
EXAMPLE 6.3 For each n N, let
fn (x) =

nx
,
1 + n2 x2

x [0, 1].

Note that fn (0) = 0, and for x 6= 0, fn (x) 0 as n . Hence, (fn ) converges


poitwise to the zero function. We do not have uniform convergence, as fn (1/n) = 1/2
for all n. Indeed, if (fn ) converges uniformly, then there exists N N such that
|fN (x)| <

x [0, 1].

In particular, we must have


1
= |fN (1/N )| <
2

x [0, 1].

This is not possible if we had chosen < 1/2.

EXAMPLE 6.4 Consider the sequence (fn ) defined by


fn (x) = tan1 (nx),

x R.

Note that fn (0) = 0, and for x 6= 0, fn (x) /2 as n . Hence, the given


sequence (fn ) converges pointwise to the function f defined by

0,
x = 0,
f (x) =
/2, x 6= 0.
However, it does not converge uniformly to f on any interval containing 0. To
see this, let J be an interval containing 0 and > 0. Let N N be such that
|fn (x) f (x)| < for all n N and for all x J. In particular, we have
|fN (x) /2| <

x J \ {0}.

Letting x 0, we have /2 = |fN (0) /2| < which is not possible if we had
chooses < /2.

Now, we give a theorem which would help us to show non-uniform convergence
of certain sequence of functions.
Theorem 6.1 Suppose fn and f are functions defined on an interval J. If there
exists a sequence (xn ) in J and c 6= 0 such that an := fn (xn )f (xn ) c as n ,
then (fn ) does not converge uniformly to f on J.

Sequence of Functions

119

Proof. Suppose (fn ) converges uniformly to f on J. Then taking = |c|/2, there


exists N N such that
|fn (x) f (x)| <

|c|
2

n N,

x J.

In particular,
|fn (xn ) f (xn )| <

|c|
2

n N.

Now, taking limit as n , it follows that


|c| = lim |fn (xn ) f (xn )| <
n

|c|
2

which is a contradiction. Hence our assumption that (fn ) converges uniformly to f


on J is wrong.
In he case of 6.2, taking xn = n/(n + 1), we see that
n

1
n
.
fn (xn ) =
n+1
e
Hence, (fn ) does not converge to f 0 uniformly on [0, 1).
In the case of Example 6.3, we may take xn = 1/n, and in the case of Example
??, we may take xn = /n, and apply Theorem 6.1.
Here is a sufficient condition for uniform convergence. Its proof is left as an
exercise.
Theorem 6.2 Suppose fn for n N and f are functions on J. If there exists a
sequence (n ) of positive reals satisfying n 0 as n and
|fn (x) f (x)| n

n N,

then (fn ) converges uniformly to f .


Exercise 6.2 Supply detailed proof for Theorem 6.2.

Here are a few examples to illustrate the above theorem.


EXAMPLE 6.5 For each n N, let
fn (x) =
Then we have

2nx
,
1 + n4 x2

x [0, 1].

1
2nx
0 fn (x)
= .
4
2
n
2 n x

Thus, by Theorem 6.2, (fn ) converges uniformly to f = 0.

120

Sequence and Series of Functions

M.T. Nair

EXAMPLE 6.6 For each n N, let


fn (x) =

1
log(1 + n4 x2 ),
n3

Then we have
0 fn (x)
Taking g(t) :=

1
t3

x [0, 1].

1
log(1 + n4 ) =: n
n3

n N.

log(1 + t4 ) for t > 0, we see, using LHospitals rule that


4t3
= 0.
t 3t2 (1 + t4 )

lim g(t) = lim

In particular,
1
log(1 + n4 ) = 0.
n n3
Thus, by Theorem 6.2, (fn ) converges uniformly to f = 0.
lim

We may observe that in Examples 6.2 and 6.4, the limit function f is not continuous, although every fn is continuous. This makes us to ask the following:
Suppose each fn is continuous function on J and (fn ) converges to f pointwise.
Under what condition can we assert that f is continuous?
Also, we may want to know the answers to the following questions:
Suppose each fn is continuous on J and (fn ) converges pointwise to f on J.
Do we have
Z b
Z b
f (x)dx = lim
fn (x)dx
a

n a

for every [a, b] J?


Suppose each fn is continuously differentiable on J and (fn ) converges to a
function f . Then, is the function f differentiable on J? If f is differentiable on J,
then do we have the relation
d
d
f (x) = lim
fn (x)dx ?
n dx
dx
The answers to the above two questions need not be affirmative as the following
two examples show.
EXAMPLE 6.7 For each n N, let
fn (x) = nx(1 x2 )n ,

0 x 1.

Then we see that


lim fn (x) = 0 x [0, 1].

Sequence of Functions

121

Indeed, for each x (0, 1),


fn+1 (x)
= x(1 x2 )
fn (x)

n+1
n

x(1 x2 )

as

n .

Since x(1 x2 ) < 1 for x (0, 1), the result follows. But,
Z

fn (x)dx =
0

n
1

2n + 2
2

as

n .

Thus, limit of the integrals is not the integral of the limit.

EXAMPLE 6.8 For each n N, let


fn (x) =

sin(nx)

,
n

x R.

Then we see that


lim fn (x) = 0 x [0, 1].

But, fn0 (x) =

n cos(nx) for all n N, so that

fn0 (0) = n as n .

Thus, limit of the derivatives is not the derivative of the limit.

6.1.2

Continuity and uniform convergence

Theorem 6.3 Suppose (fn ) is a sequence of continuous functions defined on an


interval J which converges uniformly to a function f . Then f is continuous on J.
Proof. Suppose x0 J. Then for any x J and for any n N,
|f (x) f (x0 )| |f (x) fn (x)| + |fn (x) fn (x0 )| + |fn (x0 ) f (x0 )|.

()

Let > 0 be given. Since (fn ) converges to f uniformly, there exists N N such
that
|fn (x) fn (x)| < /3 n N, x J.
Since fN is continuous, there exists > 0 such that
|fN (x) fN (x0 )| < /3

whenever

|x x0 | < .

Hence from (), we have


|f (x) f (x0 )| |f (x) fN (x)| + |fN (x) fN (x0 )| + |fN (x0 ) f (x0 )| <
whenever |x x0 | < . Thus, f is continuous at x0 . This is true for all x0 J.
Hence, f is a continuous function on J.

122

6.1.3

Sequence and Series of Functions

M.T. Nair

Integration and uniform convergence

Theorem 6.4 Suppose (fn ) is a sequence of continuous functions defined on an interval [a, b] which converges uniformly to a function f on [a, b]. Then f is continuous
and
Z
Z
b

lim

n a

f (x)dx.

fn (x)dx =
a

Proof. We already know by Theorem 6.3 that f is a continuous function. Next


we note that
Z b
Z b
Z b




f
(x)dx

f
(x)dx
|fn (x) f (x)|dx.
n


a

let > 0 be given. By uniform convergence of (fn ) to f , there exists N N such


that
|fn (x) f (x)| < /(b a) n N, x [a, b].
Hence, for all n N ,
Z b
Z b
Z b




|fn (x) f (x)|dx < .
f
(x)dx

f
(x)dx
n


a

This completes the proof.

6.1.4

Differentiation and Uniform Convergence

Theorem 6.5 Suppose (fn ) is a sequence of continuously differentiable functions


defined on an interval J such that
(i) (fn0 ) converges uniformly to a function, and
(ii) (fn (a)) converges for some a J.
Then (fn ) converges to a continuously differentiable function f and
lim f 0 (x)
n n

= f 0 (x)

x J.

Proof. Let g(x) := lim fn0 (x) for x J, and := lim fn (a). Since the convern

gence of (fn0 ) to g is uniform, by Theorem 6.4, the function g is continuous and


Z x
Z x
lim
fn0 (t)dt =
g(t)dt.
n a

Rx

0
Let (x)
a g(t)dt, x J. Then is differentiable and (x) = g(x) for x J.
R x :=
But, a fn0 (t)dt = fn (x) fn (a). Hence, we have

lim [fn (x) fn (a)] = (x).

Thus, (fn ) converges pointwise to a differentiable function f defined by f (x) =


(x) + , x J, and (fn0 ) converges to f 0 .

Series of Functions

6.2

123

Series of Functions

Definition 6.4 By a series of functions on a interval J, we mean an expression


of the form

X
X
fn or
fn (x),
n=1

n=1

where (fn ) is a sequence of functions defined on J.


Definition 6.5 Given a series

n=1 fn (x)

sn (x) :=

n
X

of functions on an interval J, let


x J.

fi (x),

i=1

Then sn is called the n-th partial sum of the series

n=1 fn .

P
on an interval J, and let
Definition 6.6 Consider a series
n=1 fn (x) of functions P
sn (x) be its n-th partial sum. Then we say that the series
n=1 fn (x)
(a) converges at a point x0 J if (sn )) converges at x0 ,
(b) converges pointwise on J if (sn ) converges pointwise on J, and
(c) converges uniformly on J if (sn ) converges uniformly on J.

The proof of the following two theorems are obvious from the statements of
Theorems 6.4 and 6.5 respectively.
P
Theorem 6.6 Suppose (fn ) is a sequence of continuous functions on J. If
n=1 fn (x)
converges uniformly on J, say to f (x), then f is continuous on J, and for [a, b] J,
Z

f (x)dx =
a

Z
X

fn (x)dx.

n=1 a

Theorem
P differentiable functions on
P 6.70 Suppose (fn ) is a sequence of continuously
f
(x)
converges
uniformly
on
J,
and
if
J. If
n=1 n
n=1 fn (x) converges at some
P
point x0 J, then n=1 fn (x) converges to a differentiable function on J, and
!

X
d X
fn (x) =
fn0 (x).
dx
n=1

n=1

Next we consider a useful sufficient condition to check uniform convergence. First


a definition.
P
Definition 6.7 We say that
n=1 fn is a dominated series if there exists a
sequence (n ) of positiveP
real numbers such that |fn (x)| n for all x J and for
all n N, and the series

n=1 n converges.

124

Sequence and Series of Functions

M.T. Nair

Theorem 6.8 A dominated series converges uniformly.


P
Proof. Let
n=1 fn be a dominated series defined on an interval J, and let (n )
be a sequence of positive reals such that
(i) |fn (x)| n for all n N and for all x J, and
P
(ii)
n=1 n converges.
P
Let sn (x) = ni=1 fi (x), n N. Then for n > m,
n

n
n
X

X
X


|sn (x) sm (x)| =
fi (x)
|fi (x)|
i = n m ,


i=m+1

i=m+1

i=m+1

Pn

where n =
n=1 n converges, the sequence (n ) is a Cauchy
k=1 k Since
sequence. Now, let > 0 be given, and let N N be such that
|n m | < n, m N.
Hence, from the relation: |sn (x) sm (x)| n m , we have
|sn (x) sm (x)| < n, m N, x J.
This, in particular implies that {sn (x)} is also a Cauchy sequence at each x J.
Hence, {sn (x)} converges for each x J. Let f (x) = limn sn (x), x J. Then,
we have
|f (x) sm (x)| = lim |sn (x) sm (x)| < m N, x J.
n

Thus, the series

n=1 fn

converges uniformly to f on J.

EXAMPLE 6.9 The series

n=1

cos nx
1



2,
2
n
n
and

1
n=1 n2

is convergent.

cos nx
n2

and

n=1

sin nx
n2



sin nx
1


n2 n2

are dominated series, since

n N


P
EXAMPLE 6.10 The series
xn is a dominated series on [, ] for 0 < < 1,
n=0
P
n
n
since |x | for all n N and n=0 n is convergent. Thus, the given series is a
dominated series, and ehnce, it is uniformly convergent.

P
x
EXAMPLE 6.11 Consider the series n=1 n(1+nx
2 ) on R. Note that


x
1
1

,
n(1 + nx2 )
n 2 n

X
1
converges. Thus, the given series is dominated series, and hence it
3/2
n
n=1
converges uniformly on R.


and

Series of Functions

125

P
x
EXAMPLE 6.12 Consider the series
n=1 1+n2 x2 for x [c, ), c > 0. Note
that
x
x
1
1
2 2 = 2 2
2
2
1+n x
n x
n x
n c

X
1
and
converges. Thus, the given series is dominated series, and hence it
n2
n=1
converges uniformly on [c, ).



n
P
x
EXAMPLE 6.13 The series
is dominated on [0, ): To see this,
n=1 xe
note that

n
xn
xn
n!
xex = nx
= n
n
e
(nx) /n!
n
P n!
and the series n=1 nn converges.
It can also be seen that |xex | 1/2 for all x [0, ).

P
n1 is not uniformly convergent on (0, 1); in
EXAMPLE 6.14 The series
n=1 x
particular, not dominated on (0, 1). This is seen as follows: Note that
sn (x) :=

n
X

xk1 =

k=1

1 xn
1
f (x) :=
1x
1x

as n .

Hence, for > 0,


|f (x) sn (x)| <

n
x


1 x < .

Hence, if there exists N N such that |f (x) sn (x)| < for all n N for all
x (0, 1), then we would get
|x|N
< x (0, 1).
|1 x|
This is not possible, as |x|N /|1 x| as x 1.
However, the above series is dominated on [a, a], where 0 < a < 1.

P
EXAMPLE 6.15 The series n=1 (1x)xn1 is not uniformly convergent on [0, 1];
in particular, not dominated on [0, 1]. This is seen as follows: Note that

n
X
1 xn
if x 6= 1
k1
sn (x) :=
(1 x)x
=
0
if x = 1.
k=1

In particular, sn (x) = 1 xn for all x [0, 1) and n N. By Example 6.2, we know


that (sn (x)) converges to f (x) 1 pointwise, but not uniformly.

P
Remark 6.1 Note that if a series n=1 fn converges uniformly to a function f on
an interval J, then we must have
n := sup |sn (x) f (x)| 0
xJ

as

n .

126

Sequence and Series of Functions

M.T. Nair

Here, sn is the n-th partial sum of the series.


PConversely, if n 0, then the series
is absolutely uniformly convergent. Thus, if
n=1 fn converges to a function f on J,
and if supxJ |sn (x) f (x)| 6 0 as n , then we can infer that the convergence
is not uniform.
As an illustration, consider the Example 6.15. There we have
 n
x
if x 6= 1
|sn (x) f (x)| =
0
if x = 1.
Hence, sup|x|1 |sn (x) f (x)| = 1. Moreover, the limit function f is not continuous.
Hence, the non-uniform convergence also follows from Theorem 6.6.

P
Exercise 6.3 Consider a series n=1 fP
n and an := supxJ |fn (x)|. Show that this
series is dominated series if and only if
J
n=1 an converges.
Next example shows that in Theorem 6.7, the condition that the derived series
converges uniformly is not a necessary condition for the the conclusion.
P n
converges to
EXAMPLE 6.16 Consider the series
n=0 x . We know
Pthat itn1
nx
converges
1/(1 x) for |x| < 1. It can be seen that the derived series P
n=1

n1
converges.
uniformly for |x| for any (0, 1). This follows since n=1 n
Hence,

X
1
d 1
=
=
nxn1 for |x| .
(1 x)2
dx 1 x
n=1

The above relation is true for x in any open interval J (1, 1); because we can
choose sufficiently close to 1 such that J [, ]. Hence, we have

X
1
=
nxn1
(1 x)2

for |x| < 1.

n=1

We know that the given series is not uniformly convergent (see, Example 6.14). 

6.3

Additional Exercises

X
x2
for
x

0.
Show
that
the
series
fn (x) does not
(1 + x2 )n
n=1
converge uniformly.
x
, x R. Show that (fn ) converge uniformly, whereas (fn0 )
2. Let fn (x) =
1 + nx2

0
does not converge uniformly. Is the relation lim fn0 (x) = lim fn (x) true
n
n
for all x R?

1. Let fn (x) =

log(1 + n3 x2 )
2nx
, and gn (x) =
for x [0, 1]. Show that
2
n
1 + n3 x2
the sequence (gn ) converges uniformly to g where g(x) = 0 for all x [0, 1].
Using this fact, show that (fn ) also converges uniformly to the zero function
on [0, 1].

3. Let fn (x) =

Additional Exercises
2
n x,
n2 x + 2n,
4. Let fn (x) =

0,

127

0 x 1/n,
1/n x 2/n,
2/n x 1.

Show that (fn ) does not converge uniformly of [0, 1].


[Hint: Use termwise integration.]
5. Suppose (an ) is such that

X
an x2n
a
is
absolutely
convergent.
Show
that
n
n=1
1 + x2n

n=1

is a dominated series on R.
6. Show that for each p > 1, the series

X
xn
n=1

np

is convergent on [1, 1] and the

limit function is continuous.


7. Show that the series

{(n+1)2 xn+1 n2 xn }(1x) converges to a continuous

n=1

function on [0, 1], but it is not dominated.


8. Show that the series

X

n=1

1
1 

is convergent on [0, 1], but


1 + (k + 1)x 1 + kx

not dominated, and


Z

1X


0 n=1

X
1 
1

dx =
1 + (k + 1)x 1 + kx

n=1 0

Z
9. Show that

1X

0 n=1

x
1
dx = .
(n + x2 )2
2

1

1
1 

dx.
1 + (k + 1)x 1 + kx

7
Power Series

7.1

Convergence and Absolute convergence

Power series is a particular case of series of functions.


Definition 7.1 Given a sequence (an ) of real numbers, a series of the form
is called a power series.

n=0 an x

P
n
Note that a power series
n=0 an x converges at the point x = 0. What can we
say about its domain of convergence?
P
n
Theorem 7.1 (Abels theorem) Consider the series
n=0 an x .
(i) If the above series converges at a point x0 , then it converges absolutely at
every x with |x| < |x0 |.
(ii) If the above series diverges at a point y0 , then it diverges at every x with
|x| > |y0 |.
P
n
Proof. (i) Suppose
n=0 an x converges at a point x0 6= 0. Let x be such that
|x| < |x0 |. Then, since we have
n

n
n x
n.
|an x | = |an x0 |
x0
Since |an xn0 | 0 as n , there exists M > 0 such that |an xn0 | M for all n, so
that we have
n
x
n
n.
|an x | M
x0

P

n
Now, since xx0 < 1, it follows, by comparison test that
n=0 |an x | converges.
P
P
n
n
(ii) Suppose
n=0 an x diverges at a point y0 6= 0. If
n=0 an x converges at a
point x with |x| > |y0 |, then by part (i), it converges at y0 as well, which contradicts
our assumption. Hence, the conclusion holds.
P
n
Theorem 7.1 shows that if the series
n=0 an x converges at some nonzero point
x0 , then there it converges for all x in an interval of the form (a, a), where a |x0 |.
128

Convergence and Absolute convergence

129

Definition
7.2 The domain (or interval) of convergence of a power series
P
n is the set
a
x
n
n=0
D := {x R :

an xn converges at x}

n=0

and
R := sup{|x| : x D}
P
n
is called the radius of convergence of
n=0 an x .

P
n
Thus, a number
R
with
0
<
R
<

is
the
radius
of
convergence
of
n=0 an x if
P
and only if n=0 an xn converges for all x with |x| < R, and diverges at all x with
|x| > R.
P
n
If the power series
n=0 an x converges only at the point 0, then the radius of
convergence is 0, and if it converges at all points in R, then sup{|x| : x D} does
not exists, and in that case we say that the radius of convergence is , i.e., we write
R = .
P
n
EXAMPLE 7.1 Consider the power series
n=0 x . In this case, we know that
the series converges for x with |x| < 1, and diverges for x with |x| > 1. Also, the
series diverges at x {1, 1}. Hence, its radius of convergence is 1, and its domain
(interval) of convergence is D = {x : 1 < x < 1} = (1, 1).

P xn
EXAMPLE 7.2 Consider the power series n=0 n . In this case, we know that the
series converges at x = 1 and diverges at x = 1. Hence, its radius of convergence
is 1, and its domain (interval) of convergence is [1, 1).

P xn
EXAMPLE 7.3 Consider the power series n=0 n2 . We know that this series
converges at x = 1 and x = 1. Since
|xn+1 /(n + 1)2 |
n
= |x|
|x| as n ,
n
2
|x /n |
n+1
xn
P

by ratio test the series
n=0 n2 converges for x with |x| < 1 and diverges for x
with |x| > 1. Therefore, the radius of convergence is 1, and the domain (interval)
of convergence is [1, 1].

P xn
EXAMPLE 7.4 Consider the power series n=0 n! . Since
|xn+1 /(n + 1)!|
1
= |x|
0
n
|x /n!|
n+1

as

n ,

by ratio test the series converges at every x R. Hence, the radius of convergence
is , and the domain (interval) of convergence is R.

For finding the radius of convergence and domain of convergence, the following
theorem will be useful.

130

Power Series

M.T. Nair

CONVENTION: In the following, we use the following convention:


(i) If an 0 for all n N and an as n , then we write limn an = .
(ii) If c = 0, then we write 1/c = , and if c = , then we write 1/c = 0.
P
n
Theorem 7.2 Consider the power series
n=0 an x , and let R be its radius of
convergence.


an+1
(a) If limn an = L [0, ], then R = 1/L.
(b) If limn |an |1/n = ` [0, ], then R = 1/`.
Proof. Taking un (x) = an xn , n N, the proofs follow from Abels Theorem 7.1
and applying ration test and root test.
Exercise 7.1 Find the domain of convergence of the following power series.
P xn
P
P xn
P
xn
n
(i)
(ii)
n=1 2n1 .
n=1 n4n , (iii)
n=0 n! , (iv)
n=0 n!x .
[Answers: (i) [1, 1), (ii) [4, 4), (iii) R, (iv) {0}.]

7.2

Uniform Convergence

Theorem
7.3 Suppose R > 0 is the radius of convergence of a power series
P
n
n=0 an x . Then
P
n
(i)
n=0 an x converges uniformly on [, ] for any with 0 < < R, and
P
n
(ii) the function f defined by f (x) :=
n=0 an x , x (R, R), is continuous
on (R, R).
Proof. Let 0 < < R, and r such that < r < R. Then for every x with |x| ,
we have
 x n
 n
|an xn | |an rn |
| |an rn |
.
r
r
P
n
n
bounded, say
Since the series
n=0 an r is convergent, the sequence (an r ) is P

n
n
|an r | M for all n N, for some M >P0. Also, since r < 1,
n=0 an x is

n
a dominated on [, ]. Hence the series n=0 an x Pis uniformly convergent on

n
[, ]. Hence, the function f defined by f (x) :=
n=0 an x is continuous on
[, ]. Since, this is true for any < R, the function f is continuous at every
x (R, R). [Given any x0 (R, R), we may take such that |x0 | < < R.]

7.3

Differentiation and Integration

Theorem
7.4 Suppose R > 0 is the radius of
convergence of a power series
P
P
n . Then the radius of convergence of
a
x
an xn1 is R. In particun=0 n
P n=1 nn1
lar, for any with 0 < < R, the series n=1 n an x
converge uniformly on
[, ] and it represents a continuous function on (R, R).

Differentiation and Integration

131

Proof. Let x0 (R, R), and let such that |x0 | < < R. Then, we have
|nan xn1
|
0

n1

= n|an


|

|x0 |

n1
n N.

P
n
n1 | 0 so that it is bounded, say |a n1 | M
Since
n
n=0 an converges, |an
for all n N. Thus,
|nan xn1
|
0

Since
P

n=0 n

n=0 an

xn

|x0 |

n1


Mn

|x0 |

n1

converges, it follows that

n N.
P

n
n=0 an x0

also converges. Thus,

for every x (R, R).

P
n1 . Suppose
It
remains
to
show
that
R
is
the
radius
of
convergence
of
n=1 nan x
P
n1 converges at some point y with |y | > R. Then taking r with
0
0
n=1 nan x
P
n1 converges (by Abels theorem applied
na
r
|y0 | > r > R, we see that
n
n=1
P
n
to this series). But, |nan rn1 | |an rn |/r so that by comparison test
n=1 an r
converges.
This is not possible since r > R. Thus, the radius of convergence of
P
n1 is R.
na
x
n
n=1
Theorem
7.5 Suppose R P
> 0 is the radius of convergence of a power series
P

n
n
n=0 an x for |x| < R. Then the following hold.
n=0 an x , and let f (x) :=
(i) f (x) is a continuous function for |x| < R, and for [a, b] (R, R),
Z

f (x)dx =
a

X
an
[bn+1 an+1 ].
n+1

n=0

(ii) f (x) is differentiable for every x (R, R), and

X
d
f (x) =
nan xn1 .
dx
n=1

Proof. The result in (i) follows from Theorems 7.3 and 6.6. Also, by Theorems
7.4 and 6.7 it follows that

X
d
f (x) =
nan xn1
dx
n=1

for every x [, ], where 0 < < R. Now, if x (R, R), then we may take
such that x [, ], and the result is valid for such x as well.

132

Power Series

7.3.1

M.T. Nair

Series that can be converted into a power series

P
n
Some of the series may not be in the standard form
n=0 an x , but can be converted
into this form after some change of variable. For example, consider the series
(i)

an (x x0 )n , (ii)

n=0

an x3n , (iii)

n=0

an

X
1
,
(iv)
an sinn x.
xn
n=0

n=0

In each of these cases, we may take a new variable as follows: In (i) y = x x0 , in


(ii) y = x3 , in (iii) y = x1 , and in (iv) y = sin x.

7.4

Additional Exercises

1. Find the interval of convergence of the following power series.


(i)

X
(1)n
n=1

(iii)

n=1

nxn

(ii)

X
(1)n 3n
.
(4n 1)xn

n=1

5)n

n(x +
,
(2n + 1)3

(iv)

X
2n sinn x

n2

n=1

[Answers: (i): (, 1) [1, ); (ii): (, 3) [3, );


(iii): [6, 4]; (iv): [ 6 + k, 6 + k], k Z.]
2. Find the radius of convergence of

X
(n 1)!
n=0

nn

xn .

Ans: e

3. Find the radius of convergence and interval of convergence of the following


series:

X
X
X
n n
n2 n
(i)
x for > 0;
(ii)
x for > 0;
(iii)
xn! .
n=0

(iv)

n=0

(1)n

n=0

xn(n+1) ;

(v)

X
sin(n/6)
n=0

2n

(x 1)n ;

(vi)

n=0

X
n=0

(i)n 2n
x .
4n n

4. Show that
(i) log(1 + x) =

(1)n+1

n=1

(ii) tan1 x =

(1)n

n=0

5. Show that the series

x2n+1
for 1 < x 1,
2n + 1

X
xn
n=1

dominated on R?

xn
for 1 < x 1,
n

n!

is dominated on any closed interval [a, b]. Is it

8
Fourier Series

While studying the heat conduction problem in the year 1804, Fourier found it
necessary to use a special type of function series associated with certain functions f ,
later known as Fourier series of f . In this chapter we study such series of functions.

8.1
8.1.1

Fourier Series of 2-Periodic functions


Fourier series and Fourier coefficients

Definition 8.1 Let (an ) and (bn ) be sequences of real numbers. Then a series of
the form

X
c0 +
(an cos nx + bn sin nx)
n=1

is called a trigonometric series.

PWe observe that the functions cos nx and sin nx are 2-periodic. Hence, if c0 +
n=1 (an cos nx + bn sin nx) converges to a function f (x), the f (x) also has to be
2-periodic. Thus, only a 2-periodic function is expected to have a trigonometric
series expansion.
Definition 8.2 A function f : R R is said to be T -periodic for some T > 0 if
f (x + T ) = f (x for all x R.

Now, suppose that f is a 2-periodic function. We would like to know whether
f can be represented as a Fourier series. Suppose, for a moment, that we can write
f (x) = c0 +

(an cos nx + bn sin nx)

x R.

n=1

Then what should be an , bn ? To answer this question, let us further assume that
the series can be termwise integrated. For instance if the above series is uniformly
convergent to f , then termwise integration is possible. Recall that if (an ) and (bn )

X
are such that
(|an | + |bn |) converges, then the above series is dominated and
n=0

133

134

Fourier Series

M.T. Nair

hence is uniformly convergent. Observe that



Z
0,
cos nx cos mxdx =
,


Z
0,
sin nx sin mxdx =
,
Z

cos nx sin mxdx = 0.

if n = m
if n 6= m,
if n = m
if n 6= m,

Therefore, under the assumption that the series can be integrated termwise, we get
Z
f (x)dx = 2c0 ,

Z
f (x) cos nxdx = an n N,

Z
f (x) sin nxdx = bn n N.

Thus, c0 =

1
2

an =

f (x)dx and for all n N,


1

f (x) cos nxdx,

bn =

f (x) sin nxdx.

Definition 8.3 The Fourier series of a 2-periodic function f is the trigonometric


series

a0 X
+
(an cos nx + bn sin nx) ,
2
n=1

where

1
an =

f (x) cos nxdx,

1
bn =

f (x) sin nxdx.

We may write this fact as

a0 X
f (x)
+
(an cos nx + bn sin nx) .
2
n=1

The numbers an and bn are called the Fourier coefficients of f .

The following two theorems give sufficient conditions for the convergence of the
Fourier series of a function f to the function f at certain points x R.
Theorem 8.1 Suppose f is a bounded monotonic function on [, ). Then the
Fourier series of f converges to a function s(x), where

f (x)
if f continuous at x,
s(x) =
1
[f
(x)
+
f
(x+)]
if f not continuous at x.
2

Fourier Series of 2-Periodic functions

135

Theorem 8.2 (Dirichlets theorem) Suppose f : R R is a 2-periodic function


which is piecewise differentiable on (, ). Then the Fourier series of f converges
to the s(x) defined by

f (x)
if f continuous at x,
s(x) =
1
if f not continuous at x.
2 [f (x) + f (x+)]
Remark 8.1 It is known that there are continuous functions f defined on [, ]
whose Fourier series does not converge point wise to f . Its proof relies on concepts
from advanced mathematics (cf. M.T.Nair, Functional Analysis: A First Course,
Prentice-Hall of India, new delhi, 2002).
We may observe the following:
Suppose f is an even function. Then f (x) cos nx is an even function and
f (x) sin nx is an odd function. Hence bn = 0 for all n N, so that in this case the
Fourier series of f is

a0 X
s(x) =
+
an cos nx
2
n=1

with

2
an =

In particular,
s(0) =

an ,

f (x) cos nxdx.


0

s() = a0 +

n=0

(1)n an .

n=1

Suppose f is an odd function. Then f (x) cos nx is an odd function and


f (x) sin nx is an even function. Hence an = 0 for all n N {0}, so that in
this case the Fourier series of f is
Z

X
2
bn sin nx with bn =
f (x) sin nxdx.
()
0
n=1

In particular,
s(/2) =

(1)n b2n+1 .

n=0

8.1.2

Even and odd expansions

Suppose a function is defined on [0, ). Then we may extend it to [, ) in any


manner, and then extend to all of R periodically, that is by defining f (x2) = f (x).
In this fashion we can get many series expansions of f all of which coincide on [0, ].
For example, by defining

fodd (x) =

f (x)
f (x)

if 0 x < ,
,
if x < 0,

136

Fourier Series

M.T. Nair


feven (x) =

f (x)
f (x)

if 0 x < ,
if x < 0,

then we may observe that


fodd (x) = fodd (x),

feven (x) = feven (x) x [, ],

so that fodd and feven are the odd extension and even extension of f respectively.
Therefore,

a0 X
f (x)
+
an cos nx, x [0, ),
()
2
n=1

and
f (x)

bn sin nx,

x [0, ),

()

n=1

with

Z
Z
2
2
f (x) cos nxdx, bn =
f (x) sin nxdx.
an =
0
0
The expansions () and () are called, respectively, the even and odd expansions
of f on [0, ).
EXAMPLE 8.1 Consider the 2-periodic function f with f (x) = x for x [, ].
Note that the f is an odd function. Hence, an = 0 for n = 0, 1, 2, . . ., and the Fourier
series is

X
bn sin nx, x [0, ]
n=1

with


Z
Z
2
2 h cos nx i
cos nx
bn =
x sin nxdx =
x
dx
+
0

n
n
0
0
2 n cos n o (1)n+1 2
=

=
.

n
n
Thus the Fourier series is

X
(1)n+1
2
sin nx.
n
n=1

In particular (using Dirichlets theorem), with x = /2 we have

n=1

n=0

X (1)n+1
n X (1)n+1
=
sin
=
.
4
n
2
2n + 1

EXAMPLE 8.2 Consider the 2-periodic function f with f (x) = |x| for x
[, ]. Note that the f is an even function. Hence, bn = 0 for n = 1, 2, . . ., and
the Fourier series is
Z

a0 X
2
+
an cos nx, x [0, ], an =
x cos nxdx.
2
0
n=1

Fourier Series of 2-Periodic functions

137

It can be see that a0 = , and


a2n = 0,
Thus,
|x|

4
,
(2n + 1)2

a2n+1 =

n = 0, 1, 2, . . . .

4 X cos(2n + 1)x
,

2
(2n + 1)2

x [0, ].

n=0

Taking x = 0 (Using Dirichlets theorem), we have

2 X
1
=
.
8
(2n + 1)2
n=0


EXAMPLE 8.3 Consider the 2-periodic function f with

1, x < 0,
f (x) =
1,
0 x .
Note that the f is an odd function. Hence, an = 0 for n = 0, 1, 2, . . ., and the Fourier
series is

X
bn sin nx, x [0, ]
n=1

with

2
bn =

2
f (x) sin nxdx =

Thus

sin nxdx =
0

2
(1 cos n).

4 X sin(2n + 1)x
.
f (x)

2n + 1
n=0

Taking x = /2, we have

X (1)n
=
.
4
2n + 1
n=0


EXAMPLE 8.4 Consider the 2-periodic function f with f (x) = x2 for x
[, ]. Note that the f is an even function. Hence, bn = 0 for n = 1, 2, . . ., and
the Fourier series is
Z

a0 X
2
+
an cos nx, x [0, ], an =
x cos nxdx.
2
0
n=1

It can be see that a0 = 2 2 /3, and an = (1)n 4/n2 . Thus

X (1)n cos nx
2
x
+4
,
3
n2
2

n=1

x.

138

Fourier Series

M.T. Nair

Taking x = 0 and x = (Using Dirichlets theorem), we have

2 X (1)n+1
,
=
12
n2

2 X 1
=
6
n2

n=1

n=1

respectively.

8.2

Fourier Series of 2`-Periodic Functions

Suppose f is a T -periodic function. We may write T = 2`. Then we may consider


the change of variable t = x/` so that the function f (x) = f (`t/), as a function
of t is 2-periodic. Hence, its Fourier series is

a0 X
+
(an cos nt + bn sin nt)
2
n=1

where

 
`t
f
cos ntdt =

 
Z
1
`t
bn =
f
sin ntdt =

1
an =

1
`

1
`

f (x) cos

nx
dx,
`

f (x) sin

nx
dx.
`

`
`

In particular,
if f is even, then bn = 0 for all n and
2
an =
`

f (x) cos
0

nx
dx,
`

if f is odd, then an = 0 for all n and


2
bn =
`

8.2.1

f (x) sin
0

nx
dx,
`

Fourier series of functions on arbitrary intervals

Suppose a function f is defined in an interval [a, b). We can obtain Fourier expansion
of it on [a, b) as follows:
Method 1: Let us consider a change of variable as y = x a+b
2 . Let (y) := f (x) =
a+b
f (y + 2 ) where ` y ` with ` = (b a)/2. We can extend as a 2`-periodic
function and obtain its Fourier series as

a0 X 
n
n 
(y)
+
an cos
y + bn sin
y
2
`
`
n=1

Additional Exercises
where
an =
bn =

1
`

1
`

(y) cos

nx
ydy,
`

(y) sin

nx
ydy.
`

`
`

139

Method 2: Considering the change of variable as y = x a and ` := b a, we


define (y) := f (x) = f (y + a) where 0 y < `. We can extend as a 2`-periodic
function in any manner and obtain its Fourier series. Here are two specific cases:
(a) For y [`, 0], define fe (y) = f(y). Thus fe on [`, `] is an even function.
In this case,

a0 X
n
(y)
+
y
an cos
2
`
n=1

where ` = (b a)/2 and


1
an =
`

(y) cos
`

nx
ydy.
`

(b) For y [`, 0], define fo (y) = f(y). Thus fo on [`, `] is an odd function.
In this case,

X
n
(y)
bn sin
y
`
n=1

where
1
bn =
`

(y) sin
`

nx
ydy.
`

From the series of f we can recover the corresponding series of f on [a, b] by


writing y = x a.

8.3

Additional Exercises

1. Find the Fourier series of the 2- period function f such that:




1,
2 x< 2
(a) f (x) =

3
0, 2 < x < 2 .


x,
2 x< 2
(b) f (x) =

3
x, 2 < x < 2 .

1 + 2x
, x 0
(c) f (x) =
2x
1 , 0 x .
(d) f (x) =

x2
4 ,

x .

140

Fourier Series

M.T. Nair

2. Using the Fourier series in Exercise 1, find the sum of the following series:
1 1
1
1 1 1
(b) 1 + + +
+ . . ..
(a) 1 + + . . .,
3 5 7
4 9 16
1 1
1
1
1
1
(c) 1 +
+ . . .,
(d) 1 + 2 + 2 + 2 + . . ..
4 9 16
3
5
7

sin x, 0 x 4
3. If f (x) =
, then show that
cos x, 4 x < 2


8
sin x sin 3x sin 10x
f (x) cos
+
+
+ ... .

4 1.3
5.7
9.11
4. Show that for 0 < x < 1,


8 sin x sin 3x sin 5x
+
+
+ ... .
xx = 2

13
33
53
2

5. Show that for 0 < x < ,


sin x +

sin 3x sin 5x

+
+ ... = .
3
5
4

6. Show that for < x < ,


2
2
2
1
cos 2x +
cos 3x
cos 4x + . . . ,
x sin x = 1 cos x
2
1.3
2.4
3.5
and find the sum of the series
1
1
1
1

cos 4x +

+ ....
1.3 3.5
5.7 7.9
7. Show that for 0 x ,


cos 2x cos 4x cos 6x
2

+
+
+ ... ,
x( x) =
6
12
22
32


8 sin x sin 3x sin 5x
x( x) =
+
+
+ ... .
13
33
53
8. Assuming that the Fourier series of f converges uniformly on [, ), show
that
Z

1
a2 X 2
[f (x)]2 dx = 0 +
(an + b2n ).

2
n=1

9. Using Exercises 7 and 8 show that

X
X
1
4
(1)n1
2
(a)
=
,
(b)
=
n4
90
n2
12
n=1

X
1
6
(c)
=
n6
945
n=1

(d)

n=1

n=1

(1)n1
3
=
(2n 1)3
32

10. Write down the Fourier series of f (x) = x for x [1, 2) so that it converges to
1/2 at x = 1.

You might also like